Complete Pharm CBA 1

Pataasin ang iyong marka sa homework at exams ngayon gamit ang Quizwiz!

Which statement or statements about caffeine does the nurse identify as true? (Select all that apply.) A. Caffeine can restore mental functioning during intoxication with alcohol. B. Caffeine promotes constriction of cerebral blood vessels. C. Caffeine promotes bronchodilation. D. Caffeine is a diuretic. E. Caffeine reduces the capacity for prolonged intellectual exertion.

A, B, C, D Caffeine increases the capacity for prolonged intellectual exertion. All other statements are true.

The nurse is conducting discharge teaching related to a new prescription for phenytoin [Dilantin]. Which statements are appropriate to include in the teaching for this patient and family? (Select all that apply.)

"Be sure to call the clinic if you or your family notice increased anxiety or agitation." "You may have some mild sedation. Do not drive until you know how this drug will affect you." "It is very important to have good oral hygiene and to visit your dentist regularly."

Which statement made by a female patient newly diagnosed with complex partial seizures and starting treatment with valproic acid indicates a need for further teaching by the nurse?

"I should take the medication on an empty stomach."

Which statement by a patient taking acetaminophen indicates to the nurse the need for further teaching? -"I take acetaminophen to decrease swelling in my joints." -"This medication should decrease my pain." -"I am glad this medication will not upset my stomach." -"If I take too much of this drug, it could hurt my liver."

"I take acetaminophen to decrease swelling in my joints." Acetaminophen is not clinically useful as an anti-inflammatory or an antirheumatic drug. It does not suppress platelet aggregation, cause gastric ulceration, or cause renal impairment.

The nurse is teaching a patient newly diagnosed with epilepsy about her disease. Which statement made by the nurse best describes the goals of therapy with antiepilepsy medication?

"Our goal is to reduce your seizures to an extent that helps you live a normal life."

Which instruction would be inappropriate to include in the teaching plan for a patient being started on carbamazepine [Tegretol]?

"Take the medication with a glass of grapefruit juice each morning."

Which statement by a patient taking celecoxib indicates to the nurse the need for further teaching? -"This medicine might upset my stomach." -"This drug is ordered to decrease my pain." -"This medication will protect me from heart attacks." -"I need to have my kidneys' function tested while I am taking this drug."

"This medication will protect me from heart attacks." Celecoxib does not provide cardiovascular benefits like aspirin because it does not suppress platelet aggregation. Celecoxib is indicated for pain and like NSAIDs may cause gastric upset. It does affect renal function, and BUN and creatinine should be checked periodically.

ANS: 0.75 mL DIF: COGNITIVE LEVEL: Applying (Application) REF: N/A TOP: NURSING PROCESS: Implementation

1. A patient is to receive hydromorphone (Dilaudid) 1.5 mg IV push now. The medication comes in a prefilled syringe, 2 mg/mL. Identify how many milliliters will the nurse administer for this dose. _______

ANS: D Verifying the patient's identity, using two identifiers, before administering any medication is essential for the patient's safety and reflects checking one of the "Nine Rights" of medication administration. Documentation is done after the medications are given. DIF: COGNITIVE LEVEL: Applying (Application) REF: pp. 104-105 TOP: NURSING PROCESS: Assessment

1. Before administering any medication, what is the nurse's priority action regarding patient safety? a. Verifying orders with another nurse b. Documenting the medications given c. Counting medications in the medication cart drawers d. Checking the patient's identification using two identifiers

ANS: B, E, F In general, aqueous medications can be given with a 22- to 27-gauge needle, and average needle lengths for adults range from 1 to inches. Insert the needle at a 90-degree angle. Checking for blood return is also part of the technique for IM injections to prevent inadvertent administration into the bloodstream. The ventrogluteal site is the preferred site for IM injections in adults. The dorsogluteal site is to be avoided because of proximity to nerves and blood vessels. DIF: COGNITIVE LEVEL: Applying (Application) REF: pp. 116 TOP: NURSING PROCESS: Implementation

1. The nurse is preparing to give an aqueous intramuscular (IM) injection to an average-sized adult. Which actions are appropriate? (Select all that apply.) a. Choose a 26- or 27-gauge, - to -inch needle. b. Choose a 22- to 27-gauge, 1- to -inch needle. c. Choose the dorsogluteal site, the preferred site for IM injections for adults. d. Insert the needle at a 45-degree angle. e. Insert the needle at a 90-degree angle. f. Before injecting the medication, withdraw the plunger to check for blood return.

ANS: B When medications are added to IV fluid containers, the medication and the IV solution are mixed by holding the bag or bottle and turning it end-to-end, mixing it gently. Shaking vigorously is not appropriate; inverting the bag just once or simply allowing the bag to stand for 10 minutes may not be sufficient to mix the medication into the fluid. DIF: COGNITIVE LEVEL: Understanding (Comprehension) REF: p. 124 TOP: NURSING PROCESS: Implementation

10. When adding medications to a bag of intravenous (IV) fluid, the nurse will use which method to mix the solution? a. Shaking the bag or bottle vigorously b. Turning the bag or bottle gently from side to side c. Inverting the bag or bottle one time after injecting the medication d. Allowing the IV solution to stand for 10 minutes to enhance even distribution of medication

ANS: D Small doses of liquid medications must be withdrawn using a calibrated oral syringe. A hypodermic syringe or a syringe with a needle or syringe cap must not be used. If hypodermic syringes are used, the drug may be inadvertently given parenterally, or the syringe cap or needle, if not removed from the syringe, may become dislodged and accidentally aspirated by the patient when the syringe plunger is pressed. The other methods are not accurate for small volumes. DIF: COGNITIVE LEVEL: Applying (Application) REF: p. 109 TOP: NURSING PROCESS: Implementation

11. The nurse is measuring 4 mL of a liquid cough elixir for a child. Which method is most appropriate? a. Using a teaspoon to measure and administer b. Holding the medication cup at eye level and filling it to the desired level c. Withdrawing the elixir from the container using a syringe without a needle attached d. Withdrawing the elixir from the container using a calibrated oral syringe

ANS: A Medications that fall to the floor must be discarded, and the procedure must be repeated with new medications. The other actions are not appropriate. DIF: COGNITIVE LEVEL: Analyzing (Analysis) REF: p. 105 TOP: NURSING PROCESS: Implementation

12. While the nurse is assisting a patient in taking his medications, the medication cup falls to the floor, spilling the tablets. What is the nurse's best action at this time? a. Discarding the medications and repeating preparation b. Asking the patient if he will take the medications c. Waiting until the next dose time, and then giving the medications d. Retrieving the medications and administering them to avoid waste

ANS: C Buccal medications are properly administered between the upper or lower molar teeth and the cheek. Caution the patient against swallowing, and do not administer with water. Medications given under the tongue are sublingually administered. DIF: COGNITIVE LEVEL: Understanding (Comprehension) REF: p. 106 TOP: NURSING PROCESS: Implementation

13. When giving a buccal medication to a patient, which action by the nurse is appropriate? a. Encouraging the patient to swallow, if necessary b. Administering water after the medication has been given c. Placing the medication between the upper or lower molar teeth and the cheek d. Placing the tablet under the patient's tongue and allowing it to dissolve completely

ANS: D For PEG tubes (and nasogastric tubes), medications are poured into the barrel of the syringe with the piston removed, and the medication is allowed to flow via gravity into the tube. Fluid must never be forced into the tube. The tubing is to be flushed with 30 mL of tap water (not saline) to ensure that the medication is cleared from the tube after the medication has been given. A 3-mL syringe is too small for this procedure. DIF: COGNITIVE LEVEL: Understanding (Comprehension) REF: p. 110 TOP: NURSING PROCESS: Implementation

14. The nurse is giving medications through a percutaneous endoscopic gastrostomy (PEG) tube. Which technique is correct? a. Administering the medications using a 3-mL medication syringe b. Applying firm pressure on the syringe's piston to infuse the medication c. Flushing the tubing with 30 mL of saline after the medication has been given d. Using the barrel of the syringe, allowing the medication to flow via gravity into the tube

ANS: D Position the patient on his or her left side for rectal suppository insertion. The suppository is then lubricated with a small amount of water-soluble lubricant, not petroleum-based substances. The patient is told to take a deep breath and exhale through the mouth during insertion. Then the patient needs to remain lying on the left side for 15 to 20 minutes to allow absorption of the drug. DIF: COGNITIVE LEVEL: Applying (Application) REF: p. 111 TOP: NURSING PROCESS: Implementation

15. The nurse is about to give a rectal suppository to a patient. Which technique would facilitate the administration and absorption of the rectal suppository? a. Having the patient lie on his or her right side, unless contraindicated b. Having the patient hold his or her breath during insertion of the medication c. Lubricating the suppository with a small amount of petroleum-based lubricant before insertion d. Encouraging the patient to lie on his or her left side for 15 to 20 minutes after insertion

ANS: D When administering ophthalmic drugs that may cause systemic effects, one's finger should be protected by a clean tissue or glove and gentle pressure applied to the patient's nasolacrimal duct for 30 to 60 seconds. The other actions are not appropriate. DIF: COGNITIVE LEVEL: Applying (Application) REF: p. 130 TOP: NURSING PROCESS: Implementation

16. A patient is receiving eyedrops that contain a beta-blocker medication. The nurse will use what method to reduce systemic effects after administering the eyedrops? a. Wiping off excess liquid immediately after instilling the drops b. Having the patient close the eye tightly after the drops are instilled c. Having the patient try to keep the eye open for 30 seconds after the drops are instilled d. Applying gentle pressure to the patient's nasolacrimal duct for 30 to 60 seconds after instilling the drops

ANS: C In an infant or a child younger than 3 years of age, the ear canal is straightened by pulling the pinna down and back. In adults, the pinna is pulled up and outward. Pulling the lobe and administering eardrops without pulling on the ear lobe are not appropriate actions. DIF: COGNITIVE LEVEL: Understanding (Comprehension) REF: p. 131 TOP: NURSING PROCESS: Implementation

17. A 2-year-old child is to receive eardrops. The nurse is teaching the parent about giving the eardrops. Which statement reflects the proper technique for administering eardrops to this child? a. Administer the drops without pulling on the ear lobe. b. Straighten the ear canal by pulling the lobe upward and back. c. Straighten the ear canal by pulling the pinna down and back. d. Straighten the ear canal by pulling the pinna upward and outward.

ANS: B Position the inhaler to an open mouth, with the inhaler 1 to 2 inches away from the mouth, or attach a spacer to the mouthpiece of the inhaler, or place the mouthpiece in the mouth. To administer, press down on the inhaler to release the medication while inhaling slowly. Wait 1 to 2 minutes between puffs if a second puff of the same medication has been ordered. DIF: COGNITIVE LEVEL: Applying (Application) REF: p. 132 TOP: NURSING PROCESS: Implementation

18. A patient with asthma is to begin medication therapy using a metered-dose inhaler. What is an important reminder to include during teaching sessions with the patient? a. Repeat subsequent puffs, if ordered, after 5 minutes. b. Inhale slowly while pressing down to release the medication. c. Inhale quickly while pressing down to release the medication. d. Administer the inhaler while holding it 3 to 4 inches away from the mouth.

ANS: D Standard Precautions include wearing clean gloves when there is potential exposure to a patient's blood or other body fluids; never recapping needles; never bending needles or syringes; and discarding all disposable syringes and needles in the appropriate puncture-resistant container. DIF: COGNITIVE LEVEL: Remembering (Knowledge) REF: p. 108 TOP: NURSING PROCESS: Implementation

19. When giving medications, the nurse will use Standard Precautions, which include what action? a. Bending the needle to prevent reuse b. Recapping needles to prevent needle sticks c. Discarding all syringes and needles in the trash can d. Discarding all syringes and needles in a puncture-resistant container

ANS: B The proper size syringe for ID injection is 1-mL tuberculin. The other syringes pictured are incorrect. Insulin syringes (marked in units) are not used for intradermal injections. DIF: COGNITIVE LEVEL: Analyzing (Analysis) REF: pp. 116-117 TOP: NURSING PROCESS: Planning

2. The nurse is giving an intradermal (ID) injection and will choose which syringe for this injection?

ANS: C Sustained-release (SR) and enteric-coated tablets or capsules are forms of medications that must not be crushed before administration so as to protect the gastrointestinal lining or the medication itself. Do not break, dissolve, or crush these tablets before administering. DIF: COGNITIVE LEVEL: Applying (Application) REF: p. 106 TOP: NURSING PROCESS: Implementation

20. A patient says he prefers to chew rather than swallow his pills. One of the pills has the abbreviation SR behind the name of the medication. The nurse needs to remember which correct instruction regarding how to give this medication? a. Break the tablet into halves or quarters. b. Dissolve the tablet in a small amount of water before giving it. c. Do not crush or break the tablet before administration. d. Crush the tablet as needed to ease administration.

ANS: A Clear the nasal passages before receiving nasal spray. Blowing one's nose after receiving the medication will remove the medication from the nasal passages. The patient will receive the spray while inhaling through the open nostril and needs to remain in a supine position for 5 minutes afterward. DIF: COGNITIVE LEVEL: Applying (Application) REF: p. 136 TOP: NURSING PROCESS: Implementation

21. When administering nasal spray, which instruction by the nurse is appropriate? a. "You will need to blow your nose before I give this medication." b. "You will need to blow your nose after I give this medication." c. "When I give this medication, you will need to hold your breath." d. "You need to sit up for 5 minutes after you receive the nasal spray."

ANS: D The proper angle for IM injections is 90 degrees. The other angles are incorrect. DIF: COGNITIVE LEVEL: Remembering (Knowledge) REF: p. 116 TOP: NURSING PROCESS: Implementation

3. A patient is to receive a penicillin intramuscular (IM) injection in the ventrogluteal site. The nurse will use which angle for the needle insertion? a. 15 degrees b. 45 degrees c. 60 degrees d. 90 degrees

ANS: B Before a medication is injected by IV push, the IV line is occluded by pinching the tubing just above the injection port. The other locations are incorrect. DIF: COGNITIVE LEVEL: Understanding (Comprehension) REF: p. 128 TOP: NURSING PROCESS: Implementation

4. When administering medication by IV bolus (push), the nurse will occlude the IV line by which method? a. Not pinching the IV tubing at all b. Pinching the tubing just above the injection port c. Pinching the tubing just below the injection port d. Pinching the tubing just above the drip chamber of the infusion set

ANS: D The vastus lateralis is the preferred site of injection of drugs such as immunizations for infants. The other sites are not appropriate for infants. The ventrogluteal site is the preferred site for adults and children. The deltoid site is used only for the administration of immunizations to toddlers, older children, and adults (not infants) and only for small volumes of medication. The dorsogluteal site is no longer recommended because of the possibility of nerve injury. DIF: COGNITIVE LEVEL: Understanding (Comprehension) REF: p. 122 TOP: NURSING PROCESS: Implementation

5. The nurse has an order to administer an intramuscular (IM) immunization to a 2-month-old child. Which site is considered the best choice for this injection? a. Deltoid b. Dorsogluteal c. Ventrogluteal d. Vastus lateralis

A child is to receive phenobarbital 2 mg/kg IV on call as a preoperative sedative. The child weighs 64 pounds. How many milligrams will the child receive for this dose?

58.2 mg

ANS: C The proper technique for a subcutaneous injection for an obese patient is to pinch the skin at the site and inject the needle to below the skin fold at a 90-degree angle. DIF: COGNITIVE LEVEL: Applying (Application) REF: p. 116 TOP: NURSING PROCESS: Implementation

6. The nurse needs to administer insulin subcutaneously to an obese patient. Which is the proper technique for this injection? a. Using the Z-track method b. Inserting the needle at a 5- to 15-degree angle until resistance is felt c. Pinching the skin at the injection site, and then inserting the needle to below the tissue fold at a 90-degree angle d. Spreading the skin tightly over the injection site, inserting the needle, and then releasing the skin

ANS: A The Z-track method is used for medications known to irritate tissues or for medications that are painful or cause stains to the tissues. It also prevents the deposit of medication into more sensitive subcutaneous tissues. The other options are not appropriate situations for the Z-track method. DIF: COGNITIVE LEVEL: Understanding (Comprehension) REF: p. 120 TOP: NURSING PROCESS: Planning

7. The nurse will plan to use the Z-track method of intramuscular (IM) injections for which situation? a. The medication is known to be irritating to tissues. b. The patient is emaciated and has very little muscle mass. c. The medication must be absorbed quickly into the tissues. d. The patient is obese and has a deep fat layer below the muscle mass.

ANS: C The formation of a small bleb is expected after an ID injection for skin testing. The other actions are not appropriate. DIF: COGNITIVE LEVEL: Applying (Application) REF: p. 116 TOP: NURSING PROCESS: Implementation

8. After administering an intradermal (ID) injection for a skin test, the nurse notices a small bleb at the injection site. The best action for the nurse to take at this time is to: a. apply heat. b. massage the area. c. do nothing. d. report the bleb to the physician.

ANS: A IV locks are to be flushed before and after each use; either heparin or saline flush is used, depending on the individual institution's policy. The other actions are not appropriate. DIF: COGNITIVE LEVEL: Applying (Application) REF: p. 128 TOP: NURSING PROCESS: Implementation

9. The nurse is administering an IV push medication through an IV lock. After injecting the medication, which action will be taken next? a. Flushing the lock b. Regulating the IV flow c. Clamping the tubing for 10 minutes d. Holding the patient's arm up to improve blood flow

The nurse is teaching a pregnant patient about the effects of medication on fetal development. The nurse understands the greatest risk for medication effects on developing fetuses occurs during which time period? A) First trimester B) Third trimester C) Birthing process D) Second trimester

A) First trimester

A nurse working with older adult patients is concerned about the number of medications prescribed for each patient. Which older adult assessment should be of highest priority related to polypharmacy? A) Nonadherence to drug regimen B) Cost of medications C) Drug interactions D) Schedule of medications

C) Drug interactions

A parent thinks a school-aged child has ADHD. The nurse asks the parent to describe the child's behaviors. Which behaviors are characteristic of ADHD? (Select all that apply.) a. Anxiety b. Compulsivity c. Hyperactivity d. Inattention e. Impulsivity

C, D, E: Hyperactivity / Inattention / Impulsivity ADHD is characterized by inattention, hyperactivity, and impulsivity. Anxiety and compulsivity are not characteristic of ADHD.

Which drug does the nurse identify as a selective serotonin reuptake inhibitor? (Select all that apply.) A. Bupropion (Wellbutrin) B. Imipramine (Tofranil) C. Fluoxetine (Prozac) D. Desvenlafaxine (Pristiq) E. Sertraline (Zoloft)

C, E Fluoxetine (Prozac) and sertraline (Zoloft) are selective serotonin reuptake inhibitors. Bupropion (Wellbutrin) is an atypical antidepressant. Imipramine (Tofranil) is a tricyclic antidepressant. Desvenlafaxine (Pristiq) is a serotonin/norepinephrine reuptake inhibitor (SNRI).

The nurse is caring for a patient taking buspirone (BuSpar). Which statement by the patient indicates a need for further teaching about this drug? A. "This medication should not make me feel drowsy." B. "This medication should help me feel less anxious." C. "I will drink grapefruit juice instead of coffee with breakfast." D. "I will take my medication three times per day."

C. "I will drink grapefruit juice instead of coffee with breakfast." Grapefruit juice can greatly increase buspirone levels and should be avoided. The other statements are appropriate.

A patient with Parkinson's disease who has been positively responding to drug treatment with levodopa/carbidopa [Sinemet] suddenly develops a relapse of symptoms. Which explanation by the nurse is appropriate? A. "You have apparently developed resistance to your current medication and will have to change to another drug." B. "This is an atypical response. Unfortunately, there are no other options of drug therapy to treat your disease." C. "This is called the 'on-off' phenomenon. Your healthcare provider can change your medication regimen to help diminish this effect." D. "You should try to keep taking your medication at the current dose. These effects will go away with time."

C. "This is called the 'on-off' phenomenon. Your healthcare provider can change your medication regimen to help diminish this effect." Patients who have been taking levodopa/carbidopa for a period of time may experience episodes of symptom return. Adding other medications to the drug regimen can help minimize this phenomenon.

The healthcare provider orders entacapone 400 mg PO every 6 hours. The nurse notes that the total dose given in a 24-hour period would be what amount? A. 800 mg B. 1400 mg C. 1600 mg D. 3200 mg

C. 1600 mg 24 hours divided by every-6-hour doses equals 4 doses of 400 mg. 400 mg times 4 doses equals 1600 mg.

The nurse is caring for a patient taking lithium (Lithobid). The nurse understands that many drugs interact with lithium. Which agent is safe to administer with lithium? A. Ibuprofen (Motrin) for muscle pain B. Hydrochlorothiazide (HCTZ) for edema C. Aspirin (ASA) for mild headache D. Diphenhydramine (Benadryl) for cold symptoms

C. Aspirin (ASA) for mild headache Aspirin is safe to use as an analgesic with lithium. Other nonsteroidal anti-inflammatory drugs (NSAIDs), such as ibuprofen, can increase lithium levels by as much as 60%. Diuretics increase lithium levels by reducing the serum sodium level. Diphenhydramine has anticholinergic properties and can aggravate lithium-induced polyuria by causing urinary hesitancy.

The nurse is caring for a patient receiving buspirone (BuSpar) for the treatment of anxiety. Which symptom is most likely explained as an adverse effect of this drug? A. Drowsiness B. Risk for abuse C. Dizziness D. Weight gain

C. Dizziness Buspirone is an antianxiety medication with few side effects. The most common effects are dizziness, nausea, headache, nervousness, lightheadedness, and excitement. Buspirone does not cause drowsiness, risk for abuse, or weight gain.

A patient with a history of Parkinson's disease treated with selegiline [Eldepryl] has returned from the operating room after an open reduction of the femur. Which physician order should the nurse question? A. Decaffeinated tea, gelatin cubes, and ginger ale when alert B. Docusate 100 mg orally daily C. Meperidine 50 mg IM every 4 hours as needed for pain D. Acetaminophen 650 mg every 6 hours as needed for temperature

C. Meperidine 50 mg IM every 4 hours as needed for pain Selegiline can have a dangerous interaction with meperidine, leading to stupor, rigidity, agitation, and hyperthermia; therefore, this order should be questioned. Foods that contain tyramine should be restricted, but there is no contraindication to the fluids that have been ordered. Docusate and acetaminophen are not contraindicated for use with selegiline.

A patient has been diagnosed with performance anxiety. The nurse anticipates use of which drug to treat this psychologic disorder? A. Clonazepam (Klonopin) B. Alprazolam (Xanax) C. Propranolol (Inderal) D. Sertraline (Zoloft)

C. Propranolol (Inderal) Propranolol (Inderal) and other beta blockers can benefit patients with performance anxiety. When taken 1 to 2 hours before a scheduled performance, beta blockers can reduce symptoms caused by autonomic hyperactivity (e.g., tremors, sweating, tachycardia, palpitations). Doses are relatively small (e.g., only 10 to 80 mg for propranolol).

The nurse is planning care for a patient taking imipramine (Tofranil). Which finding, if present, would most likely be an adverse effect of this drug? A. Blood pressure of 160/90 mm Hg B. Insomnia and diarrhea C. Sedation and dry mouth D. Tachypnea and wheezing

C. Sedation and dry mouth Anticholinergic effects (dry mouth, blurred vision, constipation, tachycardia, urinary retention) and sedation are potential adverse effects of the tricyclic antidepressants (TCAs), such as imipramine (Tofranil). The most serious common adverse effect is orthostatic hypotension; therefore, a blood pressure of 160/90 mm Hg probably is not caused by this drug. Respiratory problems are not commonly associated with the TCAs.

A patient is started on a trial of dextroamphetamine/amphetamine (Adderall) for ADHD. What is the most likely nursing diagnosis for inclusion in the plan of care? A. Decreased cardiac output B. Imbalanced nutrition, more than body requirements C. Sleep pattern disturbance D. Diversional activity deficit

C. Sleep pattern disturbance Insomnia is a common adverse effect of CNS stimulants.

What is the goal of pharmacologic therapy in the treatment of Parkinson's disease? A. To increase the amount of acetylcholine at the presynaptic neurons B. To reduce the amount of dopamine available in the substantia nigra C. To balance cholinergic and dopaminergic activity in the brain D. To block dopamine receptors in presynaptic and postsynaptic neurons

C. To balance cholinergic and dopaminergic activity in the brain Parkinson's disease results from a decrease in dopaminergic (inhibitory) activity, leaving an imbalance with too much cholinergic (excitatory) activity. With an increase in dopamine, the neurotransmitter activity becomes more balanced, and symptoms are controlled.

Which agent is most likely to be prescribed today for short-term management of insomnia? A. Secobarbital (Seconal Sodium) B. Meprobamate (Miltown) C. Zolpidem (Ambien) D. Flumazenil (Romazicon)

C. Zolpidem (Ambien) Zolpidem is a benzodiazepine-like drug that is widely used in the treatment of insomnia. It is safer than the barbiturates (secobarbital) or miscellaneous sedative-hypnotics (meprobamate). Flumazenil is a reversal agent for the benzodiazepines.

The nurse receives a laboratory report indicating that the phenytoin [Dilantin] level for the patient seen in the clinic yesterday is 16 mcg/mL. Which intervention is most appropriate?

Continue as planned, because the level is within normal limits.

Which drug used for the management of the patient with Parkinson's disease is most likely to cause postural hypotension? A. amantadine (Symmetrel) B. selegiline (Eldepryl) C. tolcapone (Tasmar) D. entacapone (Comtan)

Correct answer: A Rationale: Amantadine (Symmetrel), carbidopa-levodopa, and ropinirole (Requip) are most likely to cause postural hypotension.

A patient wants to take orlistat (Xenical) to assist in her weight loss program, but she is wary of its unpleasant adverse effects. What measure can be suggested to reduce these effects? A. Restrict dietary intake of fat B. Restrict dietary intake of fiber C. Increase intake of dairy products D. Avoid intake of carbonated beverages

Correct answer: A Rationale: Restricting dietary intake of fat to less than 30% of total calories can help reduce some of the GI adverse effects of orlistat, which include oily spotting, flatulence, and fecal incontinence in 20% to 40% of patients.

The beneficial role of the NDDRA ropinirole (Requip) is that it A. appears to delay the start of levodopa therapy. B. allows for levodopa therapy to begin earlier. C. improves the patient's tolerance of D. parkinsonian symptoms. replaces dopamine in the brain.

Correct answer: A Rationale: The longer that levodopa therapy can be delayed, the longer the benefit of the therapy will be experienced once it is eventually started.

Prior to administering a serotonin agonist, it is most important for the nurse to assess the patient for a history of A. hypertension. B. allergy to penicillin. C. chronic bronchitis. D. cataracts.

Correct answer: A Rationale: The nurse should complete a thorough cardiac history as well as measurement of blood pressure and pulse rate and rhythm. If a patient has a history of hypertension, there is risk of further increases in blood pressure to dangerous levels with use of these drugs, and thus the need for careful assessment and documentation. In fact, generally these drugs are not prescribed for patients with migraines who also have coronary artery disease unless a thorough cardiac evaluation has been performed.

A patient in a long-term care facility has a new order for carbamazepine (Tegretol) for seizure management. The nurse monitors for autoinduction, which will result in A. toxic levels of carbamazepine (Tegretol). B. lower than expected drug levels. C. gingival hyperplasia. D. cessation of seizure activity.

Correct answer: B Rationale: Carbamazepine (Tegretol) is associated with autoinduction of hepatic enzymes. Autoinduction is a process in which, over time, a drug stimulates the production of enzymes that enhance its own metabolism, which leads to lower than expected drug concentrations.

A patient with unstable epilepsy is receiving IV doses of phenytoin (Dilantin). The latest drug level is 12 mcg/mL. Which administration technique will the nurse use? A. Administer the drug by rapid IV push B. Infuse slowly, not exceeding 50 mg/min C. Mix the medication with dextrose solution D. Administer via continuous infusion

Correct answer: B Rationale: Phenytoin should be mixed only with normal saline, and it should be given by slow IV infusion (but not as a continuous infusion).

The "off-on phenomenon" that some patients with Parkinson's disease (PD) experience is best explained as the A. need to take a drug holiday to improve response to medications. B. variable response to levodopa, resulting in periods of good control and periods of poor control of PD symptoms. C. alternating schedule of medications needed to control PD. D. fluctuation of emotions that often occurs with PD.

Correct answer: B Rationale: Some patients who take levodopa on a long-term basis experience times when their PD symptoms are under control and other times when symptoms are not well controlled.

The nurse is assessing the current medication list of a newly admitted patient. The drug gabapentin (Neurontin) is listed, but the patient states that he does not have any problems with seizures. The nurse suspects that the patient A. is unaware of his own disease history. B. has been taking his wife's medication by mistake. C. may be taking this drug for neuropathic pain. D. is reluctant to admit to having a seizure disorder.

Correct answer: C Rationale: Gabapentin is commonly used to treat neuropathic pain.

A teenaged boy will be receiving atomoxetine (Strattera) as part of treatment for attention deficit hyperactivity disorder (ADHD). Which statement about this drug therapy is accurate? A. Strattera is highly addictive. B. Psychotherapy is rarely helpful in cases of ADHD. C. The patient should be monitored for possible suicidal thoughts and behavior. D. Strattera is used to treat narcolepsy as well as ADHD.

Correct answer: C Rationale: Prescribers are advised to work with parents to monitor closely for suicidal thoughts and behavior. In addition, psychosocial problems within the patient's family should be addressed if needed. Strattera is not addictive, and it is not used to treat narcolepsy.

Which information will the nurse provide to the patient who is receiving antiepileptic drug therapy? A. If you feel sleepy when taking the drug, decrease the dose by one half B. Take the drug on an empty stomach C. Call your health care provider if you experience a sore throat or fever D. Patients with epilepsy are not able to hold a job and work, so you should apply for benefits

Correct answer: C Rationale: To prevent complications, patients should be taught to call the health care provider if they experience fever, sore throat, excessive bleeding or bruising, and new onset of nosebleeds. Drowsiness is a common side effect of these drugs; the dose should never be altered without consulting the prescriber. These medications should be taken with food to decrease GI upset. Most patients with seizure disorders are able to work and are protected by the Americans with Disabilities Act.

A patient is prescribed an anorexiant. Which statement will the nurse include in patient teaching? A. "Take the medication with your evening meal." B. "You will need to take this drug for at least 2 years." C. "If you develop a dry mouth, stop taking the drug immediately." D. "Avoid intake of caffeine."

Correct answer: D Rationale: Caffeine in any form must be avoided by patients taking anorexiants. These medications should be taken in the morning to prevent interference with sleep. The drugs are taken on a short-term basis. Dry mouth frequently develops and can be managed by sucking ice chips and keeping a bottle of water nearby at all times. These drugs should not be abruptly stopped as a rebound increase of appetite may develop.

When providing teaching to a patient receiving an anticholinergic for the treatment of Parkinson's disease, the nurse will include which information? A. Take the medication first thing in the morning. B. Limit fluid intake when taking this drug. C. The tremors you experience will be reduced within 24 hours of taking this drug. D. Do not take this medication at the same time as other medications.

Correct answer: D Rationale: When anticholinergics are used for the treatment of Parkinson's disease, these medications should not be taken at the same time as other mediations. The medications should be administered at bedtime. Fluid intake should not be restricted as they cause dry mouth, and it may take several days to weeks for the beneficial effects of the medication to become evident.

A mother of a 1-month-old infant calls the clinic and asks the nurse if the medication she is taking can be passed to her infant during breastfeeding. What is the nurse's best response to the mother's question? A) "You should not take any medication while breastfeeding." B) "Only certain medications pass to infants while breastfeeding." C) "I will leave the health care provider a message to return your call." D) "Drugs can cross from mother to infant in breast milk, so it will depend on the drug you are taking."

D) "Drugs can cross from mother to infant in breast milk, so it will depend on the drug you are taking."

When calculating pediatric dosages, the nurse understands which method is MOST accurate for dosing calculations? A) Calculated doses based on body weight need to be increased by 10% because of immature renal and hepatic function B) Medication dosing calculated according to body weight because it is based on maturational growth and development C) Use of drug reference recommendations based on mg/kg of body weight. D) Dosage calculation by body surface area because it takes into account the difference in size for children and neonates

D) Dosage calculation by body surface area because it takes into account the difference in size for children and neonates

The physiologic changes that normally occur in older adult patients have which implication for drug response? A) Protein binding is more efficient. B) Drug metabolism is quicker. C) Drug elimination is faster. D) Drug half-life is lengthened.

D) Drug half-life is lengthened.

Which statement or statements about zaleplon (Sonata) does the nurse identify as true? (Select all that apply.) A. Zaleplon (Sonata) is a benzodiazepine. B. Zaleplon (Sonata) is indicated for long-term management of insomnia. C. Zaleplon (Sonata) is used to maintain sleep throughout the night. D. Zaleplon (Sonata) should not be administered with cimetidine (Tagamet). E. Zaleplon (Sonata) interacts with the neurotransmitter GABA.

D, E Zaleplon (Sonata) belongs to a new class of drugs called the pyrazolopyrimidines. It is indicated for short-term management of insomnia. Zaleplon has a rapid onset and short duration of action and therefore is beneficial for initiating sleep but not for maintaining it throughout the night. Zaleplon and cimetidine should not be administered together. Zaleplon interacts with the neurotransmitter GABA.

A family member asks the nurse about amantadine. Which statement by the nurse is the most helpful in explaining the use of amantadine? A. "Amantadine was developed as an antiviral agent but is now used for treatment of PD." B. "Amantadine works slowly over time but can lose its effectiveness in 3 to 6 months." C. "Amantadine works rapidly and does not lose its effectiveness." D. "Amantadine is not as effective as some other medications, so it is not a first-line treatment, but it may be used in addition to other medications."

D. "Amantadine is not as effective as some other medications, so it is not a first-line treatment, but it may be used in addition to other medications." Amantadine was developed as an antiviral agent, but this response is not helpful to explain the use of the medication to the family member. Medication effects develop rapidly—often within 2 to 3 days—but are much less profound than with levodopa or the dopamine agonists. Furthermore, effects may begin to diminish within 3 to 6 months. Amantadine is not considered a first-line agent. However, the drug may be helpful for managing dyskinesias caused by levodopa.

The nurse is monitoring a patient with depression in the early phase of treatment with amitriptyline (Elavil). Which question is most important for the nurse to ask the patient? A. "Have you noticed dry mouth or blurred vision?" B. "Have you had any changes in your urine function?" C. "When was your last bowel movement?" D. "Have you had any changes in your mood or anxiety level?"

D. "Have you had any changes in your mood or anxiety level?" In the early phase of treatment for depression, suicide risk may increase. Patients should be monitored closely for worsening mood, unusual changes in behavior, and suicide risk. The other questions would be useful in assessing the patient for adverse effects of amitriptyline (Elavil), but assessing suicide risk is the most important intervention.

A nurse assesses a patient receiving haloperidol (Haldol). The nurse notices that the patient is shifting in the chair, rocking back and forth, and tapping both feet constantly. What is the most accurate term to document these findings? A. Dystonia B. Tardive dyskinesia C. Parkinsonism D. Akathisia

D. Akathisia Haloperidol is a traditional antipsychotic medication with the adverse effects of extrapyramidal symptoms. Akathisia, or motor restlessness, is an extrapyramidal symptom. Dystonia manifests as severe spasm of the muscles of the tongue, face, neck, or back and may include upward deviation of the eyes, severe cramping, and impaired respiration. Tardive dyskinesia presents with involuntary twisting, writhing, wormlike movements of the tongue and face, lip smacking, and tongue flicking. Parkinsonism appears with bradykinesia, masklike facies, drooling, tremor, rigidity, shuffling gait, and stooped posture.

The nurse is caring for a patient with severe generalized anxiety disorder. Which agent would be most effective for immediate stabilization? A. Venlafaxine (Effexor) B. Buspirone (BuSpar) C. Paroxetine (Paxil) D. Alprazolam (Xanax)

D. Alprazolam (Xanax) Alprazolam, a benzodiazepine, would provide the most rapid onset of relief. Buspirone, paroxetine, and venlafaxine are also first-line agents for the treatment of generalized anxiety disorder, but their onset is delayed. They are preferred for long-term management.

A nurse assesses a patient who takes a maintenance dose of lithium carbonate (Lithobid) for bipolar disorder. The patient complains of hand tremor, nausea, vomiting, and diarrhea. The patient's gait is unsteady. The patient most likely has done what? A. Consumed some foods high in tyramine B. Not taken the lithium as directed C. Developed tolerance to the lithium D. Developed lithium toxicity

D. Developed lithium toxicity Early lithium toxicity is evidenced by diarrhea, anorexia, muscle weakness, nausea, vomiting, tremors, slurred speech, and drowsiness. Later signs include blurred vision, seizures, trembling, confusion, and ataxia.

Alprazolam (Xanax) is prescribed for an adult with panic attacks. The nurse recognizes that this drug exerts its therapeutic effect by interacting with which neurotransmitter? A. Norepinephrine B. Acetylcholine C. Serotonin (5-HT) D. Gamma-aminobutyric acid (GABA)

D. Gamma-aminobutyric acid (GABA) Alprazolam is a benzodiazepine; this class of drugs reduces anxiety by potentiating the action of GABA.

The nurse is preparing to administer phenelzine (Nardil) to a patient with depression. Why is this drug considered a second- or third-line agent in the treatment of depression? A. It increases the risk of suicide in the early phase. B. It is less effective than the tricyclic antidepressants. C. It increases the risk of psychoses and parkinsonism. D. It has more side effects and drug interactions.

D. It has more side effects and drug interactions. Phenelzine (Nardil), a monoamine oxidase inhibitor (MAOI), is considered a second- or third-line treatment because of the risk of triggering hypertensive crisis when the patient eats foods high in tyramine. Also, an increased incidence of drug-drug interactions is seen with phenelzine. Phenelzine does not pose an increased risk for suicide, psychoses, or parkinsonism, and it is as effective as the tricyclic and SSRI antidepressants.

The nurse is caring for a group of patients who have been prescribed sedative-hypnotic agents. Which agent has the greatest abuse potential? A. Diazepam (Valium) B. Triazolam (Halcion) C. Zolpidem (Ambien) D. Phenobarbital (Luminal Sodium)

D. Phenobarbital (Luminal Sodium) Phenobarbital is a barbiturate drug that carries a higher abuse potential than the benzodiazepine or benzodiazepine-like drugs.

The nurse is caring for a patient taking dextroamphetamine (DextroStat). Which symptom, if present, is most likely an adverse effect of this drug? A. Heart rate of 60 beats per minute B. Respiratory rate of 10 breaths per minute C. Weight gain D. Restlessness

D. Restlessness The adverse effects of amphetamines include central nervous system (CNS) stimulation (insomnia, restlessness, talkativeness), weight loss, cardiac stimulation (dysrhythmias, angina, hypertension), and psychosis (paranoia).

The nurse is caring for a patient receiving clozapine (Clozaril). Which assessment finding is most indicative of an adverse effect of this drug? A. Blood urea nitrogen level of 25 mg/dL B. Blood glucose level of 60 mg/dL C. Bilirubin level of 2.5 mg/dL D. White blood cell (WBC) count of 2000/mm3

D. White blood cell (WBC) count of 2000/mm3 Clozapine, an atypical antipsychotic, carries a risk of fatal agranulocytosis. For this reason, the WBC count should be monitored and should be greater than 3500/mm3. Renal function (blood urea nitrogen) should not be affected by clozapine. Clozapine may cause metabolic effects, including diabetes, that would result in an increased blood glucose level (greater than 110 mg/dL). Elevated bilirubin indicates liver disease and is not commonly an adverse effect of clozapine.

A female patient who is originally from Thailand is seen in the clinic for seizure control. She receives a new prescription for carbamazepine [Tegretol]. Before the patient takes the drug, which is the most appropriate initial nursing intervention?

Ensure that genetic testing for HLA-B*1502 is performed.

The nurse monitors the patient receiving ketorolac for which adverse effect? GI bleeding Respiratory depression Abuse potential Tolerance

GI bleeding Ketorolac has many of the same adverse effects of NSAIDs, including GI bleeding and renal impairment. However, it does not have the adverse effects associated with opioid administration; it does not cause respiratory depression or tolerance and does not have abuse potential.

The nurse recognizes what as responsible for body temperature regulation? Medulla Brainstem Hypothalamus Pituitary gland

Hypothalamus Body temperature is regulated by the hypothalamus, which maintains a balance between heat production and heat loss. The increase in set point is triggered by local synthesis of prostaglandins in response to endogenous pyrogens.

The nurse would question which drug if it were ordered for a patient receiving aspirin for its antiplatelet effects? Acetaminophen Coumadin Ibuprofen Ketorolac

Ibuprofen Ibuprofen can block the antiplatelet effects of aspirin. For that reason, in patients taking low-dose aspirin to prevent thrombosis, ibuprofen should not be administered.

The nurse correlates aspirin's therapeutic effects in a patient with primary dysmenorrhea with its ability to do what? Suppress the inflammatory process Promote endometrial sloughing Prevent platelet aggregation Inhibit prostaglandin synthesis

Inhibit prostaglandin synthesis Aspirin inhibits prostaglandin synthesis. Because prostaglandins cause uterine contraction, aspirin's inhibition of prostaglandins relieves cramping by minimizing uterine contractions.

Which drug should be used with caution in a patient with first-degree atrioventricular (AV) heart block?

Lacosamide [Vimpat]

Which medication should the nurse anticipate administering to a patient in convulsive status epilepticus to halt seizure activity?

Lorazepam [Ativan] 0.1 mg/kg IV at a rate of 2 mg/min

The nurse is assessing a patient receiving valproic acid [Depakene] for potential adverse effects associated with this drug. What is the most common problem with this drug?

Nausea, vomiting, and indigestion

In comparing cyclooxygenase inhibitors, the nurse recognizes that only aspirin can do what? Relieve pain Suppress inflammation Protect against stroke and MI Treat increased body temperature

Protect against stroke and MI Cyclooxygenase inhibitors, including aspirin, are used for suppression of inflammation, relief of pain, and reduction of fever. However, only aspirin is indicated for protection against myocardial infarction and stroke.

The nurse recognizes that inhibition of COX-1 results in what? Suppression of inflammation Protection against MI and stroke Alleviation of pain Reduction of fever

Protection against MI and stroke Inhibition of COX-1 results in only one positive effect, reduction in platelet aggregation, which provides protection against myocardial infarction and stroke. Inhibition of COX-2 results largely in beneficial effects, including suppression of inflammation, alleviation of pain, and reduction of fever.

What action does the nurse recognize for cyclooxygenase-1? Protects gastric mucosa Mediates fever Prevents platelet aggregation Suppresses the inflammatory response

Protects gastric mucosa Cyclooxygenase-1 protects the gastric mucosa, supports renal function, and promotes platelet aggregation. COX-2 agents mediate inflammation, sensitize pain receptors, and support renal function.

The nurse is caring for a patient receiving phenytoin [Dilantin] for treatment of tonic-clonic seizures. Which symptoms, if present, would indicate an adverse effect of this drug? (Select all that apply.)

Swollen, tender gums Measles-like rash Unusual hair growth

A. Question the order for further instructions. The nurse should question the order for further instructions, including the dose to be administered, the location, and the use of epinephrine. The nurse should not administer, refuse to administer, or suggest the use of another medication without further information.

The healthcare provider orders lidocaine inj SC before a skin biopsy. Which action should the nurse take? A. Question the order for further instructions. B. Administer the lidocaine. C. Refuse to administer the lidocaine. D. Suggest the use of procaine.

A. Question the order for benzocaine. Topical benzocaine can cause methemoglobinemia, a blood disorder in which hemoglobin is modified such that it cannot release oxygen to tissues. If enough hemoglobin is converted to methemoglobin, death can result. Methemoglobinemia has been associated with benzocaine liquids, sprays, and gels. Because of this risk, topical benzocaine should not be used in children under the age of 2 years without the advice of a healthcare professional.

The healthcare provider orders topical benzocaine spray for a 1-year-old child with a sunburn. Which action should the nurse take? A. Question the order for benzocaine. B. Question the order for further directions. C. Teach the parent about application of the spray. D. Apply the spray to the child.

B. Monitoring the patient's blood pressure throughout the epidural infusion

The nurse is caring for a patient who is receiving lidocaine [Xylocaine] by epidural injection. Which nursing intervention is most important when caring for this patient? A. Keeping the patient in a supine position for about 12 hours B. Monitoring the patient's blood pressure throughout the epidural infusion C. Preparing a double tourniquet for use during the infusion D. Reducing the intravenous (IV) infusion rate to prevent hypertension

C. To delay systemic absorption of the anesthetic

The nurse is caring for a patient who is to receive a local anesthetic with lidocaine and epinephrine. What is the primary purpose of the epinephrine? A. To reduce the risk of an allergic reaction B. To improve transport of anesthetic into the axon C. To delay systemic absorption of the anesthetic D. To suppress excitability of the myocardium

A. Hypotension B. Headache D. Urinary retention The most significant adverse effect of spinal anesthesia is hypotension. Autonomic blockade may disrupt function of the intestinal and urinary tracts, causing fecal incontinence and either urinary incontinence or urinary retention. Spinal anesthesia frequently causes headache. Nausea/vomiting and tachycardia are not expected adverse effects of spinal anesthesia with bupivacaine.

The nurse is caring for a postoperative patient who had spinal anesthesia with bupivacaine. The nurse knows to monitor for which possible adverse effects? (Select all that apply.) A. Hypotension B. Headache C. Nausea/vomiting D. Urinary retention E. Tachycardia

A. Pain

The nurse is explaining differential sensitivity to the nursing student with regard to the use of local anesthetics. The nurse explains that which perception is lost first? A. Pain B. Cold C. Warmth D. Touch

B. Sodium channels in the axonal membrane

The nurse knows that local anesthetics stop axonal conduction by blocking what? A. Potassium channels in the axonal membrane B. Sodium channels in the axonal membrane C. Calcium channels in the axonal membrane D. Protein channels in the axonal membrane

C. Excitation followed by depression

The nurse should monitor for which central nervous system (CNS) adverse effect that can result when sufficient amounts of local anesthetics are absorbed systemically? A. Hallucinations and nightmares B. Tremors of the lower extremities C. Excitation followed by depression D. Vertigo and nausea/vomiting

The nurse suspects that a female patient is experiencing phenytoin toxicity if which manifestation is noted? (Select all that apply.)

The patient is walking with a staggering gait. The patient complains of double vision. The nurse observes rapid back-and-forth movement of the patient's eyes.

Which medication can cause this acid-base disturbance: pH 7.32, paCO2 33, HCO3 20? {Metabolic Acidosis}

Topiramate [Topamax]

The nurse recognizes that aspirin is contraindicated in patients with what condition? Vitamin K deficiency Deep vein thrombosis History of MI Pregnancy

Vitamin K deficiency Aspirin is contraindicated in patients with bleeding disorders, such as vitamin K deficiency, because it is irritating to the gastric mucosa, and its platelet antiaggregate actions could make the bleeding worse.

A. Cocaine causes intense vasoconstriction.

Which of the following statements about cocaine is true? A. Cocaine causes intense vasoconstriction. B. Cocaine has no medicinal applications. C. Cocaine is a poor local anesthetic. D. Cocaine has no CNS effects.

A. Procaine [Novocain]

Which of these local anesthetic agents is most likely to cause an allergic response? A. Procaine [Novocain] B. Lidocaine [Xylocaine] C. Bupivacaine [Marcaine] D. Ropivacaine [Naropin]

The nurse will monitor the patient who is taking a muscle relaxant for which adverse effect? a) CNS depression b) hypertension c) peripheral edema d) blurred vision

a) CNS depression

A nurse is administering morphine sulfate to a postoperative patient. Which are appropriate routine nursing actions when giving this drug? (Select all that apply.) a. Counting respirations before and after giving the medication b. Encouraging physical activity and offering increased fluids c. Monitoring the patient's blood pressure closely for hypertension d. Palpating the patient's lower abdomen every 4 to 6 hours e. Requesting an order for methylnaltrexone [Relistor] to prevent constipation

a, b, d a. Counting respirations before and after giving the medication b. Encouraging physical activity and offering increased fluids d. Palpating the patient's lower abdomen every 4 to 6 hours Respiratory depression, constipation, and urinary retention are common adverse effects of opioid analgesics. It is important to count respirations before giving the drug and periodically thereafter to make sure that respiratory depression has not occurred. Increased physical activity, increased fluid intake, and increased fiber help alleviate constipation. It is important to assess the patient's abdomen and palpate the bladder to make sure that urinary retention has not occurred. Patients taking morphine often experience hypotension, not hypertension. Methylnaltrexone is given as a last resort to treat constipation, because it blocks mu receptors in the intestine.

A nursing student asks a nurse to discuss alcoholism and alcohol use disorder. Which statement by the nurse is correct? a. "Alcohol use disorder can occur without the development of tolerance or physical dependence." b. "Individuals with alcohol use disorder develop cross-tolerance with opioid analgesics." c. "Initial symptoms of abstinence syndrome occur within 1 to 2 hours after withdrawal of alcohol." d. "With severe alcoholism, most alcoholics have delirium tremens when alcohol is withdrawn."

a. "Alcohol use disorder can occur without the development of tolerance or physical dependence."

A patient who has just found out she is pregnant tells the nurse she wants to quit smoking. She asks about pharmacologic aids to help her quit. The nurse is correct to tell her what? a. "Nicotine replacement therapy is harmful, but it is safer than smoking, so it can be used." b. "Psychosocial support is the only recommended treatment for smoking cessation during pregnancy." c. "Varenicline [Chantix] is safe to use during pregnancy." d. "You should try to taper off your smoking gradually, because none of the drugs are safe."

a. "Nicotine replacement therapy is harmful, but it is safer than smoking, so it can be used."

A patient who is an active alcoholic is admitted to the hospital for surgery. The nurse reviewing orders for this patient would be correct to question which postoperative medication for this patient? a. Acetaminophen b. Diazepam c. Morphine d. Thiamine

a. Acetaminophen

A pregnant patient in labor tells the nurse that she is afraid she may have harmed her fetus by consuming alcohol. What is an appropriate response by the nurse? a. Ask the patient how much alcohol she consumed, and at which stage of her pregnancy. b. Reassure the patient that the risk is likely to be minimal. c. Tell the patient that no amount of alcohol is considered safe during pregnancy. d. Tell the patient that the full range of outcomes may not be evident for years.

a. Ask the patient how much alcohol she consumed, and at which stage of her pregnancy.

A nurse is screening a patient being admitted to the hospital. The patient reports being fired for drinking at work. On further questioning, the patient reveals a history of daily alcohol consumption of more than a six-pack of beer each day, regular morning drinking, and several unsuccessful attempts to stop drinking. The nurse detects alcohol on the patient's breath. The nurse will inform the provider of these findings and request an order for which medication? a. Chlordiazepoxide [Librium] b. Clonidine c. Disulfiram [Antabuse] d. Naltrexone [ReVia]

a. Chlordiazepoxide [Librium]

A nurse is obtaining an admission history on a patient who reports daily drinking for several years. When the nurse questions the patient further, the patient reports drinking up to five or six drinks each day. The patient expresses worry about liver damage. What will the nurse do? a. Contact the patient's provider to request liver function studies. b. Explain that hepatitis, progressing to severe liver impairment, is likely. c. Inform the patient that the history indicates that cirrhosis is likely to occur. d. Tell the patient that stopping drinking will reverse any effects on the liver.

a. Contact the patient's provider to request liver function studies.

The parent of a child who is taking amphetamine (Adderall) to treat attention deficit/hyperactivity disorder (ADHD) asks the provider to recommend an over-the-counter medication to treat a cold. What will the nurse tell the parent? a. "Avoid any products containing pseudoephedrine or caffeine." b. "Never give over-the-counter medications with Adderall." c. "Sudafed is a safe and effective decongestant." d. "Use any over-the-counter medication from the local pharmacy."

a. "Avoid any products containing pseudoephedrine or caffeine."

The nurse is caring for a patient who has migraine headaches. The patient reports having these headaches more frequently. Which is an appropriate recommendation for this patient? a. "Avoid chocolate and caffeine." b. "Engage in strenuous exercise." c. "Have a glass of red wine with dinner." d. "Take ibuprofen prophylactically."

a. "Avoid chocolate and caffeine."

A young adult begins taking clonidine [Kapvay] to treat ADHD symptoms after suffering anorexia with methylphenidate [Ritalin]. What will the nurse include when teaching this patient about taking clonidine? a. "Avoid consuming alcohol while taking this medication." b. "Insomnia may still occur while taking this drug." c. "You will need to pick up a written prescription every 30 days." d. "You may crush the tablets and put them in food."

a. "Avoid consuming alcohol while taking this medication."

A nurse provides teaching for a patient with a newly diagnosed partial complex seizure disorder who is about to begin therapy with antiepileptic drugs (AEDs). Which statement by the patient indicates understanding of the teaching? a. "Even with an accurate diagnosis of my seizures, it may be difficult to find an effective drug." b. "I will soon know that the drugs are effective by being seizure free for several months." c. "Serious side effects may occur, and if they do, I should stop taking the medication." d. "When drug levels are maintained at therapeutic levels, I can expect to be seizure free."

a. "Even with an accurate diagnosis of my seizures, it may be difficult to find an effective drug." Even with an accurate diagnosis of seizures, many patients have to try more than one AED to find a drug that is effective and well tolerated. Unless patients are being treated for absence seizures, which occur frequently, monitoring of the clinical outcome is not sufficient for determining effectiveness, because patients with convulsive seizures often have long seizure-free periods. Serious side effects may occur, but withdrawing a drug precipitously can induce seizures. Not all patients have seizure control with therapeutic drug levels, because not all medications work for all patients.

A nurse is discussing partial versus generalized seizures with a group of nursing students. Which statement by a student indicates understanding of the teaching? a. "Febrile seizures are a type of generalized tonic-clonic seizure." b. "Generalized seizures are characterized by convulsive activity." c. "Partial seizures do not last as long as generalized seizures." d. "Patients having partial seizures do not lose consciousness."

a. "Febrile seizures are a type of generalized tonic-clonic seizure." Febrile seizures typically manifest as a tonic-clonic seizure of short duration and are a type of generalized seizure. Generalized seizures may be convulsive or nonconvulsive. Partial seizures may last longer than some types of generalized seizures. Patients with complex partial seizures and secondarily generalized seizures, which are types of partial seizures, may lose consciousness.

A nurse counsels a patient who is to begin taking phenytoin [Dilantin] for epilepsy. Which statement by the patient indicates understanding of the teaching? a. "I should brush and floss my teeth regularly." b. "Once therapeutic blood levels are reached, they are easy to maintain." c. "I can consume alcohol in moderation while taking this drug." d. "Rashes are a common side effect but are not serious."

a. "I should brush and floss my teeth regularly. Gingival hyperplasia occurs in about 20% of patients who take phenytoin. It can be minimized with good oral hygiene, so patients should be encouraged to brush and floss regularly. Because small fluctuations in phenytoin levels can affect response, maintaining therapeutic levels is not easy. Patients should be cautioned against consuming alcohol while taking phenytoin. Rashes can be serious and should be reported immediately.

A patient has begun taking phenobarbital after experiencing several seizures and is currently receiving 60 mg PO twice daily. After two weeks of therapy, the patient has a serum drug level of 30 mcg/mL and reports feeling drowsy much of the day. What will the nurse tell this patient? a. "I will contact your provider to discuss changing your dosing to once daily to minimize the drowsiness." b. "The drug level is low and you may need a higher dose, but taking it three times daily will reduce the drowsiness." c. "This side effect is expected and should decrease over time. You should avoid driving in the meantime." d. "Your lab work shows a higher than normal level of the drug and your provider will probably lower your dose."

a. "I will contact your provider to discuss changing your dosing to once daily to minimize the drowsiness." Phenobarbital has a long half-life and may be given once daily at bedtime to help manage its sedative effects. The serum drug level is within the normal range of 15 to 40 mcg/mL, so the dose does not need to be adjusted. Increasing the dose and the frequency is not necessary since the drug levels are therapeutic and scheduling the drug to three times daily will compound the sedative effects. The sedative side effects do not increase over time

A patient with chronic pain has a fentanyl patch applied to his right shoulder. The patient reports that his arm hurts and he requests a warm pack to apply to the area. Which statement by the nurse is correct? a. "Putting a warm pack on the area where the fentanyl patch is located could accelerate fentanyl release." b. "Your arm probably hurts because of the fentanyl patch." c. "I can remove your patch and reapply it after you are done with the warm pack." d. "The fentanyl patch is heat resistant, so I will get you a warm pack."

a. "Putting a warm pack on the area where the fentanyl patch is located could accelerate fentanyl release."

A patient who is an opioid addict has undergone detoxification with buprenorphine [Subutex] and has been given a prescription for buprenorphine with naloxone [Suboxone]. The patient asks the nurse why the drug was changed. Which response by the nurse is correct? a. "Suboxone has a lower risk of abuse." b. "Suboxone has a longer half-life." c. "Subutex causes more respiratory depression." d. "Subutex has more buprenorphine."

a. "Suboxone has a lower risk of abuse."

The parent of an adolescent who has taken methylphenidate 20 mg/day for 6 months for attention deficit/hyperactivity disorder (ADHD) brings the child to clinic for evaluation of a recent onset of nausea, vomiting, and headaches. The parent expresses concern that the child seems less focused and more hyperactive than before. What will the nurse do next? a. Ask the child whether the drug is being taken as prescribed. b. Contact the provider to discuss increasing the dose to 30 mg/day. c. Recommend taking the drug with meals to reduce gastrointestinal side effects. d. Report signs of drug toxicity to the patient's provider.

a. Ask the child whether the drug is being taken as prescribed.

A patient who is taking oral contraceptives begins taking valproic acid [Depakote] for seizures. After a week of therapy with valproic acid, the patient tells the nurse that she is experiencing nausea. What will the nurse do? a. Ask the patient if she is taking the valproic acid with food, because taking the drug on an empty stomach can cause gastrointestinal side effects. b. Contact the provider to request an order for a blood ammonia level, because hyperammonemia can occur with valproic acid therapy. c. Suggest that the patient perform a home pregnancy test, because valproic acid can reduce the efficacy of oral contraceptives. d. Suspect that hepatotoxicity has occurred, because this is a common adverse effect of valproic acid.

a. Ask the patient if she is taking the valproic acid with food, because taking the drug on an empty stomach can cause gastrointestinal side effects. Gastrointestinal effects, including nausea, vomiting, and indigestion, are common with valproic acid and can be minimized by taking the drug with food or using an enteric-coated product. Hyperammonemia can occur when valproic acid is combined with topiramate. Signs of pregnancy usually do not occur within a week, so this is less likely. Hepatotoxicity is a rare adverse effect.

A child has been taking SD methylphenidate [Ritalin], 10 mg at 0800 and 1200 and 5 mg at 1600, for 2 months. The parents tell the nurse that the child sometimes misses the noon dose while at school. The child's appetite is normal. The teacher has reported a slight improvement in hyperactivity and impulsivity. What will the nurse do? a. Ask the prescriber whether this child could be given methylphenidate [Concerta]. b. Contact the prescriber to suggest using a nonstimulant medication. c. Reinforce the need to take all doses as prescribed. d. Suggest drug holidays for the child on weekends.

a. Ask the prescriber whether this child could be given methylphenidate [Concerta].

A patient will receive buprenorphine [Butrans] as a transdermal patch for pain. What is important to teach this patient about the use of this drug? a. Avoid prolonged exposure to the sun. b. Cleanse the site with soap or alcohol. c. Remove the patch daily at bedtime. d. Remove hair by shaving before applying the patch.

a. Avoid prolonged exposure to the sun. Patients using the buprenorphine transdermal patch should be cautioned against heat, heating pads, hot baths, saunas, and prolonged sun exposure. The skin should be cleaned with water only. The patch should stay on for 7 days before a new patch is applied. Patients should remove hair by clipping, not shaving.

A patient reports having recurring headaches described as 1 to 2 headaches per day for several weeks. The nurse understands that these headaches are most likely descriptive of which type of headache? a. Cluster headache b. Migraine headache c. Simple headache d. Tension headache

a. Cluster headache

A college student is brought to the emergency department by a group of friends who report that they had been dancing at a nightclub when their friend collapsed. The patient has a temperature of 105°F and shows jaw clenching and confusion. The nurse will expect to administer which medication? a. Dantrolene [Dantrium] b. Haloperidol [Haldol] c. Methadone d. Naloxone [Narcan]

a. Dantrolene [Dantrium]

A patient who has developed opioid tolerance will experience which effect? a. Decreased analgesic effect b. Decreased constipation c. Increased euphoria d. Increased respiratory depression

a. Decreased analgesic effect Patients who develop tolerance to opioids will develop tolerance to its analgesic, euphoric, and sedative effects and will also develop tolerance to respiratory depression. Very little tolerance develops to constipation.

A patient asks the nurse what can be given to alleviate severe, chronic pain of several months' duration. The patient has been taking oxycodone [OxyContin] and states that it is no longer effective. The nurse will suggest discussing which medication with the provider? a. Fentanyl [Duragesic] transdermal patch b. Hydrocodone [Vicodin] PO c. Meperidine [Demerol] PO d. Pentazocine [Talwin] PO

a. Fentanyl [Duragesic] transdermal patch Transdermal fentanyl is indicated only for persistent, severe pain in patients already opioid tolerant. Hydrocodone, a combination product, has actions similar to codeine and is not used for severe, chronic pain. Meperidine is not recommended for continued use because of the risk of harm caused by the accumulation of a toxic metabolite. Pentazocine is an agonist-antagonist opioid and is less effective for pain; moreover, when given to a patient who is already opioid tolerant, it can precipitate an acute withdrawal syndrome

A nurse is preparing a pediatric patient for surgery and is teaching the patient and the child's parents about the use of the patient-controlled analgesia pump. The parents voice concern about their child receiving an overdose of morphine. What will the nurse do? a. Instruct the parents not to activate the device when their child is sleeping. b. Reassure the parents that drug overdose is not possible with PCA. c. Suggest that the child use the PCA sparingly. d. Tell the patient that the pump can be programmed for PRN dosing only.

a. Instruct the parents not to activate the device when their child is sleeping. The nurse should instruct parents not to activate the PCA when their child is sleeping because that can lead to drug overdose. Postoperative pain should be treated appropriately with medications that are effective. Nonopioid medications are not sufficient to treat postoperative pain. Patients should be encouraged to use the PCA as needed so that pain can be controlled in a timely fashion. PRN dosing is not as effective as dosing that is continuous, so a basal dose should be given as well as a PRN dose.

In discussing the rationale for using methadone to ease opioid withdrawal, the nurse would explain that it has which pharmacologic properties or characteristics? a. Methadone can prevent abstinence syndrome. b. Methadone has a shorter duration of action than other opioids. c. Methadone is a nonopioid agent. d. Methadone lacks cross-tolerance with other opioids.

a. Methadone can prevent abstinence syndrome.

Which agent is most likely to cause serious respiratory depression as a potential adverse reaction? a. Morphine [Duramorph] b. Pentazocine [Talwin] c. Hydrocodone [Lortab] d. Nalmefene [Revex]

a. Morphine [Duramorph] Morphine is a strong opioid agonist and as such has the highest likelihood of causing respiratory depression. Pentazocine, a partial agonist, and hydrocodone, a moderate to strong agonist, may cause respiratory depression, but they do not do so as often or as seriously as morphine. Nalmefene, an opioid antagonist, would be used to reverse respiratory depression with opioids.

A postoperative patient has an epidural infusion of morphine sulfate [Astramorph]. The patient's respiratory rate declines to 8 breaths per minute. Which medication would the nurse anticipate administering? a. Naloxone [Narcan] b. Acetylcysteine [Mucomyst] c. Methylprednisolone [Solu-Medrol] d. Protamine sulfate

a. Naloxone [Narcan] Naloxone is a narcotic antagonist that can reverse the effects, both adverse and therapeutic, of opioid narcotic analgesics.

The nurse watches for which electrocardiogram changes in the patient receiving methadone? a. Prolonged QT interval b. Prolonged PR interval c. Increased heart rate d. Elevated T waves

a. Prolonged QT interval Methadone prolongs the QT interval and may pose a risk of potentially fatal dysrhythmias. Torsades de pointes has developed in patients taking 65 to 400 mg/day.

A patient has been taking methadone [Dolophine] for 5 months to overcome an opioid addiction. The nurse should monitor the patient for which of the following electrocardiographic changes? a. Prolonged QT interval b. Prolonged P-R interval c. AV block d. An elevated QRS complex

a. Prolonged QT interval Methadone prolongs the QT interval. It does not prolong the P-R interval, cause AV block, or produce an elevated QRS complex.

The nurse recognizes that the primary indication for the administration of morphine is to do what? a. Relieve pain b. Reduce anxiety c. Treat left ventricular failure d. Decrease level of consciousness

a. Relieve pain The principal indication for morphine is relief of moderate to severe pain. The drug can relieve postoperative pain, the chronic pain of cancer, and pain associated with labor and delivery. In addition, morphine can be used to relieve pain of myocardial infarction and dyspnea associated with left ventricular failure and pulmonary edema. Morphine may also be administered before surgery for sedation and reduction of anxiety.

After surgery, a patient has morphine prescribed for postoperative pain. It is most important for the nurse to make which assessment? a. Respiratory rate b. Heart rate c. Pain level d. Constipation

a. Respiratory rate

After surgery, a patient has morphine prescribed for postoperative pain. It is most important for the nurse to make which assessment? a. Respiratory rate b. Heart rate c. Pain level d. Constipation

a. Respiratory rate

A patient shows loss of consciousness, jaw clenching, contraction and relaxation of muscle groups, and periods of cyanosis. The nurse correctly identifies this as which type of seizure? a. Tonic-clonic b. Petit mal c. Myoclonic d. Atonic

a. Tonic-clonic Tonic-clonic seizures (or grand mal seizures) are considered generalized seizures and are manifested by a loss of consciousness, jaw clenching, muscle relaxation alternating with muscle contractions, and periods of cyanosis. Absence seizures (or petit mal seizures) are characterized by loss of consciousness for a brief period and usually involve eye blinking and staring into space. Myoclonic seizures consist of sudden contractions that may be limited to one limb or may involve the entire body. Atonic seizures are characterized by sudden loss of muscle tone.

A college-age student is brought to the emergency department by friends after consuming NoDoz tablets along with several cups of coffee and a few energy drinks. The patient is complaining of nausea and diarrhea and appears restless. The nurse understands that a. arrhythmias and convulsions may occur. b. caffeine dependence does not occur. c. effects of the substances will wear off shortly. d. severe adverse effects do not occur.

a. arrhythmias and convulsions may occur.

A patient has been using an amphetamine drug as an anorexiant for several weeks and asks the nurse about long-term adverse effects of this type of medication. The nurse will explain to the patient that these drugs a. can cause cardiac dysrhythmias. b. contribute to the development of narcolepsy. c. do not have severe effects when used properly. d. will cause orthostatic hypotension.

a. can cause cardiac dysrhythmias.

A patient has been admitted to the ER because of an overdose of an oral benzodiazepine. He is very drowsy but still responsive. The nurse will prepare for which immediate intervention? a) hemodialysis to remove the medication b) administration of flumazenil c) administration of naloxone d) intubation and mechanical ventilation

b) administration of flumazenil

The nurse is preparing to administer a medication for sleep. Which intervention applies to the administration of a non-benzodiazepine, such as zaleplon (Sonata)? a) these drugs need to be taken about 1 hour before bedtime b) because of their rapid onset, these drugs need to taken just before bedtime c) the patient needs to be cautioned about the high incidence of morning drowsiness that may occur after taking these drugs c) these drugs are less likely to interact w/alcohol

b) because of their rapid onset, these drugs need to taken just before bedtime

Which side effects of opioid analgesics can have therapeutic benefits? (Select all that apply.) a. Biliary colic b. Cough suppression c. Suppression of bowel motility d. Urinary retention e. Vasodilation

b, c, e b. Cough suppression c. Suppression of bowel motility e. Vasodilation Individual effects of morphine may be beneficial, detrimental, or both. Cough suppression is usually beneficial; suppression of bowel motility and vasodilation can be either beneficial or detrimental. Biliary colic and urinary retention are always detrimental side effects.

During a health history, the nurse asks a male patient about alcohol use. The patient tells the nurse that he and his wife are trying to conceive a pregnancy and he is using alcohol to lower his inhibitions. What will the nurse counsel this patient? a. "Alcohol causes increased masculinization." b. "Alcohol may cause testicular atrophy and sterility." c. "Alcohol will improve your chances of conceiving." d. "Alcohol will also help you to ejaculate."

b. "Alcohol may cause testicular atrophy and sterility."

A patient who is an alcoholic is prescribed disulfiram [Antabuse] to help prevent relapse and the nurse provides teaching about the use of this drug. Which statement by the patient indicates understanding of the teaching? a. "I may use alcohol in cooking since heating foods destroys the alcohol." b. "I should avoid shaving lotion and mouthwashes that contain even small amounts of alcohol." c. "If I decide to drink, I should stop taking the disulfiram at least one week prior to consuming alcohol." d. "If I use alcohol, the effects will be uncomfortable but are not dangerous."

b. "I should avoid shaving lotion and mouthwashes that contain even small amounts of alcohol."

A patient asks about nicotine patches for smoking cessation and wants to know the difference between the 24-hour patch and the 16-hour patch. Which response by the nurse is correct? a. "The 16-hour patch is for patients who have trouble sleeping." b. "The 16-hour patch simulates usual nicotine ingestion patterns." c. "The 24-hour patch is for persons weighing more than 100 pounds." d. "The 24-hour patch is recommended for heavier smokers."

b. "The 16-hour patch simulates usual nicotine ingestion patterns."

An alcoholic patient's spouse asks a nurse about recovery from chronic alcoholism. The patient is confused and has abnormal eye movements and nystagmus. Which statement by the nurse is correct? a. "The symptoms your spouse shows are partly reversible in most people." b. "These symptoms can be reversed with vitamin therapy and good nutrition." c. "Your spouse has symptoms of an irreversible encephalopathy." d. "Your spouse will probably recover completely after detoxification."

b. "These symptoms can be reversed with vitamin therapy and good nutrition."

A patient with a desire to stop smoking asks a nurse about nicotine chewing gum [Nicorette]. The patient currently smokes 30 cigarettes per day. Which statement by the nurse is correct? a. "Stop using the gum 6 months after you stop using cigarettes." b. "Use the 4-mg strength gum and chew one piece every 2 to 3 hours." c. "Use the gum whenever you feel a craving for a cigarette." d. "You should start with 30 pieces of the 2-mg strength gum per day."

b. "Use the 4-mg strength gum and chew one piece every 2 to 3 hours."

The spouse of a patient who smokes wonders why anyone would want to engage in "such a disgusting habit." What will the nurse tell the spouse? a. Nicotine causes relaxation and helps with sleep. b. Nicotine increases alertness and promotes dopamine release. c. Nicotine lowers blood pressure. d. Nicotine settles the stomach and reduces nausea and vomiting.

b. Nicotine increases alertness and promotes dopamine release.

A patient reports a desire to stop smoking and asks what is available without a prescription to help with smoking cessation. The nurse tells the patient that which method is best? a. Abrupt discontinuation to shorten withdrawal effects b. Nicotine replacement and 1-800-QUITNOW c. Nicotine replacement products tapered over a year d. Support groups without the use of medications

b. Nicotine replacement and 1-800-QUITNOW

A school nurse is teaching a high school health class about the effects of marijuana use. Which statement by a student indicates a need for further teaching? a. "Chronic use of marijuana can result in irreversible brain changes." b. "Higher doses of marijuana are likely to produce increased euphoria." c. "Marijuana is unique in that it produces euphoria, sedation, and hallucinations." d. "Marijuana has more prolonged effects when it is ingested than when it is smoked."

b. "Higher doses of marijuana are likely to produce increased euphoria."

The nurse is teaching a child and a parent about taking methylphenidate (Ritalin) to treat attention deficit/hyperactivity disorder (ADHD). Which statement by the parent indicates understanding of the teaching? a. "I should give this drug to my child at bedtime." b. "My child should avoid products containing caffeine." c. "The drug should be stopped immediately if my child develops aggression." d. "We should monitor my child's weight since weight gain is common."

b. "My child should avoid products containing caffeine."

A nurse is caring for a patient who is addicted to barbiturates and who will begin receiving phenobarbital. The nurse discusses the care of this patient with a nursing student. Which statement by the student indicates understanding of the teaching? a. "Phenobarbital acts as an antagonist to barbiturates and prevents toxicity." b. "Phenobarbital has a long half-life and can be tapered gradually to minimize abstinence symptoms." c. "Phenobarbital can be administered on an as-needed basis to treat withdrawal symptoms." d. "Phenobarbital prevents respiratory depression associated with barbiturate withdrawal."

b. "Phenobarbital has a long half-life and can be tapered gradually to minimize abstinence symptoms."

A nurse is teaching the parents of a child who has attention-deficit/hyperactivity disorder about methylphenidate [Concerta]. Which statement by the child's parents indicates understanding of the teaching? a. "The effects of this drug will wear off in 4 to 6 hours." b. "The tablet needs to be swallowed whole, not crushed or chewed." c. "This medication has fewer side effects than amphetamines." d. "We should call the provider if we see parts of the medicine in our child's stools."

b. "The tablet needs to be swallowed whole, not crushed or chewed."

A college student tells the nurse that several friends have been using synthetic marijuana to get high. What will the nurse tell this patient about this type of substance? a. "These substances are fairly safe because they are derived from herbs." b. "They can cause hypertension, nausea, vomiting, and hallucinations." c. "These substances do not have mind-altering affects." d. "These substances produce a high and they are not illegal."

b. "They can cause hypertension, nausea, vomiting, and hallucinations."

The healthcare provider orders oxycodone [Roxicodone] oral suspension 10 mg PO every 4 hours PRN. The medication available is oxycodone [Roxicodone] 20 mg/mL. How many mL will the nurse administer? a. 0.25 mL b. 0.5 mL c. 1 mL d. 2 mL

b. 0.5 mL

A patient admitted to the hospital with a diagnosis of pneumonia asks the nurse, "Why am I receiving codeine? I don't have any pain." The nurse's response is based on the knowledge that codeine also has which effect? a. Immunostimulant b. Antitussive c. Expectorant d. Decongestant

b. Antitussive Codeine provides analgesic and antitussive therapeutic effects.

The nurse would question the administration of morphine in the patient with a history of which condition? a. Diabetes b. Asthma c. Anorexia d. Cancer

b. Asthma Morphine should be used with caution in patients with asthma, emphysema, kyphoscoliosis, chronic cor pulmonale, and extreme obesity. Caution is also needed in patients taking other drugs that can depress respiration (e.g., barbiturates, benzodiazepines, general anesthetics). Morphine is indicated in the treatment of severe pain associated with cancer.

The nurse is working in a neonatal intensive care unit and is caring for an infant who is experiencing multiple periods of apnea and bradycardia. Which drug will the nurse expect to administer? a. Albuterol (Proventil) b. Caffeine (Cafcit) c. Doxapram (Dopram) d. Methylphenidate (Ritalin)

b. Caffeine (Cafcit)

A nurse is assessing a patient who becomes motionless and seems to stare at the wall and then experiences about 60 seconds of lip smacking and hand wringing. What should the nurse do? a. Ask the patient about a history of absence seizures. b. Contact the provider to report symptoms of a complex partial seizure. c. Notify the provider that the patient has had a grand mal seizure. d. Request an order for intravenous diazepam [Valium] to treat status epilepticus.

b. Contact the provider to report symptoms of a complex partial seizure. This patient showed signs of a complex partial seizure, characterized by impaired consciousness beginning with a period of motionlessness with a fixed gaze, followed by a period of automatism. The entire episode generally lasts 45 to 90 seconds. Absence seizures are characterized by loss of consciousness for a brief period (about 10 to 30 seconds) and may involve mild, symmetric motor activity or no motor signs. A grand mal seizure is characterized by jaw clenching and rigidity followed by alternating muscle relaxation and contraction and then periods of cyanosis, all with a loss of consciousness. Status epilepticus is a seizure that persists for 30 minutes or longer.

A nurse is caring for a patient who has been taking an antiepileptic drug for several weeks. The nurse asks the patient if the therapy is effective. The patient reports little change in seizure frequency. What will the nurse do? a. Ask the patient to complete a seizure frequency chart for the past few weeks. b. Contact the provider to request an order for serum drug levels. c. Reinforce the need to take the medications as prescribed. d. Request an order to increase the dose of the antiepileptic drug.

b. Contact the provider to request an order for serum drug levels. If medication therapy is not effective, it is important to measure serum drug levels of the medication to determine whether therapeutic levels have been reached and to help monitor patient compliance. Patients should be asked at the beginning of therapy to keep a seizure frequency chart to help deepen their involvement in therapy; asking for historical information is not helpful. Until it is determined that the patient is not complying, the nurse should not reinforce the need to take the medication. Until the drug level is known, increasing the dose is not indicated.

A patient who is agitated and profoundly anxious is brought to the emergency department. The patient acts paranoid and keeps describing things in the room that do not exist. A cardiac monitor shows an irregular ventricular tachycardia. Which medication will the nurse expect to administer? a. Anticocaine vaccine b. Diazepam [Valium] c. Disulfiram [Antabuse] d. Vigabatrin [Sabril]

b. Diazepam [Valium]

A patient who is morbidly obese is admitted for treatment. The prescriber orders lisdexamfetamine [Vyvanse]. The nurse will be concerned if this patient shows signs of: a. Anorexia. b. Dyspnea. c. Insomnia. d. Loquaciousness.

b. Dyspnea.

Clonidine is prescribed for the treatment of a patient's severe pain. The nurse is aware that for treatment of pain, this drug should be administered by which route? a. Oral b. Epidural c. Intravenous d. Transdermal

b. Epidural Clonidine has two approved applications: treatment of hypertension and relief of severe pain. As a means of relieving severe pain, clonidine is administered by way of continuous epidural infusion.

A patient arrives in the emergency department acutely intoxicated and difficult to arouse. The patient's friends tell the nurse that the patient took a handful of diazepam [Valium] pills while at a party several hours ago. The nurse will expect to administer which drug? a. Buprenorphine [Subutex] b. Flumazenil [Romazicon] c. Nalmefene [Revex] d. Naloxone [Narcan]

b. Flumazenil [Romazicon]

The nurse is checking an 8-year-old child who has attention deficit/hyperactivity disorder (ADHD) into a clinic for an annual well-child visit. The child takes methylphenidate HCl (Ritalin). Which assessments are especially important for this child? a. Heart rate, respiratory rate, and oxygen saturation b. Height, weight, and blood pressure c. Measures of fine- and gross-motor development d. Nausea, vomiting, and gastrointestinal upset

b. Height, weight, and blood pressure

The nurse is planning care for a patient receiving morphine sulfate [Duramorph] by means of a patient-controlled analgesia (PCA) pump. Which intervention may be required because of a potential adverse effect of this drug? a. Administering a cough suppressant b. Inserting a Foley catheter c. Administering an antidiarrheal d. Monitoring liver function tests

b. Inserting a Foley catheter Morphine can cause urinary hesitancy and urinary retention. If bladder distention or inability to void is noted, the prescriber should be notified.

A patient who has been taking phenobarbital for epilepsy begins taking valproic acid [Depakote] as adjunct therapy. The nurse notes that the patient is very drowsy. What will the nurse do? a. Explain to the patient that tolerance to sedation eventually will develop. b. Notify the prescriber, and request an order to reduce the dose of phenobarbital. c. Notify the prescriber of the need to increase the dose of valproic acid. d. Request an order for liver function tests to monitor for hepatotoxicity.

b. Notify the prescriber, and request an order to reduce the dose of phenobarbital. Valproic acid competes with phenobarbital for drug-metabolizing enzymes and can increase plasma levels of phenobarbital by approximately 40%. When this combination is used, the dose of phenobarbital should be reduced. Increasing the dose of valproic acid would compound the problem. Patients taking phenobarbital alone experience sedation, which diminishes as tolerance develops. Liver toxicity is a rare adverse effect of valproic acid and is marked by symptoms of nausea, vomiting, and malaise, not drowsiness.

A patient reports having taken morphine for the past 6 months. Which medication, if ordered by the physician, should the nurse question? a. Promethazine [Phenergan] b. Pentazocine [Talwin] c. Methylnaltrexone [Relistor] d. Dextromethorphan [Delsym]

b. Pentazocine [Talwin]

A patient reports having taken morphine for the past 6 months. Which medication, if ordered by the physician, should the nurse question? a. Promethazine [Phenergan] b. Pentazocine [Talwin] c. Methylnaltrexone [Relistor] d. Dextromethorphan [Delsym]

b. Pentazocine [Talwin] Pentazocine is an agonist-antagonist opioid. If pentazocine is given to a patient who is physically dependent on a pure opioid agonist such as morphine, withdrawal or abstinence syndrome will occur.

In preparing an inservice about opioids, the nurse includes which problem as the most serious adverse effect? a. Suppressed cardiac automaticity b. Respiratory depression c. Profound sedation d. Hyperthermia

b. Respiratory depression Respiratory depression is the most serious adverse effect of the opioids. At equianalgesic doses, all of the pure opioid agonists depress respiration to the same extent. Death following overdose is almost always a result of respiratory arrest.

A patient is to begin taking phenytoin [Dilantin] for seizures. The patient tells the nurse that she is taking oral contraceptives. What will the nurse tell the patient? a. She may need to increase her dose of phenytoin while taking oral contraceptives. b. She should consider a different form of birth control while taking phenytoin. c. She should remain on oral contraceptives, because phenytoin causes birth defects. d. She should stop taking oral contraceptives, because they reduce the effectiveness of phenytoin.

b. She should consider a different form of birth control while taking phenytoin. Because phenytoin can reduce the effects of oral contraceptive pills (OCPs) and because avoiding pregnancy is desirable when taking phenytoin, patients should be advised to increase the dose of oral contraceptives or use an alternative method of birth control. Increasing the patient's dose of phenytoin is not necessary; OCPs do not affect phenytoin levels. Phenytoin is linked to birth defects; OCPs have decreased effectiveness in patients treated with phenytoin, and the patient should be advised to increase the OCP dose or to use an alternative form of birth control. OCPs do not alter the effects of phenytoin.

The nurse is working on a postoperative unit in which pain management is part of routine care. Which statement is the most helpful in guiding clinical practice in this setting? a. At least 30% of the U.S. population is prone to drug addiction and abuse. b. The development of opioid dependence is rare when opioids are used for acute pain. c. Morphine is a common drug of abuse in the general population. d. The use of PRN (as needed) dosing provides the most consistent pain relief without risk of addiction.

b. The development of opioid dependence is rare when opioids are used for acute pain. The development of dependence on or addiction to opioids as a result of clinical exposure is extremely rare. In fact, some estimate that only 25% of patients receive doses of opioids that are sufficient to relieve suffering. Only about 8% of the population is estimated to be prone to drug abuse. Morphine is a drug of abuse, but this fact is not helpful in guiding clinical practice. A patient-controlled analgesia (PCA) pump provides the most consistent pain relief, better than PRN and fixed-dosing schedules.

A college student admits frequent use of LSD to a nurse and reports plans to stop using it. What will the nurse tell this student? a. Flashback episodes and episodic visual disturbances are common. b. Tolerance to the effects of LSD will fade quickly once use of the drug has stopped. c. Withdrawal symptoms can be mitigated with haloperidol [Haldol]. d. Withdrawal from LSD is associated with a severe abstinence syndrome.

b. Tolerance to the effects of LSD will fade quickly once use of the drug has stopped.

A patient with moderate to severe chronic pain has been taking oxycodone [OxyContin] 60 mg every 6 hours PRN for several months and tells the nurse that the medication is not as effective as before. The patient asks if something stronger can be taken. The nurse will contact the provider to discuss: a. administering a combination opioid analgesic/acetaminophen preparation. b. changing the medication to a continued-release preparation. c. confronting the patient about drug-seeking behaviors. d. withdrawing the medication, because physical dependence has occurred.

b. changing the medication to a continued-release preparation. Oxycodone is useful for moderate to severe pain, and a continued-release preparation may give more continuous relief. Dosing is every 12 hours, not PRN. A combination product is not recommended with increasing pain, because the nonopioid portion of the medication cannot be increased indefinitely. This patient does not demonstrate drug-seeking behaviors. Physical dependence is not an indication for withdrawing an opioid, as long as it is still needed; it indicates a need for withdrawing the drug slowly when the drug is discontinued.

An older adult has been given a benzodiazepine for sleep induction, but the night nurse noted that the patient was awake most of the night, watching TV and reading in bed. The nurse documents that the patient has had which type of reaction to the medication? a) allergic b) teratogenic c) paradoxical d) idiopathic

c) paradoxical

Which considerations are important for the nurse to remember when administering a benzodiazepine as a sedative-hypnotic drug? (select all that apply) a) these drugs are intended for long-term management of insomnia b) the drugs can be administered safely with other CNS depressants for insomnia c) the dose needs to be given about 1 hour before the patient's bedtime d) the drug is used as a first choice for treatment of sleeplessness e) the patient needs to be evaluated for the drowsiness that may occur the morning after a benzodiazepine is taken

c) the dose needs to be given about 1 hour before the patient's bedtime e) the patient needs to be evaluated for the drowsiness that may occur the morning after a benzodiazepine is taken

A nurse is providing education to a group of college students about the long-term effects of alcohol. Which statement by a student indicates understanding of the teaching? a. "Chronic alcohol use contributes to the development of osteoporosis." b. "Chronic use of alcohol can actually decrease the risk of cardiomyopathy." c. "Even small amounts of alcohol are related to the development of certain cancers." d. "Pancreatitis is not a common problem among chronic users of alcohol."

c. "Even small amounts of alcohol are related to the development of certain cancers."

A patient asks a nurse about the effects of chronic alcohol use on the heart. The nurse's best response would be which statement? a. "Chronic alcohol use affects the liver more adversely than it does the heart." b. "Drinking more than two drinks a day protects the heart from atherosclerosis." c. "Long-term alcohol use can damage the heart and cause heart failure." d. "Over time, alcohol use can lower your blood pressure."

c. "Long-term alcohol use can damage the heart and cause heart failure."

A male patient tells a nurse that he drinks a six-pack of beer a day. When the nurse begins to question him further about his alcohol consumption, he says, "You sound like my wife. She's always nagging me to quit. It's only beer!" Which response by the nurse is most appropriate? a. "Because the alcohol in beer is diluted in a larger volume, it is absorbed more slowly." b. "Have you considered switching to wine? It has chemicals that protect your heart." c. "The amount you drink is equivalent to six shots of whiskey each day." d. "You could try to cut the amount in half to a level that is better for your health."

c. "The amount you drink is equivalent to six shots of whiskey each day."

A prescriber has ordered nicotine nasal spray for a patient to assist with smoking cessation. Which statement will the nurse include when teaching the patient about the medication? a. "This will produce a steady level of nicotine to reduce your cravings." b. "You should gradually reduce the dose after 3 months of use." c. "You should use 1 spray in each nostril per dose up to 5 times per hour." d. "You will not develop dependence on the nicotine in the nasal spray."

c. "You should use 1 spray in each nostril per dose up to 5 times per hour."

A college student who is unresponsive is brought to the emergency department by friends, who say that their friend drank more than half of a large bottle of whiskey 3 hours ago. Assessment reveals a blood alcohol level of 0.32%. The vital signs are BP, 88/32 mm Hg; R, 6/min; T, 96.8°F; and P, 76/min and weak and thready. The nurse should prepare the patient for which intervention? a. IV fluids and stimulants b. Charcoal administration c. Gastric lavage and dialysis d. Naloxone [Narcan] administration

c. Gastric lavage and dialysis

A 4-year-old child is brought to the emergency department with symptoms of nausea and vomiting and a weak, thready pulse of 120 beats per minute after ingesting several cigarettes at home. The nurse caring for this child will expect to provide which treatment? a. Gastric lavage b. Hemodialysis c. Respiratory support d. Vasoconstrictors

c. Respiratory support

A nurse is discussing alcohol abuse with a group of nursing students. One student asks whether alcohol consumption has any beneficial effects. The nurse replies that, in moderate amounts, alcohol: a. helps people to sleep well. b. improves sexual responsiveness. c. may protect against dementia. d. prevents hypothermia.

c. may protect against dementia.

The parent of an obese 10-year-old child asks the nurse about medications to aid in weight loss. Which response by the nurse is correct? a. "Anorexiants are often used to 'jump start' a weight loss regimen in children." b. "Children are able to use over-the-counter anorexiants on a long-term basis." c. "Children under 12 years of age should not use weight loss drugs." d. "Side effects of anorexiants occur less often in children."

c. "Children under 12 years of age should not use weight loss drugs."

A nurse is teaching a drug prevention class to a group of parents of adolescents. Which statement by a parent indicates understanding of the teaching? a. "Compared with alcohol, marijuana b. has little or no long-term adverse effects." c. "Ecstasy causes reversible damage to serotonergic neurons." "LSD does not cause an abstinence syndrome when it is withdrawn." d. "Most individuals who abuse opioids began using them therapeutically."

c. "Ecstasy causes reversible damage to serotonergic neuron

A nurse is providing teaching to a patient newly diagnosed with partial seizures who will begin taking oxcarbazepine [Trileptal]. The patient also takes furosemide [Lasix] and digoxin [Lanoxin]. Which statement by the patient indicates understanding of the teaching? a. "I may need to increase my dose of Trileptal while taking these medications." b. "I may develop a rash and itching, but these are not considered serious." c. "I should report any nausea, drowsiness, and headache to my provider." d. "I should use salt substitutes instead of real salt while taking these drugs."

c. "I should report any nausea, drowsiness, and headache to my provider." Oxcarbazepine can cause clinically significant hyponatremia in 2.5% of patients. If oxcarbazepine is combined with other drugs that reduce sodium, the patient should be monitored. Signs of hyponatremia include nausea, drowsiness, confusion, and headache, and patients should be taught to report these symptoms. Increasing the dose of oxcarbazepine is not indicated. Rashes can indicate a serious drug reaction, and providers should be notified so that the oxcarbazepine can be withdrawn. Salt substitutes would compound the problem of hyponatremia

A nurse is discussing the differences between OxyContin OC and OxyContin OP with a group of nursing students. Which statement by a student indicates understanding of the teaching? a. "OxyContin OC cannot be drawn into a syringe for injection." b. "OxyContin OP has greater solubility in water and alcohol." c. "OxyContin OP is not easily crushed into a powder." d. "Patients using OxyContin OP are less likely to overdose."

c. "OxyContin OP is not easily crushed into a powder."

A child is diagnosed with attention-deficit/hyperactivity disorder (ADHD). The prescriber orders a central nervous system stimulant. Which statement by the child's parent indicates a need for further teaching? a."I should report insomnia and poor appetite to his provider." b."I will make sure he takes his medication after breakfast every day." c."This drug will make him less impulsive while he's at school." d."This medication will help my child focus so he can learn new behaviors."

c. "This drug will make him less impulsive while he's at school."

The nurse is teaching a patient with cancer about a new prescription for a fentanyl [Sublimaze] patch, 25 mcg/hr, for chronic back pain. Which statement is the most appropriate to include in the teaching plan? a. "You will need to change this patch every day, regardless of your pain level." b. "This type of pain medication is not as likely to cause breathing problems." c. "With the first patch, it will take about 24 hours before you feel the full effects." d. "Use your heating pad for the back pain. It will also improve the patch's effectiveness."

c. "With the first patch, it will take about 24 hours before you feel the full effects." Full analgesic effects can take up to 24 hours to develop with fentanyl patches. Most patches are changed every 72 hours.

A patient with a seizure disorder is admitted to the hospital and has a partial convulsive episode shortly after arriving on the unit. The patient has been taking phenytoin [Dilantin] 100 mg three times daily and oxcarbazepine [Trileptal] 300 mg twice daily for several years. The patient's phenytoin level is 8.6 mcg/mL, and the oxcarbazepine level is 22 mcg/mL. The nurse contacts the provider to report these levels and the seizure. What will the nurse expect the provider to order? a. A decreased dose of oxcarbazepine b. Extended-release phenytoin c. An increased dose of phenytoin d. Once-daily dosing of oxcarbazepine

c. An increased dose of phenytoin This patient's phenytoin level is low; the therapeutic range is 10 to 20 mcg/mL. An increase in the phenytoin dose is necessary. The oxcarbazepine level is within the normal range of 3 to 40 mcg/mL, so changing the dose is not necessary. Extended-release phenytoin is absorbed more slowly and would not increase this patient's serum phenytoin level. The dosing of oxcarbazepine does not need to be changed.

The nurse correlates orthostatic hypotension resulting from opioid administration with which cause? a. Decreased heart rate b. Impaired contractility c. Dilation of peripheral veins d. Elevated circulating catecholamines

c. Dilation of peripheral veins Morphine-like drugs lower blood pressure by blunting the baroreceptor reflex and by dilating peripheral arterioles and veins. Peripheral vasodilation results primarily from morphine-induced release of histamine. Hypotension is mild in the recumbent patient but can be substantial when the patient stands up.

A patient is brought to the emergency department with a drug overdose causing respiratory depression. Which drug will the nurse expect to administer? a. Albuterol (Proventil) b. Caffeine (Cafcit) c. Doxapram (Dopram) d. Methylphenidate (Ritalin)

c. Doxapram (Dopram)

The nurse is caring for a 7-year-old child who has difficulty concentrating and completing tasks and who cannot seem to sit still. Which diagnostic test may be ordered to assist with a diagnosis of attention deficit/hyperactivity disorder (ADHD) in this child? a. Computerized tomography (CT) of the head b. Electrocardiogram (ECG) c. Electroencephalogram (EEG) d. Magnetic resonance imaging (MRI) of the brain

c. Electroencephalogram (EEG)

A child will begin taking methylphenidate [Ritalin] for attention-deficit/hyperactivity disorder. Important baseline information about this patient will include: a. Results of an electrocardiogram (ECG). b. Family history of psychosis. c. Height and weight. d. Renal function.

c. Height and weight.

A nurse administers naloxone [Narcan] to a postoperative patient experiencing respiratory sedation. What undesirable effect would the nurse anticipate after giving this medication? a. Drowsiness b. Tics and tremors c. Increased pain d. Nausea and vomiting

c. Increased pain Naloxone reverses the effects of narcotics. Although the patient's respiratory status will improve after administration of naloxone, the pain will be more acute.

A 20-kg child has been taking valproic acid [Depakote] for 1 week to treat a seizure disorder. The child is receiving 200 mg PO twice daily. The child's parents report no improvement in seizure activity. The nurse will anticipate that the provider will order which change in this child's drug regimen? a. Adding another seizure medication to supplement the valproic acid b. Changing to phenytoin [Dilantin] since the valproic acid is not effective c. Increasing the dose of valproic acid to 300 mg PO twice daily d. Increasing the dose of valproic acid to 200 mg three times daily

c. Increasing the dose of valproic acid to 300 mg PO twice daily Valproic acid is given initially at a dose of 5 to 15 mg/kg/day, administered in two divided doses. This child is receiving 400 mg/day, which is 10 mg/kg/day. The dosage should be increased by 5 to 10 mg/kg/day each week until optimal levels are achieved up to a maximum dose of 60 mg/kg/day. At this point, adding another AED or changing to another AED is not recommended. Increasing the dose to three times daily is not recommended.

A patient arrives in the emergency department complaining of dizziness, lightheadedness, and a pulsating headache. Further assessment reveals a blood pressure of 82/60 mm Hg and palpitations. The patient's friends tell the nurse that they were experimenting with "poppers." The nurse will expect to administer which medication? a. Diazepam [Valium] b. Haloperidol [Haldol] c. Methylene blue and supplemental oxygen d. Naloxone [Narcan]

c. Methylene blue and supplemental oxygen

A patient reports difficulty staying awake during the daytime in spite of getting adequate sleep every night. Which medication will the nurse expect the provider to order for this patient? a. Caffeine (NoDoz) b. Methylphenidate (Ritalin) c. Modafinil (Provigil) d. Theophylline

c. Modafinil (Provigil)

The nurse prepared to administer which drug to the patient with suspected opioid overdose? a. Glucose b. Caffeine c. Naloxone d. Tramadol

c. Naloxone

A child who receives valproic acid [Depakote] begins taking lamotrigine [Lamictal] because of an increase in the number of seizures. The nurse will observe this child closely for which symptom? a. Angioedema b. Hypohidrosis c. Rash d. Psychosis

c. Rash Lamotrigine can cause life-threatening rashes, such as Stevens-Johnson syndrome and toxic epidermal necrolysis, and this risk increases with concurrent use of valproic acid. Angioedema is an adverse effect associated with pregabalin. Hypohidrosis and psychosis are associated with topiramate.

A patient who has biliary colic reports a pain level of 8 on a 1 to 10 pain scale with 10 being the most severe pain. The patient has an order for ibuprofen as needed for pain. Which action by the nurse is correct? a. Administer the ibuprofen as ordered. b. Contact the provider to discuss nonpharmacologic pain measures. c. Request an order for meperidine [Demerol]. d. Request an order for morphine sulfate.

c. Request an order for meperidine [Demerol]. Opioids can induce spasm of the common bile duct and can cause biliary colic. For patients with existing biliary colic, morphine may intensify the pain. It is important to treat pain, however, and certain opioids, such as meperidine, which cause less smooth muscle spasm, may be given. Ibuprofen is used for mild to moderate pain and is not appropriate for this patient. Nonpharmacologic methods are appropriate when used as adjunctive therapy with an opioid.

A patient is admitted with the following vital signs: 37.6 °C, P 96, R 16, 140/90 mm Hg. If the patient is scheduled for a dose of opioid, which of the vital sign parameters would require an immediate intervention? a. Pulse rate of 110 b. Temperature of 38 °C c. Respiratory rate of 10 d. Blood pressure of 160/86 mm Hg

c. Respiratory rate of 10 The drug should be withheld and the physician notified if respiratory rate is at or below 12 breaths per minute, if blood pressure is significantly below the pretreatment value, or if pulse rate is significantly above or below the pretreatment value.

A patient with cancer has been taking an opioid analgesic four times daily for several months and reports needing increased doses for pain. What will the nurse tell the patient? a. PRN dosing of the drug may be more effective. b. The risk of respiratory depression increases over time. c. The patient should discuss increasing the dose with the provider. d. The patient should request the addition of a benzodiazepine to augment pain relief.

c. The patient should discuss increasing the dose with the provider. This patient is developing tolerance, which occurs over time and is evidenced by the need for a larger dose to produce the effect formerly produced by a smaller dose. This patient should be encouraged to request an increased dose. PRN dosing is less effective than scheduled, around-the-clock dosing. The risk of respiratory depression decreases over time as patients develop tolerance to this effect. Benzodiazepines are CNS depressants and should not be given with opioids, because they increase the risk of oversedation.

A patient with a form of epilepsy that may have spontaneous remission has been taking an AED for a year. The patient reports being seizure free for 6 months and asks the nurse when the drug can be discontinued. What will the nurse tell the patient? a. AEDs must be taken for life to maintain remission. b. Another AED will be substituted for the current AED. c. The provider will withdraw the drug over a 6- to 12-week period. d. The patient should stop taking the AED now and restart the drug if seizures recur.

c. The provider will withdraw the drug over a 6- to 12-week period The most important rule about withdrawing AEDs is that they should be withdrawn slowly over 6 weeks to several months to reduce the risk of status epilepticus (SE). AEDs need not be taken for life if seizures no longer occur. Substituting one AED for another to withdraw AED therapy is not recommended. Stopping an AED abruptly increases the risk of SE.

The nurse includes which information in an inservice about the therapeutic effects of agonist-antagonists? a. These drugs relieve pain better than opioids. b. These drugs have greater potential for abuse than opioids. c. These drugs produce less respiratory depression than opioids. d. These drugs have minimal side effects when compared with opioids.

c. These drugs produce less respiratory depression than opioids.

The nurse closely monitors the patient receiving morphine and a tricyclic antidepressant for which manifestations of drug interactions? a. Bradycardia b. Hyperpyrexia c. Urinary retention d. Respiratory depression

c. Urinary retention Anticholinergic drugs (e.g., antihistamines, tricyclic antidepressants, atropine-like drugs) can exacerbate morphine-induced constipation and urinary retention. Hyperpyrexia is associated with the combination of meperidine (a morphine-like drug) with a monoamine oxidase (MAO) inhibitor. Respiratory depression is exacerbated when an opioid is given with another barbiturate.

The nurse is performing a medication history on a patient who reports using phentermine HCl (Suprenza) 15 mg/day for the past 3 months as an appetite suppressant. The nurse will contact the patient's provider to discuss a. changing the medication to phentermine-topiramate (Qsymia). b. increasing the dose to 37.5 mg/day since tolerance has likely occurred. c. initiating a slow taper of the phentermine. d. stopping the drug immediately since long-term use is not recommended.

c. initiating a slow taper of the phentermine.

A patient who has a long-term addiction to opioids takes an overdose of barbiturates. The nurse preparing to care for this patient will anticipate: a. a severe abstinence syndrome when the effects of the barbiturates are reversed. b. minimal respiratory depression, because the patient has developed a tolerance to opioids. c. observing pinpoint pupils, respiratory depression, and possibly coma in this patient. d. using naloxone [Narcan] to reverse the effects of the barbiturates, because cross-tolerance is likely.

c. observing pinpoint pupils, respiratory depression, and possibly coma in this patient.

A child is diagnosed with absence seizures, and the prescriber orders ethosuximide [Zarontin]. When teaching the child's parents about dosage adjustments for this drug, the nurse will stress the importance of: a. frequent serum drug level monitoring. b. learning as much as possible about the disorder and its treatment. c. recording the number of seizures the child has each day. d. reporting dizziness and drowsiness to the provider.

c. recording the number of seizures the child has each day. Measurements of plasma drug levels are less important than observation of seizure activity for determining effective dosages for absence seizures, because this type of seizure is characterized by as many as several hundred occurrences a day. Keeping a chart of seizure activity is the best way to monitor drug effectiveness when treating absence seizures. Frequent drug level monitoring is important when side effects occur to ensure that drug toxicity is not occurring. Learning about the disorder is an important part of adherence. Dizziness and drowsiness are common side effects that diminish with continued use.

A provider orders clonidine [Catapres] for a patient withdrawing from opioids. When explaining the rationale for this drug choice, the nurse will tell this patient that clonidine [Catapres] is used to: a. prevent opioid craving. b. reduce somnolence and drowsiness. c. relieve symptoms of nausea, vomiting, and diarrhea. d. stimulate autonomic activity.

c. relieve symptoms of nausea, vomiting, and diarrhea.

A woman in labor receives meperidine [Demerol] for pain. The nurse caring for the infant will observe the infant closely for: a. congenital anomalies. b. excessive crying and sneezing. c. respiratory depression. d. tremors and hyperreflexia.

c. respiratory depression. Use of morphine or other opioids during delivery can cause respiratory depression in the neonate, because the drug crosses the placenta. Infants should be monitored for respiratory depression and receive naloxone if needed. Opioids given during delivery do not contribute to birth defects in the newborn. Excessive crying and sneezing and tremors and hyperreflexia are signs of neonatal opioid dependence, which occurs with long-term opioid use by the mother during pregnancy and not with short-term use of these drugs during labor.

A hospitalized patient is complaining of having difficulty sleeping. Which action will the nurse take first to address this problem? a) administer a sedative-hypnotic drug if ordered b) offer tea made with the herbal preparation valerian c) encourage the patient to exercise by walking up and down the halls a few times if tolerated d) provide an environment that is restful, and reduce loud noises

d) provide an environment that is restful, and reduce loud noises

A patient tells a nurse that she drinks wine for its cardioprotective effects. She wants to know how much is safe. The nurse is correct in telling her that: a. "A couple of glasses of wine 1 or 2 days a week will help prevent heart disease." b. "Abstaining from alcohol completely is the best way to prevent heart disease." c. "If you drink only on weekends, you will minimize your risk of developing heart disease." d. "Cardioprotective effects are greatest in people who live an unhealthy lifestyle."

d. "Cardioprotective effects are greatest in people who live an unhealthy lifestyle."

A patient who wants to quit smoking has a prescription for varenicline [Chantix], which will be used with a nicotine patch. The patient asks the nurse why the varenicline is necessary. Which statement by the nurse is correct? a. "It helps patients experiencing withdrawal to sleep better." b. "It helps reduce anxiety and other withdrawal symptoms." c. "It will help reduce the likelihood of addiction to the patch." d. "The drug blocks nicotine's access to 'pleasure' receptors."

d. "The drug blocks nicotine's access to 'pleasure' receptors."

A patient who wants to quit smoking has begun taking varenicline [Chantix]. The patient reports experiencing mood swings and depression and a desire to cause harm to herself. What will the nurse tell this patient? a. "These symptoms are common and will disappear over time." b. "These symptoms may indicate an underlying psychiatric disorder." c. "You may need an increased dose to overcome these symptoms of nicotine withdrawal." d. "You should notify your provider of these symptoms immediately."

d. "You should notify your provider of these symptoms immediately."

A patient who has undergone treatment for alcoholism several times, with relapse occurring shortly after each treatment, has just completed detoxification. The patient requests a medication to help maintain abstinence. Which drug will the nurse expect the provider to prescribe? a. Acamprosate [Campral] b. Chlordiazepoxide [Librium] c. Disulfiram [Antabuse] d. Naltrexone [ReVia]

d. Naltrexone [ReVia]

A patient is brought to the emergency department after a motor vehicle accident. The patient's speech is slurred. The nurse notes the smell of alcohol on the patient's breath and observes hand tremors. The patient's blood alcohol level is 0.4%. The nurse will expect to: a. find that the patient has lost consciousness within a short time. b. administer naltrexone [ReVia] and prepare for gastric lavage. c. give carbamazepine to reduce the risk of seizures. d. provide mechanical ventilation and oxygen.

d. provide mechanical ventilation and oxygen.

A nurse working the night shift begins taking modafinil [Alertec]. The nurse is telling a coworker about the medication. Which statement is correct? a. "I can take it during pregnancy." b. "It doesn't have cardiovascular side effects." c. "It is safe and has no serious adverse effects." d. "It will not interfere with my normal sleep."

d. "It will not interfere with my normal sleep."

A patient who has a seizure disorder is admitted to the hospital after an increase in seizure frequency, and the prescriber orders carbamazepine [Tegretol] 100 mg twice daily to be added to the patient's medication regimen. The nurse reviewing the patient's medical history notes that the patient is already taking lamotrigine [Lamictal] 375 mg twice daily. The nurse will contact the provider to discuss which action? a. Reducing the carbamazepine dose to 50 mg twice daily b. Reducing the lamotrigine dose to 225 mg twice daily c. Increasing the carbamazepine dose to 200 mg twice daily d. Increasing the lamotrigine dose to 500 mg twice daily

d. Increasing the lamotrigine dose to 500 mg twice daily Carbamazepine induces hepatic drug-metabolizing enzymes and can increase the rate at which lamotrigine and other drugs are metabolized; therefore, patients taking any of these drugs would need an increased dose. Reducing the dose of either drug is not indicated. Increasing the dose of carbamazepine may be necessary but only after serum drug levels have been checked

The nurse would recommend treatment with transdermal fentanyl for a patient with which issue? a. Postoperative pain after gastric bypass b. Initial treatment for migraine headaches c. Severe pain caused by cancer metastasis to bone d. Intermittent lower back pain associated with lumbar strain

d. Intermittent lower back pain associated with lumbar strain Transdermal fentanyl is indicated only for persistent severe pain in patients who are already opioid tolerant. Use in nontolerant patients can cause fatal respiratory depression. The patch should not be used in children under 2 years of age or in anyone younger than 18 who weighs less than 110 lb. Also, the patch should not be used for postop pain, intermittent pain, or pain that responds to a less powerful analgesic

A young adult patient is admitted to the hospital for evaluation of severe weight loss. The nurse admitting this patient notes that the patient has missing teeth and severe tooth decay. The patient's blood pressure is 160/98 mm Hg. The patient has difficulty answering questions and has trouble remembering simple details. The nurse suspects abuse of which substance? a. Cocaine b. Ecstasy c. Marijuana d. Methamphetamine

d. Methamphetamine

A postoperative patient who received an intravenous infusion of morphine has a respiratory rate of 8 breaths per minute and is lethargic. Which as-needed medication should the nurse administer to the patient? a. Methadone [Dolophine] b. Nalbuphine [Nubain] c. Tramadol [Ultram] d. Naloxone [Narcan]

d. Naloxone [Narcan]

In reviewing admission orders for a patient with a myocardial infarction, the nurse would question an order for which medication? a. Aspirin b. Morphine c. Butorphanol d. Pentazocine

d. Pentazocine Pentazocine increases cardiac work. Accordingly, a pure agonist (e.g., morphine) is preferred to pentazocine for relieving pain in patients with myocardial infarction.

A patient who has had abdominal surgery has been receiving morphine sulfate via a patient-controlled analgesia (PCA) pump. The nurse assesses the patient and notes that the patient's pupils are dilated and that the patient is drowsy and lethargic. The patient's heart rate is 84 beats per minute, the respiratory rate is 10 breaths per minute, and the blood pressure is 90/50 mm Hg. What will the nurse do? a. Discuss possible opiate dependence with the patient's provider. b. Encourage the patient to turn over and cough and take deep breaths. c. Note the effectiveness of the analgesia in the patient's chart. d. Prepare to administer naloxone and possibly ventilatory support.

d. Prepare to administer naloxone and possibly ventilatory support. Opioid toxicity is characterized by coma, respiratory depression, and pinpoint pupils. Although pupils are constricted initially, they may dilate as hypoxia progresses, which also causes blood pressure to drop. This patient has a respiratory rate of fewer than 12 breaths per minute, dilated pupils, and low blood pressure; the patient also is showing signs of central nervous system (CNS) depression. The nurse should prepare to give naloxone and should watch the patient closely for respiratory collapse. Patients with opioid dependence show withdrawal symptoms when the drug is discontinued. When postoperative patients have adequate analgesia without serious side effects, encouraging patients to turn, cough, and breathe deeply is appropriate. This patient is probably relatively pain free, but providing emergency treatment is the priority.

What is the primary reason for opioid abuse? a. Ease of access b. Initial "rush" similar to orgasm c. Peer pressure d. Prolonged sense of euphoria

d. Prolonged sense of euphoria

A pregnant patient reports using marijuana during her pregnancy. She asks the nurse whether this will affect the fetus. What should the nurse tell her? a. Children born to patients who use marijuana will have smaller brains. b. Neonates born to patients who use marijuana will have withdrawal syndromes. c. Preschool-aged children born to patients who use marijuana are more likely to be hyperactive. d. School-aged children born to patients who use marijuana often have difficulty with memory.

d. School-aged children born to patients who use marijuana often have difficulty with memory.

A university student who is agitated and restless and has tremors is brought to the emergency department. The patient's heart rate is 110 beats per minute, the respiratory rate is 18 breaths per minute, and the blood pressure is 160/95 mm Hg. The patient reports using concentrated energy drinks to stay awake during finals week. What complication will the nurse monitor for in this patient? a. CNS depression b. Cardiac arrest c. Respiratory failure d. Seizures

d. Seizures

A patient who is a heroin addict is admitted to a methadone substitution program. After administering the first dose of methadone, the nurse notes that the patient shows signs of euphoria and complains of nausea. What will the nurse do? a. Administer nalmefene [Revex]. b. Contact the provider to obtain an order for naloxone [Narcan]. c. Question the patient about heroin use that day. d. Suspect that the patient exaggerated the amount of heroin used.

d. Suspect that the patient exaggerated the amount of heroin used.

A patient takes oxycodone [OxyContin] 40 mg PO twice daily for the management of chronic pain. Which intervention should be added to the plan of care to minimize the gastrointestinal adverse effects? a. The patient should take an antacid with each dose. b. The patient should eat foods high in lactobacilli. c. The patient should take the medication on an empty stomach. d. The patient should increase fluid and fiber in the diet.

d. The patient should increase fluid and fiber in the diet.

An adult patient will begin taking atomoxetine [Strattera] for attention-deficit/hyperactivity disorder. What will the nurse teach this patient? a. Appetite suppression does not occur, because this drug is not a stimulant. b. Stopping the drug abruptly will cause an abstinence syndrome. c. Suicidal thoughts may occur and should be reported to the provider. d. Therapeutic effects may not be felt for 1 to 3 weeks after beginning therapy.

d. Therapeutic effects may not be felt for 1 to 3 weeks after beginning therapy.

A patient is brought to the emergency department by friends, who report finding the patient difficult to awaken. The friends report removing two fentanyl transdermal patches from the patient's arm. On admission to the emergency department, the patient has pinpoint pupils and a respiratory rate of 6 breaths per minute. A few minutes after administration of naloxone, the respiratory rate is 8 breaths per minute and the patient's pupils are dilated. The nurse recognizes these symptoms as signs of: a. a mild opioid overdose. b. decreased opioid drug levels. c. improved ventilation. d. worsening hypoxia.

d. worsening hypoxia. The classic triad of symptoms of opioid overdose are coma, respiratory depression, and pinpoint pupils. The pupils may dilate as hypoxia worsens, and this symptom, along with continued respiratory depression (fewer than 12 breaths per minute), indicates worsening hypoxia. Fentanyl is a strong opioid, so this is not likely to be a mild overdose, because the patient was wearing two patches. Fentanyl continues to be absorbed even after the patches are removed because of residual drug in the skin, so the drug levels are not likely to be decreasing. The patient does not have improved ventilation, because the respiratory rate is still fewer than 12 breaths per minute.

A patient with chronic pain has been receiving morphine sulfate but now has decreased pain. The prescriber changes the medication to pentazocine [Talwin]. The nurse will monitor the patient for: a. euphoria. b. hypotension. c. respiratory depression. d. yawning and sweating.

d. yawning and sweating. Pentazocine is an agonist-antagonist opioid, and when given to a patient who is physically dependent on morphine, it can precipitate withdrawal. Yawning and sweating are early signs of opioid withdrawal. Pentazocine does not produce euphoria, hypotension, or respiratory depression.

The selective serotonin reuptake inhibitors (SSRIs) are recommended therapy for a number of psychologic disorders. The nurse identifies the SSRIs as effective for the treatment of patients with which psychologic disorder or disorders? (Select all that apply.) A. Depression B. Panic disorder C. Social anxiety disorder D. Post-traumatic stress disorder E. Obsessive-compulsive disorder

A, B, C, E Neither the SSRIs nor any other drugs, for that matter, have proved effective in the treatment of post-traumatic stress disorder. SSRIs are used to treat the other psychologic disorders listed.

The nurse is teaching a patient who has a new prescription for citalopram (Celexa). Which statement is appropriate to include in the teaching plan? (Select all that apply.) A. "This medication may cause some sexual side effects. Let your healthcare provider know about this if it occurs." B. "When you stop taking this medication, you should not withdraw it abruptly." C. "You will need to move slowly from a sitting to a standing position to prevent dizziness from low blood pressure." D. "This medication often causes drowsiness. You should take it at bedtime." E. "Let your family or your healthcare provider know if you experience a worsening mood, agitation, or increased anxiety."

A, B, E Citalopram (Celexa) and other SSRIs can cause sexual side effects that patients may be hesitant to report. SSRIs should be withdrawn slowly to prevent dizziness, headache, dysphoria, and/or other symptoms of withdrawal. The SSRIs do not generally cause orthostatic hypotension or drowsiness. All antidepressants initially increase the risk of suicide, and patients should be monitored for worsening mood and other signs of suicide risk.

The nurse knows that which statements about postpartum depression are true? (Select all that apply.) A. About 80% of women experience depressive symptoms after giving birth. B. Thyroid insufficiency has been indicated as a contributing factor in postpartum depression. C. Monoamine oxidase inhibitors are the first-line agents of choice for the treatment of postpartum depression. D. Once a woman has had postpartum depression, it will not recur with future deliveries. E. Sertraline (Zoloft) is the drug of choice for treating postpartum depression in breast-feeding mothers.

A, B, E The drug of choice for postpartum depression is an SSRI, such as sertraline (Zoloft), because these drugs are effective, well tolerated, and present little risk of toxicity if taken in overdose. If a woman has responded to another antidepressant in the past, that drug should be used first. The risk of relapse is high, as is the risk of postpartum depression with subsequent pregnancies. The statements in options C and D are false.

The nurse identifies which drug as the principal mood stabilizer used in the treatment of bipolar disorder? (Select all that apply.) A. Lithium B. Risperidone C. Divalproex sodium (Valproate) D. Carbamazepine E. Venlafaxine (Effexor)

A, C, D Lithium, divalproex sodium (Valproate), and carbamazepine are the principal mood stabilizers used in the treatment of bipolar disorder. Risperidone is an antipsychotic used in the management of bipolar disorder. Venlafaxine (Effexor) is an antidepressant used in the treatment of bipolar disorder.

Which statement or statements about the treatment of bipolar disorder does the nurse identify as true? (Select all that apply.) A. Mood stabilizers are used to prevent recurrent manic-depressive episodes. B. Antipsychotics are used to treat depressive episodes. C. Antidepressants should be used with mood stabilizers in the treatment of patients with bipolar depression. D. Lithium and valproate are the preferred mood stabilizers for BPD. E. A lithium level of 2.0 mEq/L is considered therapeutic.

A, C, D The statements in options A, C, and D are true. Antipsychotics are used to treat manic episodes. A lithium level above 1.5 mEq/L is considered to be above the therapeutic index.

When teaching the patient and family about clozapine therapy, which statements should the nurse include? (Select all that apply.) A. "It is important for you to obtain ordered blood tests when taking this medication." B. "Most patients who take this medication lose weight, so you should increase the number of calories you consume each day." C. "If you experience increased urination, increased thirst, or increasd appetite, contact your healthcare provider." D. "Inform your healthcare provider if you are taking any medications to control seizures." E. "Contact your healthcare provider if you experience any unexplained tiredness, shortness of breath, increased respirations, chest pain, or heart palpitations."

A, C, D, E Clozapine can cause agranulocytosis. Patients should be taught that clozapine will not be dispensed without repeated proof of blood counts. Patients taking clozapine are at increased risk of weight gain and dyslipidemia; they should be taught about the risk of weight gain and encouraged to control caloric intake and get regular exercise. Patients should be informed about early signs of infection (fever, sore throat, fatigue, mucous membrane ulceration) and instructed to notify their healthcare provider immediately if these should develop. Patients taking clozapine are at increased risk for the development of diabetes mellitus; they should be taught about symptoms of diabetes (e.g., hyperglycemia, polyuria, polydipsia, polyphagia, dehydration) and instructed to contact the prescriber if these occur. Clozapine should be used with caution in patients with seizure disorders. In rare cases, clozapine causes myocarditis; patients should be informed about the signs and symptoms (e.g., unenplained fatigue, dyspnea, tachypnea, chest pain, palpitations) and advised to seek immediate medical attention if these develop. Clozapine should be withheld until myocarditis has been ruled out. If myocarditis is diagnosed, the drug should never be used again.

The nurse identifies the mechanism of action of the amphetamines as what? (Select all that apply.) A. Causing the release of norepinephrine B. Causing the release of epinephrine C. Causing the release of dopamine D. Inhibiting the reuptake of norepinephrine E. Inhibiting the reuptake of dopamine

A, C, D, E The amphetamines act primarily by causing the release of norepinephrine (NE) and dopamine (DA) and partly by inhibiting the reuptake of both transmitters. These actions take place in the CNS and in peripheral nerves. Most pharmacologic effects result from the release of NE.

When comparing benzodiazepines to barbiturates, the nurse identifies which statement about benzodiazepines as true? (Select all that apply.) A. Benzodiazepines have a high safety profile. B. Benzodiazepines have a significant ability to depress central nervous system (CNS) function. C. Benzodiazepines are associated with a high suicide potential. D. Benzodiazepines have a low ability to cause tolerance. E. Benzodiazepines have a low abuse potential.

A, D, E Statements A, D, and E are true. Compared to barbiturates, benzodiazepines have a low suicide potential and low ability to cause CNS depression.

The nurse is teaching a patient with attention-deficit/hyperactivity disorder (ADHD) about his prescription for methylphenidate (Ritalin LA). Which statement by the patient indicates that the teaching was effective? A. "I will take my medication once per day in the morning." B. "I will chew this medication and take it with my ice cream." C. "I will take this medication right before my evening meal." D. "I will use an alarm to remind me to take this medicine three times daily."

A. "I will take my medication once per day in the morning." Ritalin LA is an extended-release formulation designed to be taken once daily. It should be taken in the morning and should not be crushed or chewed.

A patient has elected to use rasagiline as a monotherapy treatment for PD. Which teaching statement by the nurse is incorrect? A. "Rasagiline may cause insomnia, so monitor your sleeping habits." B. "It is important to avoid tyramine-containing foods." C. "You may be at increased risk for malignant melanoma, so have regular skin checks." D. "You may experience side effects, such as headache, arthralgia, dyspepsia, depression, and flu-like symptoms."

A. "Rasagiline may cause insomnia, so monitor your sleeping habits." Unlike selegiline, rasagiline does not cause insomnia. Rasagiline may pose a risk of hypertensive crisis and hence patients should be instructed to avoid tyramine-containing foods. Rasagiline may increase the risk of malignant melanoma, a potentially deadly cancer of the skin. Periodic monitoring of the skin is recommended. Side effects are headache, arthralgia, dyspepsia, depression, and flu-like symptoms.

The nurse is teaching a patient with a new prescription for alprazolam (Xanax). Which statement is the most appropriate to include in the teaching plan? A. "When it is time to discontinue this drug, you will need to taper it off slowly." B. "Protect your skin from the sun to prevent rash and exaggerated sunburn." C. "Increase your intake of fluid and high-fiber foods to prevent constipation." D. "Take this medication on an empty stomach at least 2 hours after meals."

A. "When it is time to discontinue this drug, you will need to taper it off slowly." Alprazolam (Xanax) is a benzodiazepine for which abrupt discontinuation can precipitate withdrawal symptoms. Patients should withdraw the drug gradually over several weeks. The other statements are not related to alprazolam (Xanax).

Which statement will the nurse include when teaching a patient about atomoxetine (Strattera)? A. Atomoxetine has no potential for abuse. B. Atomoxetine is a central nervous system stimulant. C. Atomoxetine exerts its therapeutic effect by increasing the release of dopamine. D. Atomoxetine is not approved for the treatment of adults with attention-deficit/hyperactivity disorder.

A. Atomoxetine has no potential for abuse. Atomoxetine (Strattera) is a nonstimulant drug. It increases the release of norepinephrine, and it is approved for the treatment of adults with attention-deficit/hyperactivity disorder. It has no potential for abuse.

A family member of a patient who is experiencing a severe manic episode asks the nurse why the patient is receiving an antipsychotic medication. The nurse informs the family member that antipsychotics are used in the treatment of severe manic episodes to do what? A. Help control symptoms during the severe manic episode B. Elevate mood during the severe manic episode C. Produce sedating effects during the severe manic episode D. Reduce the amount of physical pain the patient experiences during the severe manic episode

A. Help control symptoms during the severe manic episode Antipsychotic drugs are given to help control symptoms during severe manic episodes, even if psychotic symptoms are absent. Benzodiazepines are given for their sedating effects. Antidepressants help elevate mood during manic episodes.

Which statement about aripiprazole would the nurse identify as true? A. It is the first representative of a unique class of antipsychotic drugs called dopamine system stabilizers. B. It must be administered on an empty stomach. C. Gynecomastia is a common adverse effect. D. It is safe to use in older adult patients with dementia-related psychosis.

A. It is the first representative of a unique class of antipsychotic drugs called dopamine system stabilizers. Aripiprazole is well absorbed in the presence and absence of food, and gynecomastia is not a side effect. Increased mortality is seen when aripiprazole is used in the treatment of older adult patients with dementia-related psychosis. mine system stabilizers.

The nurse is caring for a patient with a seizure disorder who takes mephobarbital (Mebaral) at bedtime each night to control seizures. Which symptom, if present, would most likely indicate an adverse effect of this drug? A. Morning sedation B. A respiratory rate of 30 breaths per minute C. Constipation D. A blood pressure of 160/88 mm Hg

A. Morning sedation The adverse effects of the barbiturates include respiratory depression, risk of suicide, risk of abuse, and hangover (sedation, impaired judgment, and reduced motor skills).

A distraught patient is admitted to the emergency department with symptoms of palpitations, tachycardia, chest pain, and shortness of breath. The physical examination reveals no physiologic basis for the symptoms. Which diagnosis therefore is most likely? A. Panic disorder B. Bipolar disorder C. Generalized anxiety disorder D. Clinical depression

A. Panic disorder Panic disorder is characterized by symptoms similar to those of a myocardial infarction (MI). Patients often fear losing control and dying and also may experience dizziness, nausea, depersonalization, and tingling or numbness in the hands. Generalized anxiety disorder is characterized by excessive worrying about events, but it also can include trembling, muscle tension, restlessness, palpitations, tachycardia, sweating, and clammy hands. Bipolar disorder is characterized by mood swings with periods of mania and depression.

The nurse is caring for a patient receiving fluoxetine (Prozac) for depression. Which adverse effect is most likely associated with this drug? A. Sexual dysfunction B. Dry mouth C. Orthostatic hypotension D. Bradycardia

A. Sexual dysfunction Fluoxetine (Prozac), a selective serotonin reuptake inhibitor (SSRI), does not cause anticholinergic effects, orthostatic hypotension, or cardiotoxicity, as do the tricyclic antidepressants. The most common adverse effects are sexual dysfunction, nausea, headache, and central nervous system stimulation.

The nurse is caring for a patient with bipolar disorder (BPD) who is taking lithium (Lithobid). Which abnormal laboratory value is most essential for the nurse to communicate to the healthcare provider because this patient is taking lithium? A. Sodium level of 128 mEq/L B. Prothrombin time of 8 seconds C. Blood urea nitrogen level of 25 mg/dL D. Potassium level of 5.6 mEq/L

A. Sodium level of 128 mEq/L The sodium level is well below the normal range of 135 to 145 mEq/L. When the serum sodium level is reduced, lithium excretion also is reduced, and lithium accumulates. Because lithium has a narrow therapeutic index, this is a dangerous situation, which can result in symptoms of toxicity and even death.

The nurse is caring for a group of patients being treated for depression. Why might an SSRI be chosen over a TCA? A. To reduce the risk of suicide with overdose B. To avoid weight gain and other gastrointestinal (GI) effects C. To help prevent sexual dysfunction D. To prevent the risk of serotonin syndrome

A. To reduce the risk of suicide with overdose The SSRIs may be chosen because they have fewer side effects and are safer with overdose. However, the SSRIs can cause sexual dysfunction and weight gain, and they carry a risk of serotonin syndrome.

1. Levothyroxine is available in 88-mcg tablet form. Convert this dose to milligram strength. (do not round) _______

ANS: 0.088 mg One mg equals 1000 mcg. To convert 88 mcg to mg, divide 88 by 1000 to equal 0.088 mg, or move the decimal point to the left three spaces. Do not forget to include the leading zero. DIF: COGNITIVE LEVEL: Applying (Application) REF: N/A TOP: NURSING PROCESS: Implementation

1. An agitated patient is to receive an intravenous dose of diazepam (Valium). The order reads, "Give diazepam, 2 mg, IV push, now. Repeat in 15 minutes if needed." Identify how many milliliters will the nurse administer for this dose. The medication vial contains 5 mg/mL. _______

ANS: 0.4 mL DIF: COGNITIVE LEVEL: Applying (Application) REF: N/A TOP: NURSING PROCESS: Implementation

A patient is to receive codeine, 40 mg subcutaneously, every 6 hours as needed for pain. The solution is available in a concentration of 30 mg/mL. Identify how many milliliters of codeine will be drawn up for this dose. (record answer to one decimal place) _______

ANS: 1.3 mL

1. A patient is to receive codeine, 40 mg subcutaneously, every 6 hours as needed for pain. The solution is available in a concentration of 30 mg/mL. Identify how many milliliters of codeine will be drawn up for this dose. (record answer to one decimal place) _______

ANS: 1.3 mL DIF: COGNITIVE LEVEL: Applying (Application) REF: N/A TOP: NURSING PROCESS: Implementation

1. A 9-year-old child will be receiving carbamazepine (Tegretol) suspension, 200 mL daily. The medication is available in a strength of 100 mg/5 mL. Identify how many milliliters will the nurse give to the patient for each dose. _______

ANS: 10 mL DIF: COGNITIVE LEVEL: Applying (Application) REF: N/A TOP: NURSING PROCESS: Implementation

1. The prescriber writes this order, "Give amantadine (Symmetrel) 100 mg per PEG tube twice a day." The medication is available in a liquid form with a concentration 50 mg/5 mL. Identify how many milliliters will the nurse give with each dose. _______

ANS: 10 mL DIF: COGNITIVE LEVEL: Applying (Application) REF: N/A TOP: NURSING PROCESS: Implementation

2. Digoxin is available in 0.125-mg tablet form. Convert this dose to microgram strength. (do not round) _______

ANS: 125 mcg One mg equals 1000 mcg. To convert 0.125 mg to mcg, multiply by 1000 to equal 125 mcg, or move the decimal point to the right three spaces. DIF: COGNITIVE LEVEL: Applying (Application) REF: N/A TOP: NURSING PROCESS: Implementation

1. A drug dose that delivers 250 mg has a half-life of 5 hours. Identify how much drug will remain in the body after one half-life. _______

ANS: 125 mg A drug's half-life is the time required for one half of an administered dose of a drug to be eliminated by the body, or the time it takes for the blood level of a drug to be reduced by 50%. Therefore, one half of 250 mg equals 125 mg. DIF: COGNITIVE LEVEL: Applying (Application) REF: N/A TOP: NURSING PROCESS: Implementation

1. A patient is to receive midazolam (Versed) 2 mg IV push over 2 minutes just before an endoscopy procedure. The medication is available in a strength of 1 mg/mL. Identify how many milliliters of medication will the nurse draw up into the syringe for this dose. _______

ANS: 2 mL DIF: COGNITIVE LEVEL: Applying (Application) REF: N/A TOP: NURSING PROCESS: Implementation

1. The order reads, "Give atomoxetine (Strattera) 0.5 mg/kg/day once daily in the morning before school." The child weighs 88 pounds. Identify how many milligrams will be administered per dose. _______

ANS: 20 mg DIF: COGNITIVE LEVEL: Applying (Application) REF: N/A TOP: NURSING PROCESS: Implementation

1. A patient is to receive prednisone 7.5 mg PO daily. The tablets are available in a 2.5- mg strength. Identify how many tablets will the patient receive. _______

ANS: 3 tablets DIF: COGNITIVE LEVEL: Applying (Application) REF: N/A TOP: NURSING PROCESS: Implementation

2. The patient is to receive oral guaifenesin (Mucinex) twice a day. Today, the nurse was busy and gave the medication 2 hours after the scheduled dose was due. What type of problem does this represent? a. "Right time" b. "Right dose" c. "Right route" d. "Right medication"

ANS: A "Right time" is correct because the medication was given more than 30 minutes after the scheduled dose was due. "Dose" is incorrect because the dose is not related to the time the medication administration is scheduled. "Route" is incorrect because the route is not affected. "Medication" is incorrect because the medication ordered will not change. DIF: COGNITIVE LEVEL: Applying (Application) REF: p. 11 TOP: NURSING PROCESS: Implementation

7. The nurse is teaching a patient how to self-administer triptan injections for migraine headaches. Which statement by the patient indicates that he needs further teaching? a. "I will take this medication regularly to prevent a migraine headache from occurring." b. "I will take this medication when I feel a migraine headache starting." c. "This medication does not reduce the number of migraines I will have." d. "I will keep a journal to record the headaches I have and how the injections are working."

ANS: A Although they may be taken during aura symptoms by patients who have auras with their headaches, these drugs are not indicated for preventive migraine therapy. The medication is intended to relieve the migraine and not to prevent it or to reduce the number of attacks. The triptans do not reduce the number of migraines a person will have. Journal recordings of headaches and the patient's responses to the medication are helpful. DIF: COGNITIVE LEVEL: Applying (Application) REF: p. 209 TOP: NURSING PROCESS: Implementation

1. A patient who has received some traumatic news is panicking and asks for some medication to help settle down. The nurse anticipates giving which drug that is most appropriate for this situation? a. Diazepam (Valium) b. Zolpidem (Ambien) c. Phenobarbital d. Cyclobenzaprine (Flexeril)

ANS: A Benzodiazepines such as diazepam are used as anxiolytics, or sedatives. Zolpidem is used as a hypnotic for sleep. Phenobarbital is not used as an anxiolytic but is used for seizure control. Cyclobenzaprine is a muscle relaxant and is not used to reduce anxiety. DIF: COGNITIVE LEVEL: Applying (Application) REF: p. 191 TOP: NURSING PROCESS: Planning

1. The nurse reads in the patient's medication history that the patient is taking buspirone (BuSpar). The nurse interprets that the patient may have which disorder? a. Anxiety disorder b. Depression c. Schizophrenia d. Bipolar disorder

ANS: A Buspirone is indicated for the treatment of anxiety disorders, not depression, schizophrenia, or bipolar disorder. DIF: COGNITIVE LEVEL: Understanding (Comprehension) REF: p. 253 TOP: NURSING PROCESS: Implementation

9. A patient taking entacapone (Comtan) for the first time calls the clinic to report a dark discoloration of his urine. After listening to the patient, the nurse realizes that what is happening in this situation? a. This is a harmless effect of the drug. b. The patient has taken this drug along with red wine or cheese. c. The patient is having an allergic reaction to the drug. d. The ordered dose is too high for this patient.

ANS: A COMT inhibitors, including entacapone, may darken a patient's urine and sweat. DIF: COGNITIVE LEVEL: Applying (Application) REF: p. 239 TOP: NURSING PROCESS: Implementation

1. A patient is receiving instructions regarding the use of caffeine. The nurse shares that caffeine should be used with caution if which of these conditions is present? a. A history of peptic ulcers b. Migraine headaches c. Asthma d. A history of kidney stones

ANS: A Caffeine should be used with caution by patients who have histories of peptic ulcers or cardiac dysrhythmias or who have recently had myocardial infarctions. The other conditions are not contraindications to the use of caffeine. DIF: COGNITIVE LEVEL: Understanding (Comprehension) REF: p. 212 TOP: NURSING PROCESS: Assessment

2. The patient is asking the nurse about current U.S. laws and regulations of herbal products. According to the Dietary Supplement and Health Education Act of 1994, which statement is true? a. Medicinal herbs are viewed as dietary supplements. b. Herbal remedies are held to the same standards as drugs. c. Producers of herbal products must prove therapeutic efficacy. d. Herbal remedies are protected by patent laws.

ANS: A Current U.S. laws view herbal products as dietary supplements and do not hold them to the same efficacy standards as drugs. The other options do not correctly reflect current U.S. laws regarding herbal supplements. DIF: COGNITIVE LEVEL: Understanding (Comprehension) REF: p. 89 TOP: NURSING PROCESS: General

9. The nurse notes in the patient's medication history that the patient is taking cyclobenzaprine (Flexeril). Based on this finding, the nurse interprets that the patient has which disorder? a. A musculoskeletal injury b. Insomnia c. Epilepsy d. Agitation

ANS: A Cyclobenzaprine (Flexeril) is the muscle relaxant most commonly used to reduce spasms following musculoskeletal injuries. It is not appropriate for insomnia, epilepsy, or agitation. DIF: COGNITIVE LEVEL: Understanding (Comprehension) REF: p. 197 TOP: NURSING PROCESS: Assessment

6. A 22-year-old patient has been taking lithium for 1 year, and the most recent lithium level is 0.9 mEq/L. Which statement about the laboratory result is correct? a. The lithium level is therapeutic. b. The lithium level is too low. c. The lithium level is too high. d. Lithium is not usually monitored with blood levels.

ANS: A Desirable long-term maintenance lithium levels range between 0.6 and 1.2 mEq/L. The other responses are incorrect. DIF: COGNITIVE LEVEL: Analyzing (Analysis) REF: p. 269 TOP: NURSING PROCESS: Evaluation

8. A patient is experiencing status epilepticus. The nurse prepares to give which drug of choice for the treatment of this condition? a. Diazepam (Valium) b. Midazolam (Versed) c. Valproic acid (Depakote) d. Carbamazepine (Tegretol)

ANS: A Diazepam (Valium) is considered by many to be the drug of choice for status epilepticus. Other drugs that are used are listed in Table 14-3 and do not include the drugs listed in the other options. DIF: COGNITIVE LEVEL: Understanding (Comprehension) REF: p. 221 TOP: NURSING PROCESS: Planning

9. The patient is experiencing chest pain and needs to take a sublingual form of nitroglycerin. Where does the nurse instruct the patient to place the tablet? a. Under the tongue b. On top of the tongue c. At the back of the throat d. In the space between the cheek and the gum

ANS: A Drugs administered via the sublingual route are placed under the tongue. Drugs administered via the buccal route are placed in the space between the cheek and the gum; oral drugs are swallowed. The other options are incorrect. DIF: COGNITIVE LEVEL: Understanding (Comprehension) REF: p. 22 TOP: NURSING PROCESS: Implementation

3. The nurse is administering parenteral drugs. Which statement is true regarding parenteral drugs? a. Parenteral drugs bypass the first-pass effect. b. Absorption of parenteral drugs is affected by reduced blood flow to the stomach. c. Absorption of parenteral drugs is faster when the stomach is empty. d. Parenteral drugs exert their effects while circulating in the bloodstream.

ANS: A Drugs given by the parenteral route bypass the first-pass effect. Reduced blood flow to the stomach and the presence of food in the stomach apply to enteral drugs (taken orally), not to parenteral drugs. Parenteral drugs must be absorbed into cells and tissues from the circulation before they can exert their effects; they do not exert their effects while circulating in the bloodstream. DIF: COGNITIVE LEVEL: Understanding (Comprehension) REF: p. 22 TOP: NURSING PROCESS: General

5. Which activity best reflects the implementation phase of the nursing process for the patient who is newly diagnosed with hypertension? a. Providing education on keeping a journal of blood pressure readings b. Setting goals and outcome criteria with the patient's input c. Recording a drug history regarding over-the-counter medications used at home d. Formulating nursing diagnoses regarding deficient knowledge related to the new treatment regimen

ANS: A Education is an intervention that occurs during the implementation phase. Setting goals and outcomes reflects the planning phase. Recording a drug history reflects the assessment phase. Formulating nursing diagnoses reflects analysis of data as part of planning. DIF: COGNITIVE LEVEL: Applying (Application) REF: pp. 8-9 TOP: NURSING PROCESS: Implementation

12. A patient is taking gabapentin (Neurontin), and the nurse notes that there is no history of seizures on his medical record. What is the best possible rationale for this medication order? a. The medication is used for the treatment of neuropathic pain. b. The medication is helpful for the treatment of multiple sclerosis. c. The medication is used to reduce the symptoms of Parkinson's disease. d. The medical record is missing the correct information about the patient's history of seizures.

ANS: A Gabapentin (Neurontin) is commonly used to treat neuropathic pain. The other options are incorrect. DIF: COGNITIVE LEVEL: Understanding (Comprehension) REF: p. 227 TOP: NURSING PROCESS: Assessment

9. Phenytoin (Dilantin) has a narrow therapeutic index. The nurse recognizes that this characteristic indicates which of these? a. The safe and the toxic plasma levels of the drug are very close to each other. b. The phenytoin has a low chance of being effective. c. There is no difference between safe and toxic plasma levels. d. A very small dosage can result in the desired therapeutic effect.

ANS: A Having a "narrow therapeutic index" means that there is a small difference between safe and toxic drug levels. These drugs require monitoring of therapeutic plasma levels. The other options are incorrect. DIF: COGNITIVE LEVEL: Understanding (Comprehension) REF: p. 222 TOP: NURSING PROCESS: Planning

7. The U.S. Food and Drug Administration has issued a warning for users of antiepileptic drugs. Based on this report, the nurse will monitor for which potential problems with this class of drugs? a. Increased risk of suicidal thoughts and behaviors b. Signs of bone marrow depression c. Indications of drug addiction and dependency d. Increased risk of cardiovascular events, such as strokes

ANS: A In December 2008, the U.S. Food and Drug Administration (FDA) required black box warnings on all antiepileptic drugs regarding the risk of suicidal thoughts and behaviors. Patients being treated with antiepileptic drugs for any indication need to be monitored for the emergence or worsening of depression, suicidal thoughts or behavior, or any unusual changes in mood or behavior. The other options are incorrect. DIF: COGNITIVE LEVEL: Understanding (Comprehension) REF: p. 222 TOP: NURSING PROCESS: Evaluation

5. A patient is recovering from a minor automobile accident that occurred 1 week ago. He is taking cyclobenzaprine (Flexeril) for muscular pain and goes to physical therapy three times a week. Which nursing diagnosis would be appropriate for him? a. Risk for injury related to decreased sensorium b. Risk for addiction related to psychologic dependency c. Decreased fluid volume related to potential adverse effects d. Disturbed sleep pattern related to the drug's interference with REM sleep

ANS: A Musculoskeletal relaxants have a depressant effect on the CNS; thus, the patient needs to be taught the importance of taking measures to minimize self-injury and falls related to decreased sensorium. "Risk for addiction" is not a NANDA nursing diagnosis. The other nursing diagnoses are not appropriate for this situation. DIF: COGNITIVE LEVEL: Applying (Application) REF: p. 198 TOP: NURSING PROCESS: Nursing Diagnosis

6. A patient is taking flurazepam (Dalmane) three to four nights a week for sleeplessness. She is concerned that she cannot get to sleep without taking the medication. What nonpharmacologic measures should the nurse suggest to promote sleep for this patient? a. Providing a quiet environment b. Exercising before bedtime to become tired c. Consuming heavy meals in the evening to promote sleepiness d. Drinking hot tea or coffee just before bedtime

ANS: A Nonpharmacologic approaches to induce sleep include providing a quiet environment, avoiding heavy exercise before bedtime, avoiding heavy meals late in the evening, and drinking warm decaffeinated drinks, such as warm milk, before bedtime. DIF: COGNITIVE LEVEL: Applying (Application) REF: p. 201 TOP: NURSING PROCESS: Implementation

8. The nurse is conducting a smoking-cessation program. Which statement regarding drugs used in cigarette-smoking-cessation programs is true? a. Rapid chewing of the nicotine gum releases an immediate dose of nicotine. b. Quick relief from withdrawal symptoms is most easily achieved by using a transdermal patch. c. Compliance with treatment is higher with use of the gum rather than the transdermal patch. d. The nicotine gum can be used only up to six times per day.

ANS: A Quick or acute relief from withdrawal symptoms is most easily achieved with the use of the gum because rapid chewing of the gum produces an immediate dose of nicotine. However, treatment compliance is higher with the use of the transdermal patch system. Nicotine gum can be used whenever the patient has a strong urge to smoke. DIF: COGNITIVE LEVEL: Applying (Application) REF: p. 285 TOP: NURSING PROCESS: Implementation

2. The nurse has given medication instructions to a patient receiving phenytoin (Dilantin). Which statement by the patient indicates that the patient has an adequate understanding of the instructions? a. "I will need to take extra care of my teeth and gums while on this medication." b. "I can go out for a beer while on this medication." c. "I can skip doses if the side effects bother me." d. "I will be able to stop taking this drug once the seizures stop."

ANS: A Scrupulous dental care is necessary to prevent gingival hypertrophy during therapy with phenytoin. Alcohol and other central nervous system depressants may cause severe sedation. Consistent dosing is important to maintain therapeutic drug levels. Therapy with AEDs usually must continue for life and must not be stopped once seizures stop. DIF: COGNITIVE LEVEL: Analyzing (Analysis) REF: p. 225 TOP: NURSING PROCESS: Evaluation

6. A patient with a new prescription for a diuretic has just reviewed with the nurse how to include more potassium in her diet. This reflects learning in which domain? a. Cognitive b. Affective c. Physical d. Psychomotor

ANS: A The cognitive domain refers to problem-solving abilities and may involve recall and knowledge of facts. The affective domain refers to values and beliefs. The term physical does not refer to one of the learning domains. The psychomotor domain involves behaviors such as learning how to perform a procedure. DIF: COGNITIVE LEVEL: Understanding (Comprehension) REF: p. 75 TOP: NURSING PROCESS: Implementation

11. A patient who has advanced cancer is receiving opioid medications around the clock to keep him comfortable as he nears the end of his life. Which term best describes this type of therapy? a. Palliative therapy b. Maintenance therapy c. Empiric therapy d. Supplemental therapy

ANS: A The goal of palliative therapy is to make the patient as comfortable as possible. It is typically used in the end stages of illnesses when all attempts at curative therapy have failed. Maintenance therapy is used for the treatment of chronic illnesses such as hypertension. Empiric therapy is based on clinical probabilities and involves drug administration when a certain pathologic condition has an uncertain but high likelihood of occurrence based on the patient's initial presenting symptoms. Supplemental (or replacement therapy) supplies the body with a substance needed to maintain normal function. DIF: COGNITIVE LEVEL: Understanding (Comprehension) REF: p. 30 TOP: NURSING PROCESS: Implementation

1. The patient is receiving two different drugs. At current dosages and dosage forms, both drugs are absorbed into the circulation in identical amounts. Which term is used to identify this principle? a. Bioequivalent b. Synergistic c. Prodrugs d. Steady state

ANS: A Two drugs absorbed into the circulation in the same amount (in specific dosage forms) have the same bioavailability; thus, they are bioequivalent. A drug's steady state is the physiologic state in which the amount of drug removed via elimination is equal to the amount of drug absorbed from each dose. The term synergistic refers to two drugs, given together, with a resulting effect that is greater than the sum of the effects of each drug given alone. A prodrug is an inactive drug dosage form that is converted to an active metabolite by various biochemical reactions once it is inside the body. DIF: COGNITIVE LEVEL: Understanding (Comprehension) REF: p. 21 TOP: NURSING PROCESS: Implementation

2. Place the phases of the nursing process in the correct order, with 1 as the first phase and 5 as the last phase. (Select all that apply.) a. Planning b. Evaluation c. Assessment d. Implementation e. Nursing Diagnoses

ANS: A, B, C, D, E The nursing process is an ongoing process that begins with assessing and continues with diagnosing, planning, implementing, and evaluating. DIF: COGNITIVE LEVEL: Applying (Application) REF: p. 4 TOP: NURSING PROCESS: General

1. When giving medications, the nurse will follow the rights of medication administration. The rights include the right documentation, the right reason, the right response, and the patient's right to refuse. Which of these are additional rights? (Select all that apply.) a. Right drug b. Right route c. Right dose d. Right diagnosis e. Right time f. Right patient

ANS: A, B, C, E, F Additional rights of medication administration must always include the right drug, right dose, right time, right route, and right patient. The right diagnosis is incorrect. DIF: COGNITIVE LEVEL: Remembering (Knowledge) REF: p. 9 TOP: NURSING PROCESS: Implementation

2. The barbiturate phenobarbital is prescribed for a patient with epilepsy. While assessing the patient's current medications, the nurse recognizes that interactions may occur with which drugs? (Select all that apply.) a. Antihistamines b. Opioids c. Diuretics d. Anticoagulants e. Oral contraceptives f. Insulin

ANS: A, B, D, E The co-administration of barbiturates and alcohol, antihistamines, benzodiazepines, opioids, and tranquilizers may result in additive CNS depression. Co-administration of anticoagulants and barbiturates can result in decreased anticoagulation response and possible clot formation. Co-administration of barbiturates and oral contraceptives can result in accelerated metabolism of the contraceptive drug and possible unintended pregnancy. There are no interactions with diuretics and insulin. DIF: COGNITIVE LEVEL: Understanding (Comprehension) REF: p. 195 TOP: NURSING PROCESS: Assessment

1. When assessing the medication history of a patient with a new diagnosis of Parkinson's disease, which conditions are contraindications for the patient who will be taking carbidopa-levodopa? (Select all that apply.) a. Angle-closure glaucoma b. History of malignant melanoma c. Hypertension d. Benign prostatic hyperplasia e. Concurrent use of monoamine oxidase inhibitors (MAOIs)

ANS: A, B, E Angle-closure glaucoma, a history of melanoma or other undiagnosed skin conditions, and concurrent use of MAOIs are contraindications to the use of carbidopa-levodopa. The other options are incorrect. DIF: COGNITIVE LEVEL: Applying (Application) REF: pp. 242-243 TOP: NURSING PROCESS: Assessment

1. The nurse is preparing to administer dexmedetomidine (Precedex) to a patient. Which is an appropriate indication for dexmedetomidine? (Select all that apply.) a. Procedural sedation b. Surgeries of short duration c. Surgeries of long duration d. Postoperative anxiety e. Sedation of mechanically ventilated patients

ANS: A, B, E Dexmedetomidine (Precedex) is used for procedural sedation and for surgeries of short duration, and it is also used in the intensive care setting for sedation of mechanically ventilated patients. The other options are incorrect. DIF: COGNITIVE LEVEL: Applying (Application) REF: p. 174 TOP: NURSING PROCESS: Planning

2. The nurse is reviewing the criteria for over-the-counter drugs. Which criteria for over-the-counter status in the United States are accurate? (Select all that apply.) a. The drug must be easy to use. b. The drug must have a low therapeutic index. c. The consumer must be able to monitor the drug's effectiveness. d. The drug must have a low potential for abuse. e. The drug must not have any interactions with other drugs.

ANS: A, C, D In the United States, criteria for over-the-counter status include the drug being easy to use, the drug having a low potential for abuse, and the consumer must be able to monitor the drug's effectiveness for the condition. The drug must have a high therapeutic index (not a low one), and the drug must have limited interactions with other drugs. DIF: COGNITIVE LEVEL: Applying (Application) REF: p. 87 TOP: NURSING PROCESS: General

3. The nurse is monitoring a patient who is experiencing severe ethanol withdrawal. Which are signs and symptoms of severe ethanol withdrawal? (Select all that apply.) a. Agitation b. Drowsiness c. Tremors d. Systolic blood pressure higher than 200 mm Hg e. Temperature over 100° F (37.7° C) f. Pulse rate 110 beats/min

ANS: A, C, D Signs and symptoms of severe ethanol withdrawal (delirium tremens) include systolic blood pressure higher than 200 mm Hg, diastolic blood pressure higher than 140 mm Hg, pulse rate higher than 140 beats/min, temperature above 101° F (38.3° C), tremors, insomnia, and agitation. See Box 17-6 for all signs and symptoms of ethanol withdrawal. DIF: COGNITIVE LEVEL: Applying (Application) REF: p. 283 TOP: NURSING PROCESS: Assessment

1. Which are appropriate considerations when the nurse is assessing the learning needs of a patient? (Select all that apply.) a. Cultural background b. Family history c. Level of education d. Readiness to learn e. Health beliefs

ANS: A, C, D, E Family history is not a part of what the nurse considers when assessing learning needs. The other options are appropriate to consider when the nurse is assessing learning needs. DIF: COGNITIVE LEVEL: Understanding (Comprehension) REF: pp. 75-77 TOP: NURSING PROCESS: Assessment

1. A patient who has been taking a selective serotonin reuptake inhibitor (SSRI) is complaining of "feeling so badly" when he started taking an over-the-counter St. John's wort herbal product at home. The nurse suspects that he is experiencing serotonin syndrome. Which of these are symptoms of serotonin syndrome? (Select all that apply.) a. Agitation b. Drowsiness c. Tremors d. Bradycardia e. Sweating f. Constipation

ANS: A, C, E Common symptoms of serotonin syndrome include delirium, agitation, tachycardia, sweating, hyperreflexia, shivering, coarse tremors, and others. See Box 16-1 for a full list of symptoms. DIF: COGNITIVE LEVEL: Understanding (Comprehension) REF: p. 260 TOP: NURSING PROCESS: Assessment

1. The nurse is conducting a class for senior citizens about the use of over-the-counter (OTC) drugs. Which statements are true regarding the use of OTC drugs? (Select all that apply.) a. Use of OTC drugs may delay treatment of serious ailments. b. Drug interactions with OTC medications are rare. c. OTC drugs may relieve symptoms without addressing the cause of the problem. d. OTC drugs are indicated for long-term treatment of conditions. e. Patients may misunderstand product labels and use the drugs improperly.

ANS: A, C, E It is true that use of OTC drugs may delay treatment of serious ailments; OTC drugs may relieve symptoms without addressing the cause of the problem, and patients may misunderstand product labels and use the drugs improperly. These statements should be included when teaching patients about their use. In contrast, drug interactions with OTC medications are not rare and may indeed occur with prescription medications and other OTC drugs. Normally, OTC drugs are intended for short-term treatment of minor ailments. DIF: COGNITIVE LEVEL: Applying (Application) REF: pp. 86-88 TOP: NURSING PROCESS: Implementation

1. The nurse is reviewing the use of central nervous system stimulants. Which of these are indications for this class of drugs? (Select all that apply.) a. Narcolepsy b. Depression c. Panic attacks d. Neonatal apnea e. Attention deficit hyperactivity disorder (ADHD) f. Appetite suppression

ANS: A, D, E, F Central nervous system stimulants can be used for narcolepsy, neonatal apnea, ADHD, and appetite suppression in the treatment of obesity. They are not used for depression and panic attacks. DIF: COGNITIVE LEVEL: Understanding (Comprehension) REF: p. 204 TOP: NURSING PROCESS: Assessment

1. The nurse is reviewing medication errors. Which situation is an example of a medication error? a. A patient refuses her morning medications. b. A patient receives a double dose of a medication because the nurse did not cut the pill in half. c. A patient develops hives after having started an IV antibiotic 24 hours earlier. d. A patient complains of severe pain still present 60 minutes after a pain medication was given.

ANS: B A medication error is defined as a preventable adverse drug event that involves inappropriate medication use by a patient or health care provider. The other options are not preventable. The patient's refusing to take medications and complaining of pain after a medication is given are patient behaviors, and the development of hives is a possible allergic reaction. DIF: COGNITIVE LEVEL: Applying (Application) REF: p. 67 TOP: NURSING PROCESS: Implementation

10. A patient has been taking an AED for several years as part of his treatment for partial seizures. His wife has called because he ran out of medication this morning and wonders if he can go without it for a week until she has a chance to go to the drugstore. What is the nurse's best response? a. "He is taking another antiepileptic drug, so he can go without the medication for a week." b. "Stopping this medication abruptly may cause withdrawal seizures. A refill is needed right away." c. "He can temporarily increase the dosage of his other antiseizure medications until you get the refill." d. "He can stop all medications because he has been treated for several years now."

ANS: B Abrupt discontinuation of antiepileptic drugs can lead to withdrawal seizures. The other options are incorrect. The nurse cannot change the dose or stop the medication without a prescriber's order. DIF: COGNITIVE LEVEL: Applying (Application) REF: p. 220 TOP: NURSING PROCESS: Implementation

3. A 50-year-old man who has been taking phenobarbital for 1 week is found very lethargic and unable to walk after eating out for dinner. His wife states that he has no other prescriptions and that he did not take an overdose—the correct number of pills is in the bottle. The nurse suspects that which of these may have happened? a. He took a multivitamin. b. He drank a glass of wine. c. He took a dose of aspirin. d. He developed an allergy to the drug.

ANS: B Alcohol has an additive effect when combined with barbiturates and causes central nervous system (CNS) depression. Multivitamins and aspirin do not interact with barbiturates, and this situation does not illustrate an allergic reaction. DIF: COGNITIVE LEVEL: Applying (Application) REF: p. 195 TOP: NURSING PROCESS: Assessment

11. A patient is receiving gabapentin (Neurontin), an anticonvulsant, but has no history of seizures. The nurse expects that the patient is receiving this drug for which condition? a. Inflammation pain b. Pain associated with peripheral neuropathy c. Depression associated with chronic pain d. Prevention of seizures

ANS: B Anticonvulsants are often used as adjuvants for treatment of neuropathic pain to enhance analgesic efficacy. The other indications listed are not correct. DIF: COGNITIVE LEVEL: Understanding (Comprehension) REF: p. 166 TOP: NURSING PROCESS: Planning

A patient is receiving gabapentin (Neurontin), an anticonvulsant, but has no history of seizures. The nurse expects that the patient is receiving this drug for which condition? a. Inflammation pain b. Pain associated with peripheral neuropathy c. Depression associated with chronic pain d. Prevention of seizures

ANS: B Anticonvulsants are often used as adjuvants for treatment of neuropathic pain to enhance analgesic efficacy. The other indications listed are not correct. DIF: COGNITIVE LEVEL: Understanding (Comprehension) REF: p. 166 TOP: NURSING PROCESS: Planning MSC: NCLEX: Physiological Integrity: Pharmacological and Parenteral Therapies

3. During a nursing assessment, which question by the nurse allows for greater clarification and additional discussion with the patient? a. "Are you allergic to penicillin?" b. "What medications do you take?" c. "Have you had a reaction to this drug?" d. "Are you taking this medication with meals?"

ANS: B Asking "What medications do you take?" is an open-ended question that will encourage greater clarification and additional discussion with the patient. The other options are examples of closed-ended questions, which prompt only a "yes" or "no" answer and provide limited information. DIF: COGNITIVE LEVEL: Applying (Application) REF: p. 77 TOP: NURSING PROCESS: Assessment

5. A 10-year-old patient will be started on methylphenidate hydrochloride (Ritalin) therapy. The nurse will perform which essential baseline assessment before this drug is started? a. Eye examination b. Height and weight c. Liver function studies d. Hearing test

ANS: B Assessment of baseline height and weight is important before beginning Ritalin therapy because it may cause a temporary slowing of growth in prepubertal children. The other studies are not as essential at this time. DIF: COGNITIVE LEVEL: Understanding (Comprehension) REF: p. 212 TOP: NURSING PROCESS: Assessment

1. A patient has been taking selegiline (Eldepryl), 20 mg/day for 1 month. Today, during his office visit, he tells the nurse that he forgot and had a beer with dinner last evening, and "felt awful." What did the patient most likely experience? a. Hypotension b. Hypertension c. Urinary discomfort d. Gastrointestinal upset

ANS: B At doses that exceed 10 mg/day, selegiline becomes a nonselective monoamine oxidase inhibitor (MAOI), contributing to the development of the cheese effect, so-called because it interacts with tyramine-containing foods (cheese, red wine, beer, and yogurt) and can cause severe hypertension. DIF: COGNITIVE LEVEL: Understanding (Comprehension) REF: p. 238 TOP: NURSING PROCESS: Evaluation

1. A patient was diagnosed with pancreatic cancer last month, and has complained of a dull ache in the abdomen for the past 4 months. This pain has been gradually increasing, and the pain relievers taken at home are no longer effective. What type of pain is the patient experiencing? a. Acute pain b. Chronic pain c. Somatic pain d. Neuropathic pain

ANS: B Chronic pain is associated with cancer and is characterized by slow onset, long duration, and dull, persistent aching. The patient's symptoms are not characteristics of acute pain, somatic pain, or neuropathic pain. DIF: COGNITIVE LEVEL: Understanding (Comprehension) REF: p. 145 TOP: NURSING PROCESS: Assessment

A patient was diagnosed with pancreatic cancer last month, and has complained of a dull ache in the abdomen for the past 4 months. This pain has been gradually increasing, and the pain relievers taken at home are no longer effective. What type of pain is the patient experiencing? a. Acute pain b. Chronic pain c. Somatic pain d. Neuropathic pain

ANS: B Chronic pain is associated with cancer and is characterized by slow onset, long duration, and dull, persistent aching. The patient's symptoms are not characteristics of acute pain, somatic pain, or neuropathic pain. DIF: COGNITIVE LEVEL: Understanding (Comprehension) REF: p. 145 TOP: NURSING PROCESS: Assessment MSC: NCLEX: Physiological Integrity: Basic Care and Comfort

1. A 38-year-old male patient stopped smoking 6 months ago. He tells the nurse that he still feels strong cigarette cravings and wonders if he is ever going to feel "normal" again. Which statement by the nurse is correct? a. "It's possible that these cravings will never stop." b. "These cravings may persist for several months." c. "The cravings tell us that you are still using nicotine." d. "The cravings show that you are about to experience nicotine withdrawal."

ANS: B Cigarette cravings may persist for months after nicotine withdrawal. The other statements are false. DIF: COGNITIVE LEVEL: Applying (Application) REF: p. 285 TOP: NURSING PROCESS: Implementation

10. While a patient is receiving drug therapy for Parkinson's disease, the nurse monitors for dyskinesia, which is manifested by which finding? a. Rigid, tense muscles b. Difficulty in performing voluntary movements c. Limp extremities with weak muscle tone d. Confusion and altered mental status

ANS: B Dyskinesia is the difficulty in performing voluntary movements that is experienced by some patients with Parkinson's disease. The other options are incorrect. DIF: COGNITIVE LEVEL: Understanding (Comprehension) REF: p. 237 TOP: NURSING PROCESS: Assessment

14. A patient wants to take a ginseng dietary supplement. The nurse instructs the patient to look for which potential adverse effect? a. Drowsiness b. Palpitations and anxiety c. Dry mouth d. Constipation

ANS: B Elevated blood pressure, chest pain or palpitations, anxiety, insomnia, headache, nausea, vomiting, and diarrhea are potential adverse effects of ginseng. Drowsiness, difficulty with urination, and constipation are not potential adverse effects of ginseng. DIF: COGNITIVE LEVEL: Understanding (Comprehension) REF: p. 269 TOP: NURSING PROCESS: Implementation

9. The nurse is reviewing herbal therapies. Which is a common use of the herb feverfew? a. Muscle aches b. Migraine headaches c. Leg cramps d. Incision pain after surgery

ANS: B Feverfew is commonly used for migraine headaches, menstrual problems, arthritis, and fever. Possible adverse effects include muscle stiffness and muscle and joint pain. DIF: COGNITIVE LEVEL: Remembering (Knowledge) REF: p. 162 TOP: NURSING PROCESS: Planning

The nurse is reviewing herbal therapies. Which is a common use of the herb feverfew? a. Muscle aches b. Migraine headaches c. Leg cramps d. Incision pain after surgery

ANS: B Feverfew is commonly used for migraine headaches, menstrual problems, arthritis, and fever. Possible adverse effects include muscle stiffness and muscle and joint pain. DIF: COGNITIVE LEVEL: Remembering (Knowledge) REF: p. 162 TOP: NURSING PROCESS: Planning MSC: NCLEX: Physiological Integrity: Pharmacological and Parenteral Therapies

4. The nurse is presenting a substance-abuse lecture for teenage girls and is asked about "roofies." The nurse recognizes that this is the slang term for which substance? a. Cocaine b. Flunitrazepam c. Secobarbital d. Methamphetamine

ANS: B Flunitrazepam is a benzodiazepine that has recently gained popularity as a recreational drug and is commonly called roofies (the "date-rape" drug). The other drugs are not known as roofies. DIF: COGNITIVE LEVEL: Remembering (Knowledge) REF: p. 281 TOP: NURSING PROCESS: Implementation

8. A 57-year-old woman being treated for end-stage breast cancer has been using a transdermal opioid analgesic as part of the management of pain. Lately, she has been experiencing breakthrough pain. The nurse expects this type of pain to be managed by which of these interventions? a. Administering NSAIDs b. Administering an immediate-release opioid c. Changing the opioid route to the rectal route d. Making no changes to the current therapy

ANS: B If a patient is taking long-acting opioid analgesics, breakthrough pain must be treated with an immediate-release dosage form that is given between scheduled doses of the long-acting opioid. The other options are not appropriate actions. DIF: COGNITIVE LEVEL: Applying (Application) REF: p. 147 TOP: NURSING PROCESS: Planning

A 57-year-old woman being treated for end-stage breast cancer has been using a transdermal opioid analgesic as part of the management of pain. Lately, she has been experiencing breakthrough pain. The nurse expects this type of pain to be managed by which of these interventions? a. Administering NSAIDs b. Administering an immediate-release opioid c. Changing the opioid route to the rectal route d. Making no changes to the current therapy

ANS: B If a patient is taking long-acting opioid analgesics, breakthrough pain must be treated with an immediate-release dosage form that is given between scheduled doses of the long-acting opioid. The other options are not appropriate actions. DIF: COGNITIVE LEVEL: Applying (Application) REF: p. 147 TOP: NURSING PROCESS: Planning MSC: NCLEX: Physiological Integrity: Pharmacological and Parenteral Therapies

4. During a period of time when the computerized medication order system was down, the prescriber wrote admission orders, and the nurse is transcribing them. The nurse is having difficulty transcribing one order because of the prescriber's handwriting. Which is the best action for the nurse to take at this time? a. Ask a colleague what the order says. b. Contact the prescriber to clarify the order. c. Wait until the prescriber makes rounds again to clarify the order. d. Ask the patient what medications he takes at home.

ANS: B If a prescriber writes an order that is illegible, the nurse should contact the prescriber for clarification. Asking a colleague is not useful because the colleague did not write the order. Waiting for the prescriber to return is incorrect because it would delay implementation of the order. Asking the patient about medications is incorrect because this question will not clarify the current order written by the prescriber. DIF: COGNITIVE LEVEL: Applying (Application) REF: p. 69 TOP: NURSING PROCESS: Implementation

9. While monitoring a depressed patient who has just started SSRI antidepressant therapy, the nurse will observe for which problem during the early time frame of this therapy? a. Hypertensive crisis b. Self-injury or suicidal tendencies c. Extrapyramidal symptoms d. Loss of appetite

ANS: B In 2005, the U.S. Food and Drug Administration (FDA) issued special black-box warnings regarding the use of all classes of antidepressants in both adult and pediatric patient populations. Data from the FDA indicated a higher risk for suicide in patients receiving these medications. As a result, current recommendations for all patients receiving antidepressants include regular monitoring for signs of worsening depressive symptoms, especially when the medication is started or the dosage is changed. The other options are incorrect. DIF: COGNITIVE LEVEL: Applying (Application) REF: p. 256 TOP: NURSING PROCESS: Evaluation

13. The nurse will be injecting a drug into the fatty tissue of the patient's abdomen. Which route does this describe? a. Intradermal b. Subcutaneous c. Intramuscular d. Transdermal

ANS: B Injections into the fatty subcutaneous tissue under the dermal layer of skin are referred to as subcutaneous injections. Injections under the more superficial skin layers immediately underneath the epidermal layer of skin and into the dermal layer are known as intradermal injections. Injections into the muscle beneath the subcutaneous fatty tissue are referred to as intramuscular injections. Transdermal drugs are applied to the skin via an adhesive patch. DIF: COGNITIVE LEVEL: Remembering (Knowledge) REF: p. 24 TOP: NURSING PROCESS: Implementation

6. The nurse is giving an intravenous dose of phenytoin (Dilantin). Which action is correct when administering this drug? a. Give the dose as a fast intravenous (IV) bolus. b. Mix the drug with normal saline, and give it as a slow IV push. c. Mix the drug with dextrose (D5W), and give it as a slow IV push. d. Mix the drug with any available solution as long as the administration rate is correct.

ANS: B Intravenous phenytoin is given only with normal saline solution to prevent precipitation formation caused by incompatibilities. The IV push dose must be given slowly (not exceeding 50 mg/min in adults), and the patient must be monitored for bradycardia and decreased blood pressure. DIF: COGNITIVE LEVEL: Applying (Application) REF: p. 226 TOP: NURSING PROCESS: Implementation

6. A patient is being prepared for an oral endoscopy, and the nurse anesthetist reminds him that he will be awake during the procedure but probably will not remember it. What type of anesthetic technique is used in this situation? a. Local anesthesia b. Moderate sedation c. Topical anesthesia d. Spinal anesthesia

ANS: B Moderate sedation effectively reduces patient anxiety, sensitivity to pain, and recall of the medical procedure, yet it preserves a patient's ability to maintain his or her own airway and respond to verbal commands. The other options are incorrect. DIF: COGNITIVE LEVEL: Understanding (Comprehension) REF: p. 175 TOP: NURSING PROCESS: Implementation

3. A patient is recovering from abdominal surgery, which he had this morning. He is groggy but complaining of severe pain around his incision. What is the most important assessment data to consider before the nurse administers a dose of morphine sulfate to the patient? a. His pulse rate b. His respiratory rate c. The appearance of the incision d. The date of his last bowel movement

ANS: B One of the most serious adverse effects of opioids is respiratory depression. The nurse must assess the patient's respiratory rate before administering an opioid. The other options are incorrect. DIF: COGNITIVE LEVEL: Applying (Application) REF: p. 153 TOP: NURSING PROCESS: Assessment

A patient is recovering from abdominal surgery, which he had this morning. He is groggy but complaining of severe pain around his incision. What is the most important assessment data to consider before the nurse administers a dose of morphine sulfate to the patient? a. His pulse rate b. His respiratory rate c. The appearance of the incision d. The date of his last bowel movement

ANS: B One of the most serious adverse effects of opioids is respiratory depression. The nurse must assess the patient's respiratory rate before administering an opioid. The other options are incorrect. DIF: COGNITIVE LEVEL: Applying (Application) REF: p. 153 TOP: NURSING PROCESS: Assessment MSC: NCLEX: Physiological Integrity: Reduction of Risk Potential

6. A patient has been treated for lung cancer for 3 years. Over the past few months, the patient has noticed that the opioid analgesic is not helping as much as it had previously and more medication is needed for the same pain relief. The nurse is aware that this patient is experiencing which of these? a. Opioid addiction b. Opioid tolerance c. Opioid toxicity d. Opioid abstinence syndrome

ANS: B Opioid tolerance is a common physiologic result of long-term opioid use. Patients with opioid tolerance require larger doses of the opioid agent to maintain the same level of analgesia. This situation does not describe toxicity (overdose), addiction, or abstinence syndrome (withdrawal). DIF: COGNITIVE LEVEL: Understanding (Comprehension) REF: p. 147 TOP: NURSING PROCESS: Evaluation

A patient has been treated for lung cancer for 3 years. Over the past few months, the patient has noticed that the opioid analgesic is not helping as much as it had previously and more medication is needed for the same pain relief. The nurse is aware that this patient is experiencing which of these? a. Opioid addiction b. Opioid tolerance c. Opioid toxicity d. Opioid abstinence syndrome

ANS: B Opioid tolerance is a common physiologic result of long-term opioid use. Patients with opioid tolerance require larger doses of the opioid agent to maintain the same level of analgesia. This situation does not describe toxicity (overdose), addiction, or abstinence syndrome (withdrawal). DIF: COGNITIVE LEVEL: Understanding (Comprehension) REF: p. 147 TOP: NURSING PROCESS: Evaluation MSC: NCLEX: Physiological Integrity: Pharmacological and Parenteral Therapies

2. A patient in a rehabilitation center is beginning to experience opioid withdrawal symptoms. The nurse expects to administer which drug as part of the treatment? a. Diazepam (Valium) b. Methadone c. Disulfiram (Antabuse) d. Bupropion (Zyban)

ANS: B Opioid withdrawal can be managed with either methadone or clonidine (Catapres). Diazepam and disulfiram are used for treatment of alcoholism, and bupropion is used to assist with smoking cessation. DIF: COGNITIVE LEVEL: Remembering (Knowledge) REF: p. 279 TOP: NURSING PROCESS: Planning

11. The wife of a patient who has been diagnosed with depression calls the office and says, "It's been an entire week since he started that new medicine for his depression, and there's no change! What's wrong with him?" What is the nurse's best response? a. "The medication may not be effective for him. He may need to try another type." b. "It may take up to 6 weeks to notice any therapeutic effects. Let's wait a little longer to see how he does." c. "It sounds like the dose is not high enough. I'll check about increasing the dosage." d. "Some patients never recover from depression. He may not respond to this therapy."

ANS: B Patients and family members need to be told that antidepressant drugs commonly require several weeks before full therapeutic effects are noted. The other answers are incorrect. DIF: COGNITIVE LEVEL: Analyzing (Analysis) REF: p. 257 TOP: NURSING PROCESS: Implementation

7. The nurse is developing a care plan for a patient who is taking an anticholinergic drug. Which nursing diagnosis would be appropriate for this patient? a. Diarrhea b. Urinary retention c. Risk for infection d. Disturbed sleep pattern

ANS: B Patients receiving anticholinergic drugs are at risk for urinary retention and constipation, not diarrhea. The other nursing diagnoses are not applicable to anticholinergic drugs. DIF: COGNITIVE LEVEL: Applying (Application) REF: p. 245 TOP: NURSING PROCESS: Nursing Diagnosis

1. The nurse is reviewing the dosage schedule for several different antiepileptic drugs (AEDs). Which antiepileptic drug allows for once-a-day dosing? a. Levetiracetam (Keppra) b. Phenobarbital c. Valproic acid (Depakote) d. Gabapentin (Neurontin)

ANS: B Phenobarbital has the longest half-life of all standard AEDs, including those listed in the other options, so it allows for once-a-day dosing. DIF: COGNITIVE LEVEL: Understanding (Comprehension) REF: p. 224 TOP: NURSING PROCESS: Assessment

12. Chlorpromazine (Thorazine) is prescribed for a patient, and the nurse provides instructions to the patient about the medication. The nurse includes which information? a. The patient needs to avoid caffeine while on this drug. b. The patient needs to wear sunscreen while outside because of photosensitivity. c. Long-term therapy may result in nervousness and excitability. d. The medication may be taken with an antacid to reduce gastrointestinal upset.

ANS: B Sun exposure and tanning booths need to be avoided with conventional antipsychotics because of the adverse effect of photosensitivity. Instruct the patient to apply sunscreen liberally and to wear sun-protective clothing and hats. DIF: COGNITIVE LEVEL: Applying (Application) REF: p. 264 TOP: NURSING PROCESS: Implementation

7. During the immediate postoperative period, the Post Anesthesia Care Unit nurse is assessing a patient who had hip surgery. The patient is experiencing tachycardia, tachypnea, and muscle rigidity, and his temperature is 103° F (39.4° C). The nurse will prepare for what immediate treatment? a. Naltrexone hydrochloride (Narcan) injection, an opioid reversal drug b. Dantrolene (Dantrium) injection, a skeletal muscle relaxant c. An anticholinesterase drug, such as neostigmine d. Cardiopulmonary resuscitation (CPR) and intubation

ANS: B Tachycardia, tachypnea, muscle rigidity, and raised temperature are symptoms of malignant hyperthermia, which is treated with cardiorespiratory supportive care as needed to stabilize heart and lung function as well as with immediate treatment with the skeletal muscle relaxant dantrolene. CPR is not immediately needed because the patient still has a pulse and respirations. Naltrexone and anticholinesterase drugs are not appropriate in this situation. DIF: COGNITIVE LEVEL: Applying (Application) REF: p. 174 TOP: NURSING PROCESS: Implementation

5. The nurse is asking a patient about his family history as part of an assessment. Which component is included in an effective family history? a. Asking the patient about the current and past health status of the patient's children b. Covering at least three generations of family history c. Obtaining a family history of the patient's spouse d. Asking about the family history for the patient's siblings and parents only

ANS: B The family history is most effective if it covers at least three generations and includes the current and past health status of each family member. The other options are incorrect. DIF: COGNITIVE LEVEL: Applying (Application) REF: p. 100 TOP: NURSING PROCESS: Assessment

3. When a patient is receiving a second-generation antipsychotic drug, such as risperidone (Risperdal), the nurse will monitor for which therapeutic effect? a. Fewer panic attacks b. Decreased paranoia and delusions c. Decreased feeling of hopelessness d. Improved tardive dyskinesia

ANS: B The therapeutic effects of the antipsychotic drugs include improvement in mood and affect, and alleviation or decrease in psychotic symptoms (decrease in hallucinations, paranoia, delusions, garbled speech). Tardive dyskinesia is a potential adverse effect of these drugs. The other options are incorrect. DIF: COGNITIVE LEVEL: Applying (Application) REF: p. 272 TOP: NURSING PROCESS: Evaluation

7. A patient is being treated for ethanol alcohol abuse in a rehabilitation center. The nurse will include which information when teaching him about disulfiram (Antabuse) therapy? a. He should not smoke cigarettes while on this drug. b. He needs to know about the common over-the-counter substances that contain alcohol. c. This drug will cause the same effects as the alcohol did, without the euphoric effects. d. Mouthwashes and cough medicines that contain alcohol are safe because they are used in small amounts.

ANS: B The use of disulfiram (Antabuse) with alcohol-containing over-the-counter products will elicit severe adverse reactions. As little as 7 mL of alcohol may cause symptoms in a sensitive person. Cigarette smoking does not cause problems when taking disulfiram. Disulfiram does not have the same effects as alcohol. DIF: COGNITIVE LEVEL: Applying (Application) REF: p. 284 TOP: NURSING PROCESS: Implementation

5. During an assessment, the patient tells the nurse that he eats large amounts of garlic for its cardiovascular benefits. Which drug or drug class, if taken, would have a potential interaction with the garlic? a. Acetaminophen (Tylenol) b. Insulin c. Antilipemic drugs d. Sedatives

ANS: B The use of garlic may interfere with hypoglycemic drugs. The other options are incorrect because acetaminophen, antilipemic drugs, and sedatives do not have interactions with garlic. DIF: COGNITIVE LEVEL: Understanding (Comprehension) REF: p. 90 TOP: NURSING PROCESS: Planning

7. A patient wants to take the herb gingko to help his memory. The nurse reviews his current medication list and would be concerned about potential interactions if he is taking a medication from which class of drugs? a. Digitalis b. Anticoagulants c. Sedatives d. Immunosuppressants

ANS: B The use of gingko increases the risk of bleeding with anticoagulants (warfarin, heparin) and antiplatelets (aspirin, clopidogrel). The other concerns do not occur with gingko supplements. DIF: COGNITIVE LEVEL: Applying (Application) REF: p. 90 TOP: NURSING PROCESS: Planning

2. Before beginning a patient's therapy with selective serotonin reuptake inhibitor (SSRI) antidepressants, the nurse will assess for concurrent use of which medications or medication class? a. Aspirin b. Anticoagulants c. Diuretics d. Nonsteroidal anti-inflammatory drugs

ANS: B Use of selective serotonin reuptake inhibitor (SSRI) antidepressants with warfarin results in an increased anticoagulant effect. SSRI antidepressants do not interact with the other drugs or drug classes listed. See Table 16-6 for important drug interactions with SSRIs. DIF: COGNITIVE LEVEL: Understanding (Comprehension) REF: p. 258 TOP: NURSING PROCESS: Assessment

2. A patient has been given a prescription for levodopa-carbidopa (Sinemet) for her newly diagnosed Parkinson's disease. She asks the nurse, "Why are there two drugs in this pill?" The nurse's best response reflects which fact? a. Carbidopa allows for larger doses of levodopa to be given. b. Carbidopa prevents the breakdown of levodopa in the periphery. c. There are concerns about drug-food interactions with levodopa therapy that do not exist with the combination therapy. d. Carbidopa is the biologic precursor of dopamine and can penetrate into the central nervous system.

ANS: B When given in combination with levodopa, carbidopa inhibits the breakdown of levodopa in the periphery and thus allows smaller doses of levodopa to be used. Lesser amounts of levodopa result in fewer unwanted adverse effects. Levodopa, not carbidopa, is the biologic precursor of dopamine and can penetrate into the CNS. DIF: COGNITIVE LEVEL: Applying (Application) REF: p. 243 TOP: NURSING PROCESS: Implementation

5. The nurse is monitoring a patient who has been taking carbamazepine (Tegretol) for 2 months. Which effects would indicate that autoinduction has started to occur? a. The drug levels for carbamazepine are higher than expected. b. The drug levels for carbamazepine are lower than expected. c. The patient is experiencing fewer seizures. d. The patient is experiencing toxic effects from the drug.

ANS: B With carbamazepine, autoinduction occurs and leads to lower than expected drug concentrations. Therefore, the dosage may have to be adjusted with time. The other options are incorrect. DIF: COGNITIVE LEVEL: Understanding (Comprehension) REF: p. 226 TOP: NURSING PROCESS: Evaluation

1. The nurse can prevent medication errors by following which principles? (Select all that apply.) a. Assess for allergies after giving medications. b. Use two patient identifiers before giving medications. c. Do not give a medication that another nurse has drawn up in a syringe. d. Minimize the use of verbal and telephone orders. e. Use trade names instead of generic names to avoid confusion.

ANS: B, C, D Measures that prevent medication errors include using two patient identifiers, giving only medications that you have drawn up or prepared, and minimizing the use of verbal and telephone orders. Assessment for allergies should be done before medications are given. Generic names should be used to avoid the many sound-alike trade names of medications. DIF: COGNITIVE LEVEL: Applying (Application) REF: p. 69 TOP: NURSING PROCESS: Implementation

2. A patient has a new prescription for phentermine (Ionamin) as part of the treatment for weight loss. Which information will the nurse include when teaching this patient about a stimulant such as phentermine? (Select all that apply.) a. Take this medication after meals. b. Take this medication in the morning. c. This drug is taken along with supervised exercise and suitable diet. d. Use mouth rinses, sugarless gum, or hard candies to minimize dry mouth. e. Avoid foods that contain caffeine, such as coffee, tea, and colas.

ANS: B, C, D, E This drug should be taken in the morning to avoid interference with sleep, and the patient should also be on a supervised exercise and dietary regime. Caffeine-containing products should be avoided because of possible additional stimulation. Dry mouth can be minimized by the use of mouth rinses, sugarless gum, or hard candy. The other option is incorrect. DIF: COGNITIVE LEVEL: Applying (Application) REF: p. 214 TOP: NURSING PROCESS: Implementation

1. The nurse is reviewing antiepileptic drug (AED) therapy. Which statements about AED therapy are accurate? (Select all that apply.) a. AED therapy can be stopped when seizures are stopped. b. AED therapy is usually lifelong. c. Consistent dosing is the key to controlling seizures. d. A dose may be skipped if the patient is experiencing adverse effects. e. Do not abruptly discontinue AEDs because doing so may cause rebound seizure activity.

ANS: B, C, E Patients need to know that AED therapy is usually lifelong, and compliance (with consistent dosing) is important for effective seizure control. Abruptly stopping AED therapy may cause withdrawal (or rebound) seizure activity. DIF: COGNITIVE LEVEL: Applying (Application) REF: p. 232 TOP: NURSING PROCESS: Implementation

1. Vicodin (acetaminophen/hydrocodone) is prescribed for a patient who has had surgery. The nurse informs the patient that which common adverse effects can occur with this medication? (Select all that apply.) a. Diarrhea b. Constipation c. Lightheadedness d. Nervousness e. Urinary retention f. Itching

ANS: B, C, E, F Constipation (not diarrhea), lightheadedness (not nervousness), urinary retention, and itching are some of the common adverse effects that the patient may experience while taking Vicodin. DIF: COGNITIVE LEVEL: Understanding (Comprehension) REF: p. 152 TOP: NURSING PROCESS: Implementation

Vicodin (acetaminophen/hydrocodone) is prescribed for a patient who has had surgery. The nurse informs the patient that which common adverse effects can occur with this medication? (Select all that apply.) a. Diarrhea b. Constipation c. Lightheadedness d. Nervousness e. Urinary retention f. Itching

ANS: B, C, E, F Constipation (not diarrhea), lightheadedness (not nervousness), urinary retention, and itching are some of the common adverse effects that the patient may experience while taking Vicodin. DIF: COGNITIVE LEVEL: Understanding (Comprehension) REF: p. 152 TOP: NURSING PROCESS: Implementation MSC: NCLEX: Physiological Integrity: Pharmacological Therapies

1. A nurse is providing teaching for a patient who will be taking varenicline (Chantix) as part of a smoking-cessation program. Which teaching points are appropriate for a patient taking this medication? (Select all that apply.) a. This drug is available as a chewing gum that can be taken to reduce cravings. b. Use caution when driving because drowsiness may be a problem. c. There have been very few adverse effects reported for this drug. d. Notify the prescriber immediately if feelings of sadness or thoughts of suicide occur. e. Avoid caffeine while on this drug.

ANS: B, D Patients taking varenicline have reported drowsiness, which has prompted the U.S. Food and Drug Administration (FDA) to recommend caution when driving and engaging in other potentially hazardous activities until the patient can determine how the drug affects his or her mental status. In addition, the FDA has warned about psychiatric symptoms including agitation, depression, and suicidality. Varenicline is an oral tablet, and common adverse effects include nausea, vomiting, headache, and insomnia. There are no cautions about taking caffeine while on this drug. DIF: COGNITIVE LEVEL: Applying (Application) REF: p. 285 TOP: NURSING PROCESS: Implementation

A nurse is providing education to a group of patients regarding amphetamines. To evaluate the group's understanding, the nurse asks a participant what effects amphetamines would have on her. The participant shows that she understands the effects of these drugs if she gives which of the following answers? (Select all that apply.) a." Amphetamines increase fatigue." b. "Amphetamines suppress the perception of pain." c. "Amphetamines increase appetite." d. "Amphetamines increase the heart rate." e. "Amphetamines elevate mood."

ANS: B, D, E At customary doses, amphetamines increase wakefulness and alertness, reduce fatigue, elevate mood, and augment self-confidence and initiative. Amphetamines also suppress appetite and the perception of pain and increase the heart rate. Amphetamines do not increase fatigue or appetite.

1. The nurse is preparing to administer a barbiturate. Which conditions or disorders would be a contraindication to the use of these drugs? (Select all that apply.) a. Gout b. Pregnancy c. Epilepsy d. Severe chronic obstructive pulmonary disease e. Severe liver disease f. Diabetes mellitus

ANS: B, D, E Contraindications to barbiturates include pregnancy, significant respiratory difficulties, and severe liver disease. The other disorders are not contraindications. DIF: COGNITIVE LEVEL: Applying (Application) REF: p. 194 TOP: NURSING PROCESS: Assessment

1. The nurse is performing an assessment of a patient. Which assessment findings may indicate a higher risk for genetic disorders? (Select all that apply.) a. The patient's father was diagnosed with heart disease at 60 years of age. b. The patient's mother was diagnosed with breast cancer at 33 years of age. c. The patient's grandfather died of a cerebral vascular accident at 78 years of age. d. The patient's sister has a history of both renal and lung cancer. e. The patient has two uncles and a grandparent who have been diagnosed with Alzheimer's disease.

ANS: B, D, E The nurse should assess for factors that may indicate a risk for genetic disorders. A few examples of factors that may indicate a risk for genetic disorders are a higher incidence of a particular disease or disorder in the patient's family than in the general population; diagnosis of a disease in family members at an unusually young age; or diagnosis of a family member with an unusual form of cancer or with more than one type of cancer. The options regarding heart disease at 60 years of age and cerebral vascular accident at 78 years of age are not factors that indicate a higher risk for genetic disorders. DIF: COGNITIVE LEVEL: Applying (Application) REF: p. 100 TOP: NURSING PROCESS: Assessment

2. A patient has been taking disulfiram (Antabuse) as part of his rehabilitation therapy. However, this evening, he attended a party and drank half a beer. As a result, he became ill and his friends took him to the emergency department. The nurse will look for which adverse effects associated with acetaldehyde syndrome? (Select all that apply.) a. Euphoria b. Severe vomiting c. Diarrhea d. Pulsating headache e. Difficulty breathing f. Sweating

ANS: B, D, E, F Acetaldehyde syndrome results when alcohol is taken while on disulfiram (Antabuse) therapy. Adverse effects include CNS effects (pulsating headache, sweating, marked uneasiness, weakness, vertigo, others); GI effects (nausea, copious vomiting, thirst); and difficulty breathing. Cardiovascular effects also occur; see Table 17-2. Euphoria and diarrhea are not adverse effects associated with acetaldehyde syndrome. DIF: COGNITIVE LEVEL: Applying (Application) REF: p. 284 TOP: NURSING PROCESS: Implementation

6. The medication order reads, "Give ondansetron (Zofran) 4 mg, 30 minutes before beginning chemotherapy to prevent nausea." The nurse notes that the route is missing from the order. What is the nurse's best action? a. Give the medication intravenously because the patient might vomit. b. Give the medication orally because the tablets are available in 4-mg doses. c. Contact the prescriber to clarify the route of the medication ordered. d. Hold the medication until the prescriber returns to make rounds.

ANS: C A complete medication order includes the route of administration. If a medication order does not include the route, the nurse must ask the prescriber to clarify it. The intravenous and oral routes are not interchangeable. Holding the medication until the prescriber returns would mean that the patient would not receive a needed medication. DIF: COGNITIVE LEVEL: Applying (Application) REF: p. 12 TOP: NURSING PROCESS: Implementation

5. The nurse is reviewing pharmacology terms for a group of newly graduated nurses. Which sentence defines a drug's half-life? a. The time it takes for the drug to cause half of its therapeutic response b. The time it takes for one half of the original amount of a drug to reach the target cells c. The time it takes for one half of the original amount of a drug to be removed from the body d. The time it takes for one half of the original amount of a drug to be absorbed into the circulation

ANS: C A drug's half-life is the time it takes for one half of the original amount of a drug to be removed from the body. It is a measure of the rate at which drugs are removed from the body. The other options are incorrect definitions of half-life. DIF: COGNITIVE LEVEL: Understanding (Comprehension) REF: p. 27 TOP: NURSING PROCESS: General

1. The nurse is reviewing the applications of gene therapy. Which drug is manufactured as a result of indirect gene therapy? a. Vitamin K b. Warfarin c. Human insulin d. Heparin

ANS: C A recombinant form of human insulin is one of the most widespread uses of indirect gene therapy. Other examples include hormones, vaccines, antitoxins, and monoclonal antibodies. The other options listed are not examples of drugs manufactured by indirect gene therapy. DIF: COGNITIVE LEVEL: Remembering (Knowledge) REF: p. 98 TOP: NURSING PROCESS: General

6. When admitting a patient with a suspected diagnosis of chronic alcohol use, the nurse will keep in mind that chronic use of alcohol might result in which condition? a. Renal failure b. Cerebrovascular accident c. Korsakoff's psychosis d. Alzheimer's disease

ANS: C A variety of serious neurologic and mental disorders, such as Korsakoff's psychosis and Wernicke's encephalopathy, as well as cirrhosis of the liver, may occur with chronic use of alcohol. Renal failure, cerebrovascular accident, and Alzheimer's disease are not associated directly with chronic use of alcohol. DIF: COGNITIVE LEVEL: Understanding (Comprehension) REF: p. 283 TOP: NURSING PROCESS: Assessment

10. A patient is to receive acetylcysteine (Mucomyst) as part of the treatment for an acetaminophen (Tylenol) overdose. Which action by the nurse is appropriate when giving this medication? a. Giving the medication undiluted for full effect b. Avoiding the use of a straw when giving this medication c. Disguising the flavor with soda or flavored water d. Preparing to give this medication via a nebulizer

ANS: C Acetylcysteine has the flavor of rotten eggs and so is better tolerated if it is diluted and disguised by mixing with a drink such as cola or flavored water to help increase its palatability. The use of a straw helps to minimize contact with the mucous membranes of the mouth and is recommended. The nebulizer form of this medication is used for certain types of pneumonia, not for acetaminophen overdose. DIF: COGNITIVE LEVEL: Applying (Application) REF: p. 162 TOP: NURSING PROCESS: Implementation

A patient is to receive acetylcysteine (Mucomyst) as part of the treatment for an acetaminophen (Tylenol) overdose. Which action by the nurse is appropriate when giving this medication? a. Giving the medication undiluted for full effect b. Avoiding the use of a straw when giving this medication c. Disguising the flavor with soda or flavored water d. Preparing to give this medication via a nebulizer

ANS: C Acetylcysteine has the flavor of rotten eggs and so is better tolerated if it is diluted and disguised by mixing with a drink such as cola or flavored water to help increase its palatability. The use of a straw helps to minimize contact with the mucous membranes of the mouth and is recommended. The nebulizer form of this medication is used for certain types of pneumonia, not for acetaminophen overdose. DIF: COGNITIVE LEVEL: Applying (Application) REF: p. 162 TOP: NURSING PROCESS: Implementation MSC: NCLEX: Physiological Integrity: Pharmacological and Parenteral Therapies

13. The nurse is reviewing the food choices of a patient who is taking a monoamine oxidase inhibitor ( MAOI). Which food choice would indicate the need for additional teaching? a. Orange juice b. Fried eggs over-easy c. Salami and Swiss cheese sandwich d. Biscuits and honey

ANS: C Aged cheeses, such a Swiss or cheddar cheese, and Salami contain tyramine. Patients who are taking MAOIs need to avoid tyramine-containing foods because of a severe hypertensive reaction that may occur. Orange juice, eggs, biscuits, and honey do not contain tyramine. DIF: COGNITIVE LEVEL: Applying (Application) REF: p. 260 TOP: NURSING PROCESS: Implementation

2. When given an intravenous medication, the patient says to the nurse, "I usually take pills. Why does this medication have to be given in the arm?" What is the nurse's best answer? a. "The medication will cause fewer adverse effects when given intravenously." b. "The intravenous medication will have delayed absorption into the body's tissues." c. "The action of the medication will begin sooner when given intravenously." d. "There is a lower chance of allergic reactions when drugs are given intravenously."

ANS: C An intravenous (IV) injection provides the fastest route of absorption. The IV route does not affect the number of adverse effects, nor does it cause delayed tissue absorption (it results in faster absorption). The IV route does not affect the number of allergic reactions. DIF: COGNITIVE LEVEL: Understanding (Comprehension) REF: p. 22 TOP: NURSING PROCESS: Implementation

4. A patient has been taking temazepam (Restoril) for intermittent insomnia. She calls the nurse to say that when she takes it, she sleeps well, but the next day she feels "so tired." Which explanation by the nurse is correct? a. "Long-term use of this drug results in a sedative effect." b. "If you take the drug every night, this hangover effect will be reduced." c. "These drugs affect the sleep cycle, resulting in daytime sleepiness." d. "These drugs increase the activity of the central nervous system, making you tired the next day."

ANS: C Benzodiazepines suppress REM sleep to a degree (although not as much as barbiturates) and, thus, result in daytime sleepiness (a hangover effect). The other statements are incorrect. DIF: COGNITIVE LEVEL: Applying (Application) REF: p. 191 TOP: NURSING PROCESS: Implementation

16. A patient is suffering from tendonitis of the knee. The nurse is reviewing the patient's medication administration record and recognizes that which adjuvant medication is most appropriate for this type of pain? a. Antidepressant b. Anticonvulsant c. Corticosteroid d. Local anesthesia

ANS: C Corticosteroids have an anti-inflammatory effect, which may help to reduce pain. The other medications do not have anti-inflammatory properties. DIF: COGNITIVE LEVEL: Understanding (Comprehension) REF: p. 166 TOP: NURSING PROCESS: Planning

A patient is suffering from tendonitis of the knee. The nurse is reviewing the patient's medication administration record and recognizes that which adjuvant medication is most appropriate for this type of pain? a. Antidepressant b. Anticonvulsant c. Corticosteroid d. Local anesthesia

ANS: C Corticosteroids have an anti-inflammatory effect, which may help to reduce pain. The other medications do not have anti-inflammatory properties. DIF: COGNITIVE LEVEL: Understanding (Comprehension) REF: p. 166 TOP: NURSING PROCESS: Planning MSC: NCLEX: Physiological Integrity: Pharmacological and Parenteral Therapies

15. The nurse is reviewing medications used for depression. Which of these statements is a reason that selective serotonin reuptake inhibitors (SSRIs) are more widely prescribed today than tricyclic antidepressants? a. SSRIs have fewer sexual side effects. b. Unlike tricyclic antidepressants, SSRIs do not have drug-food interactions. c. Tricyclic antidepressants cause serious cardiac dysrhythmias if an overdose occurs. d. SSRIs cause a therapeutic response faster than tricyclic antidepressants.

ANS: C Death from overdose of tricyclic antidepressants usually results from either seizures or dysrhythmias. SSRIs are associated with significantly fewer and less severe systemic adverse effects, especially anticholinergic and cardiovascular adverse effects. The other options are incorrect. DIF: COGNITIVE LEVEL: Applying (Application) REF: p. 257 TOP: NURSING PROCESS: Planning

2. The nurse is reviewing a list of verbal medication orders. Which is the proper notation of the dose of the drug ordered? a. Digoxin .125 mg b. Digoxin .1250 mg c. Digoxin 0.125 mg d. Digoxin 0.1250 mg

ANS: C Digoxin 0.125 mg illustrates the correct notation with a leading zero before the decimal point. Omitting the leading zero may cause the order to be misread, resulting in a large drug overdose. Digoxin .125 mg and digoxin .1250 mg do not have the leading zero before the decimal point. Digoxin 0.1250 mg has a trailing zero, which also is incorrect. DIF: COGNITIVE LEVEL: Applying (Application) REF: p. 69 TOP: NURSING PROCESS: Assessment

2. The nurse is discussing gene therapy in a continuing education class. Which is the best definition of eugenics? a. The use of gene therapy to prevent disease b. The development of new drugs based on gene therapy c. Intentional selection, before birth, of genotypes that are considered more desirable than others d. The determination of genetic factors that influence a person's response to medications

ANS: C Eugenics is the intentional selection of genotypes, before birth, that are considered more desirable than others, and it is a major ethical issue concerning gene therapy. The other options do not describe eugenics. DIF: COGNITIVE LEVEL: Understanding (Comprehension) REF: p. 99 TOP: NURSING PROCESS: General

4. A patient has been taking haloperidol (Haldol) for 3 months for a psychotic disorder, and the nurse is concerned about the development of extrapyramidal symptoms. The nurse will monitor the patient closely for which effects? a. Increased paranoia b. Drowsiness and dizziness c. Tremors and muscle twitching d. Dry mouth and constipation

ANS: C Extrapyramidal symptoms are manifested by tremors and muscle twitching, and the incidence of such symptoms is high during haloperidol therapy. The other options are incorrect. DIF: COGNITIVE LEVEL: Understanding (Comprehension) REF: p. 264 TOP: NURSING PROCESS: Assessment

5. When taking a telephone order for a medication, which action by the nurse is most appropriate? a. Verify the order with the charge nurse. b. Call back the prescriber to review the order. c. Repeat the order to the prescriber before hanging up the telephone. d. Ask the pharmacist to double-check the order.

ANS: C For telephone or verbal orders, repeat the order back to the prescriber before hanging up the telephone. The other options are incorrect. DIF: COGNITIVE LEVEL: Applying (Application) REF: p. 69 TOP: NURSING PROCESS: Implementation

5. A patient will be discharged with a 1-week supply of an opioid analgesic for pain management after abdominal surgery. The nurse will include which information in the teaching plan? a. How to prevent dehydration due to diarrhea b. The importance of taking the drug only when the pain becomes severe c. How to prevent constipation d. The importance of taking the drug on an empty stomach

ANS: C Gastrointestinal (GI) adverse effects, such as nausea, vomiting, and constipation, are the most common adverse effects associated with opioid analgesics. Physical dependence usually occurs in patients undergoing long-term treatment. Diarrhea is not an effect of opioid analgesics. Taking the dose with food may help minimize GI upset. DIF: COGNITIVE LEVEL: Applying (Application) REF: p. 149 TOP: NURSING PROCESS: Implementation

A patient will be discharged with a 1-week supply of an opioid analgesic for pain management after abdominal surgery. The nurse will include which information in the teaching plan? a. How to prevent dehydration due to diarrhea b. The importance of taking the drug only when the pain becomes severe c. How to prevent constipation d. The importance of taking the drug on an empty stomach

ANS: C Gastrointestinal (GI) adverse effects, such as nausea, vomiting, and constipation, are the most common adverse effects associated with opioid analgesics. Physical dependence usually occurs in patients undergoing long-term treatment. Diarrhea is not an effect of opioid analgesics. Taking the dose with food may help minimize GI upset. DIF: COGNITIVE LEVEL: Applying (Application) REF: p. 149 TOP: NURSING PROCESS: Implementation MSC: NCLEX: Physiological Integrity: Reduction of Risk Potential

7. A 38-year-old man has come into the urgent care center with severe hip pain after falling from a ladder at work. He says he has taken several pain pills over the past few hours but cannot remember how many he has taken. He hands the nurse an empty bottle of acetaminophen (Tylenol). The nurse is aware that the most serious toxic effect of acute acetaminophen overdose is which condition? a. Tachycardia b. Central nervous system depression c. Hepatic necrosis d. Nephropathy

ANS: C Hepatic necrosis is the most serious acute toxic effect of an acute overdose of acetaminophen. The other options are incorrect. DIF: COGNITIVE LEVEL: Understanding (Comprehension) REF: p. 158 TOP: NURSING PROCESS: Assessment

A 38-year-old man has come into the urgent care center with severe hip pain after falling from a ladder at work. He says he has taken several pain pills over the past few hours but cannot remember how many he has taken. He hands the nurse an empty bottle of acetaminophen (Tylenol). The nurse is aware that the most serious toxic effect of acute acetaminophen overdose is which condition? a. Tachycardia b. Central nervous system depression c. Hepatic necrosis d. Nephropathy

ANS: C Hepatic necrosis is the most serious acute toxic effect of an acute overdose of acetaminophen. The other options are incorrect. DIF: COGNITIVE LEVEL: Understanding (Comprehension) REF: p. 158 TOP: NURSING PROCESS: Assessment MSC: NCLEX: Safe and Effective Care Environment: Safety and Infection Control

4. A patient has a 9-year history of a seizure disorder that has been managed well with oral phenytoin (Dilantin) therapy. He is to be NPO (consume nothing by mouth) for surgery in the morning. What will the nurse do about his morning dose of phenytoin? a. Give the same dose intravenously. b. Give the morning dose with a small sip of water. c. Contact the prescriber for another dosage form of the medication. d. Notify the operating room that the medication has been withheld.

ANS: C If there are any questions about the medication order or the medication prescribed, contact the prescriber immediately for clarification and for an order of the appropriate dose form of the medication. Do not change the route without the prescriber's order. There is an increased risk of seizure activity if one or more doses of the AED are missed. DIF: COGNITIVE LEVEL: Applying (Application) REF: p. 231 TOP: NURSING PROCESS: Implementation

14. The nurse is assessing a patient for contraindications to drug therapy with acetaminophen (Tylenol). Which patient should not receive acetaminophen? a. A patient with a fever of 101° F (38.3° C) b. A patient who is complaining of a mild headache c. A patient with a history of liver disease d. A patient with a history of peptic ulcer disease

ANS: C Liver disease is a contraindication to the use of acetaminophen. Fever and mild headache are both possible indications for the medication. Having a history of peptic ulcer disease is not a contraindication. DIF: COGNITIVE LEVEL: Applying (Application) REF: p. 157 TOP: NURSING PROCESS: Assessment

The nurse is assessing a patient for contraindications to drug therapy with acetaminophen (Tylenol). Which patient should not receive acetaminophen? a. A patient with a fever of 101° F (38.3° C) b. A patient who is complaining of a mild headache c. A patient with a history of liver disease d. A patient with a history of peptic ulcer disease

ANS: C Liver disease is a contraindication to the use of acetaminophen. Fever and mild headache are both possible indications for the medication. Having a history of peptic ulcer disease is not a contraindication. DIF: COGNITIVE LEVEL: Applying (Application) REF: p. 157 TOP: NURSING PROCESS: Assessment MSC: NCLEX: Physiological Integrity: Pharmacological and Parenteral Therapies

4. A 78-year-old patient is in the recovery room after having a lengthy surgery on his hip. As he is gradually awakening, he requests pain medication. Within 10 minutes after receiving a dose of morphine sulfate, he is very lethargic and his respirations are shallow, with a rate of 7 per minute. The nurse prepares for which priority action at this time? a. Assessment of the patient's pain level b. Immediate intubation and artificial ventilation c. Administration of naloxone (Narcan) d. Close observation of signs of opioid tolerance

ANS: C Naloxone, an opioid-reversal agent, is used to reverse the effects of acute opioid overdose and is the drug of choice for reversal of opioid-induced respiratory depression. This situation is describing an opioid overdose, not opioid tolerance. Intubation and artificial ventilation are not appropriate because the patient is still breathing at 7 breaths/min. It would be inappropriate to assess the patient's level of pain. DIF: COGNITIVE LEVEL: Applying (Application) REF: p. 153 TOP: NURSING PROCESS: Implementation

A 78-year-old patient is in the recovery room after having a lengthy surgery on his hip. As he is gradually awakening, he requests pain medication. Within 10 minutes after receiving a dose of morphine sulfate, he is very lethargic and his respirations are shallow, with a rate of 7 per minute. The nurse prepares for which priority action at this time? a. Assessment of the patient's pain level b. Immediate intubation and artificial ventilation c. Administration of naloxone (Narcan) d. Close observation of signs of opioid tolerance

ANS: C Naloxone, an opioid-reversal agent, is used to reverse the effects of acute opioid overdose and is the drug of choice for reversal of opioid-induced respiratory depression. This situation is describing an opioid overdose, not opioid tolerance. Intubation and artificial ventilation are not appropriate because the patient is still breathing at 7 breaths/min. It would be inappropriate to assess the patient's level of pain. DIF: COGNITIVE LEVEL: Applying (Application) REF: p. 153 TOP: NURSING PROCESS: Implementation MSC: NCLEX: Physiological Integrity: Pharmacological and Parenteral Therapies

6. When evaluating a patient who is taking orlistat (Xenical), which is an intended therapeutic effect? a. Increased wakefulness b. Increased appetite c. Decreased weight d. Decreased hyperactivity

ANS: C Orlistat (Xenical) is a nonstimulant drug that is used as part of a weight loss program. The other options are incorrect. DIF: COGNITIVE LEVEL: Understanding (Comprehension) REF: p. 208 TOP: NURSING PROCESS: Evaluation

8. Ramelteon (Rozerem) is prescribed for a patient with insomnia. The nurse checks the patient's medical history, knowing that this medication is contraindicated in which disorder? a. Coronary artery disease b. Renal insufficiency c. Liver disease d. Anemia

ANS: C Ramelteon is contraindicated in cases of severe liver dysfunction. The other conditions are not contraindications. DIF: COGNITIVE LEVEL: Understanding (Comprehension) REF: p. 193 TOP: NURSING PROCESS: Assessment

8. The nurse is performing an assessment of a newly admitted patient. Which is an example of subjective data? a. Blood pressure 158/96 mm Hg b. Weight 255 pounds c. The patient reports that he uses the herbal product ginkgo. d. The patient's laboratory work includes a complete blood count and urinalysis.

ANS: C Subjective data include information shared through the spoken word by any reliable source, such as the patient. Objective data may be defined as any information gathered through the senses or that which is seen, heard, felt, or smelled. A patient's blood pressure, weight, and laboratory tests are all examples of objective data. DIF: COGNITIVE LEVEL: Understanding (Comprehension) REF: p. 6 TOP: NURSING PROCESS: Assessment

5. A 29-year-old male patient is admitted to the intensive care unit with the following symptoms: restlessness, hyperactive reflexes, talkativeness, confusion and periods of panic and euphoria, tachycardia, and fever. The nurse suspects that he may be experiencing the effects of taking which substance? a. Opioids b. Alcohol c. Stimulants d. Depressants

ANS: C The adverse effects listed may occur with use of stimulants and are commonly an extension of their therapeutic effects. Opioids, alcohol, and depressants do not have these effects. DIF: COGNITIVE LEVEL: Applying (Application) REF: p. 280 TOP: NURSING PROCESS: Assessment

4. A 22-year-old nursing student has been taking NoDoz (caffeine) tablets for the past few weeks to "make it through" the end of the semester and exam week. She is in the university clinic today because she is "exhausted." What nursing diagnosis may be appropriate for her? a. Noncompliance b. Impaired physical mobility c. Disturbed sleep pattern d. Imbalanced nutrition: less than body requirements

ANS: C The main ingredient in NoDoz, caffeine, is a central nervous system stimulant that can be used to increase mental alertness. Restlessness, anxiety, and insomnia are common adverse effects. Thus, disturbed sleep pattern is the most appropriate nursing diagnosis of those listed. DIF: COGNITIVE LEVEL: Applying (Application) REF: p. 213 TOP: NURSING PROCESS: Nursing Diagnosis

12. The nurse is assessing a patient who has been admitted to the emergency department for a possible opioid overdose. Which assessment finding is characteristic of an opioid drug overdose? a. Dilated pupils b. Restlessness c. Respiration rate of 6 breaths/min d. Heart rate of 55 beats/min

ANS: C The most serious adverse effect of opioid use is CNS depression, which may lead to respiratory depression. Pinpoint pupils, not dilated pupils, are seen. Restlessness and a heart rate of 55 beats/min are not indications of an opioid overdose. DIF: COGNITIVE LEVEL: Applying (Application) REF: p. 152 TOP: NURSING PROCESS: Assessment

The nurse is assessing a patient who has been admitted to the emergency department for a possible opioid overdose. Which assessment finding is characteristic of an opioid drug overdose? a. Dilated pupils b. Restlessness c. Respiration rate of 6 breaths/min d. Heart rate of 55 beats/min

ANS: C The most serious adverse effect of opioid use is CNS depression, which may lead to respiratory depression. Pinpoint pupils, not dilated pupils, are seen. Restlessness and a heart rate of 55 beats/min are not indications of an opioid overdose. DIF: COGNITIVE LEVEL: Applying (Application) REF: p. 152 TOP: NURSING PROCESS: Assessment MSC: NCLEX: Physiological Integrity: Pharmacological and Parenteral Therapies

7. When the nurse considers the timing of a drug dose, which factor is appropriate to consider when deciding when to give a drug? a. The patient's ability to swallow b. The patient's height c. The patient's last meal d. The patient's allergies

ANS: C The nurse must consider specific pharmacokinetic/pharmacodynamic drug properties that may be affected by the timing of the last meal. The patient's ability to swallow, height, and allergies are not factors to consider regarding the timing of the drug's administration. DIF: COGNITIVE LEVEL: Understanding (Comprehension) REF: p. 12 TOP: NURSING PROCESS: Assessment

7. During an admission assessment, the nurse discovers that the patient does not speak English. Which is considered the ideal resource for translation? a. A family member of the patient b. A close family friend of the patient c. A translator who does not know the patient d. Prewritten note cards with both English and the patient's language

ANS: C The nurse should communicate with the patient in the patient's native language if at all possible. If the nurse is not able to speak the patient's native language, a translator should be made available so as to prevent communication problems, minimize errors, and help boost the patient's level of trust and understanding of the nurse. In practice, this translator may be another nurse or health care professional, a nonprofessional member of the health care team, or a layperson, family member, adult friend, or religious leader or associate. However, it is best to avoid family members as translators, if possible, because of issues with bias, misinterpretation, and potential confidentiality issues. DIF: COGNITIVE LEVEL: Applying (Application) REF: p. 78 TOP: NURSING PROCESS: Implementation

6. A patient calls the clinic to ask about taking cranberry dietary supplement capsules because a friend recommended them. The nurse will discuss which possible concern when a patient is taking cranberry supplements? a. It may increase the risk for bleeding if the patient is taking anticoagulants. b. It may increase the risk of toxicity of some psychotherapeutic drugs. c. It may reduce elimination of drugs that are excreted by the kidneys. d. Cranberry may increase the intensity and duration of effects of caffeine.

ANS: C The use of cranberry decreases the elimination of many drugs that are renally excreted. The other concerns do not occur with cranberry supplements. DIF: COGNITIVE LEVEL: Applying (Application) REF: p. 90 TOP: NURSING PROCESS: Planning

11. When treating patients with medications for Parkinson's disease, the nurse knows that the wearing-off phenomenon occurs for which reason? a. There are rapid swings in the patient's response to levodopa. b. The patient cannot tolerate the medications at times. c. The medications begin to lose effectiveness against Parkinson's disease. d. The patient's liver is no longer able to metabolize the drug.

ANS: C The wearing-off phenomenon occurs when antiparkinson medications begin to lose their effectiveness, despite maximal dosing, as the disease progresses. The other options are incorrect. DIF: COGNITIVE LEVEL: Understanding (Comprehension) REF: p. 237 TOP: NURSING PROCESS: Assessment

2. The nurse is developing a care plan for a patient who will be self-administering insulin injections. Which statement reflects a measurable outcome? a. The patient will know about self-administration of insulin injections. b. The patient will understand the principles of self-administration of insulin injections. c. The patient will demonstrate the proper technique of self-administering insulin injections. d. The patient will comprehend the proper technique of self-administering insulin injections.

ANS: C The word demonstrate is a measurable verb, and measurable terms should be used when developing goals and outcome criteria statements. The other options are incorrect because the terms know, understand, and comprehend are not measurable terms. DIF: COGNITIVE LEVEL: Applying (Application) REF: p. 77 TOP: NURSING PROCESS: Planning

7. A patient is brought to the emergency department for treatment of a suspected overdose. The patient was found with an empty prescription bottle of a barbiturate by his bedside. He is lethargic and barely breathing. The nurse would expect which immediate intervention? a. Starting an intravenous infusion of diluted bicarbonate solution b. Administering medications to increase blood pressure c. Implementing measures to maintain the airway and support respirations d. Administrating naloxone (Narcan) as an antagonist

ANS: C There are no antagonists/antidotes for barbiturates. Treatment supports respirations and maintains the airway. The other interventions are not appropriate. DIF: COGNITIVE LEVEL: Applying (Application) REF: p. 195 TOP: NURSING PROCESS: Implementation

6. The nurse is assessing the medication history of a patient with a new diagnosis of Parkinson's disease. Which condition is a contraindication for the patient, who will be taking tolcapone (Tasmar)? a. Glaucoma b. Seizure disorder c. Liver failure d. Benign prostatic hyperplasia

ANS: C Tolcapone is contraindicated in patients who have shown a hypersensitivity reaction to it, and it should be used with caution in patients with pre-existing liver disease. The other conditions listed are not contraindications. DIF: COGNITIVE LEVEL: Understanding (Comprehension) REF: p. 241 TOP: NURSING PROCESS: Assessment

10. A patient has been admitted to the emergency department with a suspected overdose of a tricyclic antidepressant. The nurse will prepare for what immediate concern? a. Hypertension b. Renal failure c. Cardiac dysrhythmias d. Gastrointestinal bleeding

ANS: C Tricyclic antidepressant overdoses are notoriously lethal. The primary organ systems affected are the central nervous system and the cardiovascular system, and death usually results from either seizures or dysrhythmias. DIF: COGNITIVE LEVEL: Applying (Application) REF: p. 256 TOP: NURSING PROCESS: Planning

3. The patient wants to take the herb valerian to help him rest at night. The nurse would be concerned about potential interactions if he is taking a medication from which class of drugs? a. Digitalis b. Anticoagulants c. Sedatives d. Immunosuppressants

ANS: C Valerian may cause increased central nervous system depression if used with sedatives. Digitalis, anticoagulants, and immunosuppressants do not have interactions with valerian. DIF: COGNITIVE LEVEL: Understanding (Comprehension) REF: p. 90 TOP: NURSING PROCESS: Assessment

2. Which statements are true regarding the selective serotonin reuptake inhibitors (SSRIs)? (Select all that apply.) a. Avoid foods and beverages that contain tyramine. b. Monitor the patient for extrapyramidal symptoms. c. Therapeutic effects may not be seen for about 4 to 6 weeks after the medication is started. d. If the patient has been on an MAOI, a 2- to 5-week or longer time span is required before beginning an SSRI medication. e. These drugs have anticholinergic effects, including constipation, urinary retention, dry mouth, and blurred vision. f. Cogentin is often also prescribed to reduce the adverse effects that may occur.

ANS: C, D During SSRI medication, therapeutic effects may not be seen for 4 to 6 weeks. To prevent the potentially fatal pharmacodynamic interactions that can occur between the SSRIs and the MAOIs, a 2- to 5-week washout period is recommended between uses of these two classes of medications. The other options apply to other classes of psychotherapeutic drugs, not SSRIs. DIF: COGNITIVE LEVEL: Applying (Application) REF: p. 261 TOP: NURSING PROCESS: Planning

1. Which drugs would be affected by the first-pass effect? (Select all that apply.) a. Morphine given by IV push injection b. Sublingual nitroglycerin tablets c. Diphenhydramine (Benadryl) elixir d. Levothyroxine (Synthroid) tablets e. Transdermal nicotine patches f. Esomeprazole (Nexium) capsules g. Penicillin given by IV piggyback infusion

ANS: C, D, F Orally administered drugs (elixirs, tablets, capsules) undergo the first-pass effect because they are metabolized in the liver after being absorbed into the portal circulation from the small intestine. IV medications (IV push and IV piggyback) enter the bloodstream directly and do not go directly to the liver. Sublingual tablets and transdermal patches also enter the bloodstream without going directly to the liver, thus avoiding the first-pass effect. DIF: COGNITIVE LEVEL: Applying (Application) REF: p. 24 TOP: NURSING PROCESS: General

4. The nurse is assigned to a patient who is newly diagnosed with type 1 diabetes mellitus. Which statement best illustrates an outcome criterion for this patient? a. The patient will follow instructions. b. The patient will not experience complications. c. The patient will adhere to the new insulin treatment regimen. d. The patient will demonstrate correct blood glucose testing technique.

ANS: D "Demonstrating correct blood glucose testing technique" is a specific and measurable outcome criterion. "Following instructions" and "not experiencing complications" are not specific criteria. "Adhering to new regimen" would be difficult to measure. DIF: COGNITIVE LEVEL: Applying (Application) REF: p. 8 TOP: NURSING PROCESS: Planning

1. Which nursing diagnosis is appropriate for the patient who has just received a prescription for a new medication? a. Noncompliance related to new drug therapy b. Impaired memory related to new drug therapy c. Lack of knowledge regarding newly prescribed drug therapy d. Deficient knowledge related to newly prescribed drug therapy

ANS: D A patient who has a limited understanding of newly prescribed drug therapy may have the nursing diagnosis of deficient knowledge. Noncompliance is incorrect because that term implies that the patient does not follow a recommended regimen, which is not the case with a newly prescribed drug. Impaired memory is not appropriate in this situation. "Lack of knowledge" is not a nursing diagnosis. DIF: COGNITIVE LEVEL: Applying (Application) REF: p. 77 TOP: NURSING PROCESS: Nursing Diagnosis

4. When monitoring the patient receiving an intravenous infusion to reduce blood pressure, the nurse notes that the patient's blood pressure is extremely low, and the patient is lethargic and difficult to awaken. This would be classified as which type of adverse drug reaction? a. Adverse effect b. Allergic reaction c. Idiosyncratic reaction d. Pharmacologic reaction

ANS: D A pharmacologic reaction is an extension of a drug's normal effects in the body. In this case, the antihypertensive drug lowered the patient's blood pressure levels too much. The other options do not describe a pharmacologic reaction. An adverse effect is a predictable, well-known adverse drug reaction that results in minor or no changes in patient management. An allergic reaction (also known as a hypersensitivity reaction) involves the patient's immune system. An idiosyncratic reaction is unexpected and is defined as a genetically determined abnormal response to normal dosages of a drug. DIF: COGNITIVE LEVEL: Understanding (Comprehension) REF: p. 32 TOP: NURSING PROCESS: General

2. While monitoring a patient who had surgery under general anesthesia 2 hours ago, the nurse notes a sudden elevation in body temperature. This finding may be an indication of which problem? a. Tachyphylaxis b. Postoperative infection c. Malignant hypertension d. Malignant hyperthermia

ANS: D A sudden elevation in body temperature during the postoperative period may indicate the occurrence of malignant hyperthermia, a life-threatening emergency. The elevated temperature does not reflect the other problems listed. DIF: COGNITIVE LEVEL: Applying (Application) REF: p. 174 TOP: NURSING PROCESS: Assessment

8. The nurse is teaching a 16-year-old patient who has a new diagnosis of type 1 diabetes about blood glucose monitoring and the importance of regulating glucose intake. When developing a teaching plan for this teenager, which of Erikson's stages of development should the nurse consider? a. Trust versus mistrust b. Intimacy versus isolation c. Industry versus inferiority d. Identity versus role confusion

ANS: D According to Erikson, the adolescent (12 to 18 years of age) is in the identity versus role confusion stage of development. Trust versus mistrust reflects the infancy stage; intimacy versus isolation reflects the young adulthood stage; and industry versus inferiority reflects the school-age stage of development. DIF: COGNITIVE LEVEL: Understanding (Comprehension) REF: p. 76 TOP: NURSING PROCESS: Assessment

15. A patient arrives at the urgent care center complaining of leg pain after a fall when rock climbing. The x-rays show no broken bones, but he has a large bruise on his thigh. The patient says he drives a truck and does not want to take anything strong because he needs to stay awake. Which statement by the nurse is most appropriate? a. "It would be best for you not to take anything if you are planning to drive your truck." b. "We will discuss with your doctor about taking an opioid because that would work best for your pain." c. "You can take acetaminophen, also known as Tylenol, for pain, but no more than 1000 mg per day." d. "You can take acetaminophen, also known as Tylenol, for pain, but no more than 3000 mg per day."

ANS: D Acetaminophen is indicated for mild-to-moderate pain and does not cause drowsiness, as an opioid would. Currently, the maximum daily amount of acetaminophen is 3000 mg/day. The 1000-mg amount per day is too low. Telling the patient not to take any pain medications is incorrect. DIF: COGNITIVE LEVEL: Applying (Application) REF: p. 157 TOP: NURSING PROCESS: Planning

A patient arrives at the urgent care center complaining of leg pain after a fall when rock climbing. The x-rays show no broken bones, but he has a large bruise on his thigh. The patient says he drives a truck and does not want to take anything strong because he needs to stay awake. Which statement by the nurse is most appropriate? a. "It would be best for you not to take anything if you are planning to drive your truck." b. "We will discuss with your doctor about taking an opioid because that would work best for your pain." c. "You can take acetaminophen, also known as Tylenol, for pain, but no more than 1000 mg per day." d. "You can take acetaminophen, also known as Tylenol, for pain, but no more than 3000 mg per day."

ANS: D Acetaminophen is indicated for mild-to-moderate pain and does not cause drowsiness, as an opioid would. Currently, the maximum daily amount of acetaminophen is 3000 mg/day. The 1000-mg amount per day is too low. Telling the patient not to take any pain medications is incorrect. DIF: COGNITIVE LEVEL: Applying (Application) REF: p. 157 TOP: NURSING PROCESS: Planning MSC: NCLEX: Physiological Integrity: Pharmacological and Parenteral Therapies

2. A patient has been taking phenobarbital for 2 weeks as part of his therapy for epilepsy. He tells the nurse that he feels tense and that "the least little thing" bothers him now. Which is the correct explanation for this problem? a. These are adverse effects that usually subside after a few weeks. b. The drug must be stopped immediately because of possible adverse effects. c. This drug causes the rapid eye movement (REM) sleep period to increase, resulting in nightmares and restlessness. d. This drug causes deprivation of REM sleep and may cause the inability to deal with normal stress.

ANS: D Barbiturates such as phenobarbital deprive people of REM sleep, which can result in agitation and the inability to deal with normal stress. A rebound phenomenon occurs when the drug is stopped (not during therapy), and the proportion of REM sleep increases, sometimes resulting in nightmares. The other options are incorrect. DIF: COGNITIVE LEVEL: Understanding (Comprehension) REF: p. 194 TOP: NURSING PROCESS: Evaluation

8. A patient has a new order for a catechol ortho-methyltransferase (COMT) inhibitor as part of treatment for Parkinson's disease. The nurse recognizes that which of these is an advantage of this drug class? a. It has a shorter duration of action. b. It causes less gastrointestinal distress. c. It has a slower onset than traditional Parkinson's disease drugs. d. It is associated with fewer wearing-off effects.

ANS: D COMT inhibitors are associated with fewer wearing-off effects and have prolonged therapeutic benefits. They have a quicker onset, and they prolong the duration of action of levodopa. DIF: COGNITIVE LEVEL: Understanding (Comprehension) REF: pp. 240-241 TOP: NURSING PROCESS: Implementation

9. A 60-year-old patient is on several new medications and expresses worry that she will forget to take her pills. Which action by the nurse would be most helpful in this situation? a. Teaching effective coping strategies b. Asking the patient's prescriber to reduce the number of drugs prescribed c. Assuring the patient that she will not forget once she is accustomed to the routine d. Assisting the patient with obtaining and learning to use a calendar or pill container

ANS: D Calendars, pill containers, or diaries may be helpful to patients who may forget to take prescribed drugs as scheduled. The nurse must ensure that the patient knows how to use these reminder tools. Teaching coping strategies is a helpful suggestion but will not help with remembering to take medications. Asking the prescriber to reduce the number of drugs that are prescribed is not an appropriate action by the nurse. Assuring the patient that she will not forget is false reassurance by the nurse and inappropriate when education is needed. DIF: COGNITIVE LEVEL: Applying (Application) REF: p. 78 TOP: NURSING PROCESS: Implementation

3. A 6-year-old boy has been started on an extended-release form of methylphenidate hydrochloride (Ritalin) for the treatment of attention deficit hyperactivity disorder (ADHD). During a follow-up visit, his mother tells the nurse that she has been giving the medication at bedtime so that it will be "in his system" when he goes to school the next morning. What is the nurse's appropriate evaluation of the mother's actions? a. She is giving him the medication dosage appropriately. b. The medication should not be taken until he is at school. c. The medication should be taken with meals for optimal absorption. d. The medication should be given 4 to 6 hours before bedtime to diminish insomnia.

ANS: D Central nervous system stimulants should be taken 4 to 6 hours before bedtime to decrease insomnia. Generally speaking, once-a-day dosing is used with extended-release or long-acting preparations. These formulations eliminate the need to take this medication at school. DIF: COGNITIVE LEVEL: Applying (Application) REF: p. 214 TOP: NURSING PROCESS: Evaluation

5. Carbidopa-levodopa (Sinemet) is prescribed for a patient with Parkinson's disease. The nurse informs the patient that which common adverse effects can occur with this medication? a. Drowsiness, headache, weight loss b. Dizziness, insomnia, nausea c. Peripheral edema, fatigue, syncope d. Heart palpitations, hypotension, urinary retention

ANS: D Common adverse reactions associated with carbidopa-levodopa include palpitations, hypotension, urinary retention, dyskinesia, and depression. The other effects may occur with other antiparkinson drugs. DIF: COGNITIVE LEVEL: Understanding (Comprehension) REF: p. 239 TOP: NURSING PROCESS: Implementation

3. When teaching a patient about taking a newly prescribed antiepileptic drug (AED) at home, the nurse will include which instruction? a. "Driving is allowed after 2 weeks of therapy." b. "If seizures recur, take a double dose of the medication." c. "Antacids can be taken with the AED to reduce gastrointestinal adverse effects." d. "Regular, consistent dosing is important for successful treatment."

ANS: D Consistent dosing, taken regularly at the same time of day, at the recommended dose, and with meals to reduce the common gastrointestinal adverse effects, is the key to successful management of seizures when taking AEDs. Noncompliance is the factor most likely to lead to treatment failure. DIF: COGNITIVE LEVEL: Applying (Application) REF: p. 231 TOP: NURSING PROCESS: Implementation

4. The nurse is setting up a teaching session with an 85-year-old patient who will be going home on anticoagulant therapy. Which educational strategy would reflect consideration of the age-related changes that may exist with this patient? a. Show a video about anticoagulation therapy. b. Present all the information in one session just before discharge. c. Give the patient pamphlets about the medications to read at home. d. Develop large-print handouts that reflect the verbal information presented.

ANS: D Developing large-print handouts addresses altered perception in two ways. First, by using visual aids to reinforce verbal instructions, one addresses the possibility of decreased ability to hear high-frequency sounds. By developing the handouts in large print, one addresses the possibility of decreased visual acuity. Showing a video does not allow discussion of the information; furthermore, the text and print may be small and difficult to read and understand. Presenting all the information in one session before discharge also does not allow for discussion, and the patient may not be able to hear or see the information sufficiently. Because of the possibility of decreased short-term memory and slowed cognitive function, giving pamphlets to read may not be appropriate. DIF: COGNITIVE LEVEL: Applying (Application) REF: p. 79 TOP: NURSING PROCESS: Implementation

8. When administering a new medication to a patient, the nurse reads that it is highly protein bound. Assuming that the patient's albumin levels are normal, the nurse would expect which result, as compared to a medication that is not highly protein bound? a. Renal excretion will be faster. b. The drug will be metabolized quickly. c. The duration of action of the medication will be shorter. d. The duration of action of the medication will be longer.

ANS: D Drugs that are bound to plasma proteins are characterized by longer duration of action. Protein binding does not make renal excretion faster, does not speed up drug metabolism, and does not cause the duration of action to be shorter. DIF: COGNITIVE LEVEL: Applying (Application) REF: pp. 24-25 TOP: NURSING PROCESS: Planning

3. When a patient is taking an anticholinergic such as benztropine (Cogentin) as part of the treatment for Parkinson's disease, the nurse should include which information in the teaching plan? a. Minimize the amount of fluid taken while on this drug. b. Discontinue the medication if adverse effects occur. c. Take the medication on an empty stomach to enhance absorption. d. Use artificial saliva, sugarless gum, or hard candy to counteract dry mouth.

ANS: D Dry mouth can be managed with artificial saliva through drops or gum, frequent mouth care, forced fluids, and sucking on sugar-free hard candy. Anticholinergics should be taken with or after meals to minimize GI upset and must not be discontinued suddenly. The patient must drink at least 3000 mL/day unless contraindicated. Drinking water is important, even if the patient is not thirsty or in need of hydration, to prevent and manage the adverse effect of constipation. DIF: COGNITIVE LEVEL: Applying (Application) REF: p. 246 TOP: NURSING PROCESS: Planning

6. When administering drugs, the nurse remembers that the duration of action of a drug is defined as which of these? a. The time it takes for a drug to elicit a therapeutic response b. The amount of time needed to remove a drug from circulation c. The time it takes for a drug to achieve its maximum therapeutic response d. The time period at which a drug's concentration is sufficient to cause a therapeutic response

ANS: D Duration of action is the time during which drug concentration is sufficient to elicit a therapeutic response. The other options do not define duration of action. A drug's onset of action is the time it takes for the drug to elicit a therapeutic response. A drug's peak effect is the time it takes for the drug to reach its maximum therapeutic response. Elimination is the length of time it takes to remove a drug from circulation. DIF: COGNITIVE LEVEL: Understanding (Comprehension) REF: p. 28 TOP: NURSING PROCESS: General

10. A patient has experienced insomnia for months, and the physician has prescribed a medication to help with this problem. The nurse expects which drug to be used for long-term treatment of insomnia? a. Secobarbital (Seconal), a barbiturate b. Diazepam (Valium), a benzodiazepine c. Midazolam (Versed), a benzodiazepine d. Eszopiclone (Lunesta), a nonbenzodiazepine sleep aid

ANS: D Eszopiclone (Lunesta) is one of the newest prescription hypnotics to be approved for long-term use in treatment of insomnia. Barbiturates and benzodiazepines are not appropriate for long-term treatment of insomnia; midazolam is used for procedural (moderate) sedation. DIF: COGNITIVE LEVEL: Understanding (Comprehension) REF: p. 193 TOP: NURSING PROCESS: Planning

5. A patient has been taking the monoamine oxidase inhibitor (MAOI) phenelzine (Nardil) for 6 months. The patient wants to go to a party and asks the nurse, "Will just one beer be a problem?" Which advice from the nurse is correct? a. "You can drink beer as long as you have a designated driver." b. "Now that you've had the last dose of that medication, there will be no further dietary restrictions." c. "If you begin to experience a throbbing headache, rapid pulse, or nausea, you'll need to stop drinking." d. "You need to avoid all foods that contain tyramine, including beer, while taking this medication."

ANS: D Foods containing tyramine, such as beer and aged cheeses, should be avoided while a patient is taking an MAOI. Drinking beer while taking an MAOI may precipitate a dangerous hypertensive crisis. The other options are incorrect. DIF: COGNITIVE LEVEL: Analyzing (Analysis) REF: p. 258 TOP: NURSING PROCESS: Implementation

1. The nurse is writing a nursing diagnosis for a plan of care for a patient who has been newly diagnosed with type 2 diabetes. Which statement reflects the correct format for a nursing diagnosis? a. Anxiety b. Anxiety related to new drug therapy c. Anxiety related to anxious feelings about drug therapy, as evidenced by statements such as "I'm upset about having to test my blood sugars." d. Anxiety related to new drug therapy, as evidenced by statements such as "I'm upset about having to test my blood sugars."

ANS: D Formulation of nursing diagnoses is usually a three-step process. "Anxiety" is missing the "related to" and "as evidenced by" portions of defining characteristics. "Anxiety related to new drug therapy" is missing the "as evidenced by" portion of defining characteristics. The statement beginning "Anxiety related to anxious feelings" is incorrect because the "related to" section is simply a restatement of the problem "anxiety," not a separate factor related to the response. DIF: COGNITIVE LEVEL: Understanding (Comprehension) REF: p. 7 TOP: NURSING PROCESS: Nursing Diagnosis

1. . During a fishing trip, a patient pierced his finger with a large fishhook. He is now in the emergency department to have it removed. The nurse anticipates that which type of anesthesia will be used for this procedure? a. No anesthesia b. Topical benzocaine spray on the area c. Topical prilocaine (EMLA) cream around the site d. Infiltration of the puncture wound with lidocaine

ANS: D Infiltration anesthesia is commonly used for minor surgical procedures. It involves injecting the local anesthetic solution intradermally, subcutaneously, or submucosally across the path of nerves supplying the area to be anesthetized. The local anesthetic may be administered in a circular pattern around the operative field. The other types are not appropriate for this injury. This is a painful procedure; therefore, the option of "no anesthesia" is incorrect. DIF: COGNITIVE LEVEL: Applying (Application) REF: p. 176 TOP: NURSING PROCESS: Planning

4. A patient has just been told that she has the genetic markers for a severe type of breast cancer. After the patient meets with the physician, the patient's daughter asks the nurse, "What did the doctor tell my mother? She seems upset." What is the nurse's best response? a. "I'm sorry, but I'm not allowed to discuss that." b. "The physician will discuss this with you." c. "It seems that your mother has the genetic markers for a type of breast cancer." d. "This is information that your mother will need to discuss with you."

ANS: D Maintaining privacy and confidentiality is of utmost importance during genetic testing and counseling. The patient (not the nurse or the physician) is the one who decides whether to include or exclude any family members from the discussion and from knowledge of the results of genetic testing. Telling the patient's daughter that you are "not allowed" to discuss the matter would cause more anxiety. Telling the daughter about the genetic markers would be a violation of the patient's privacy. DIF: COGNITIVE LEVEL: Applying (Application) REF: pp. 100-101 TOP: NURSING PROCESS: Implementation

3. The nurse has been monitoring the patient's progress on a new drug regimen since the first dose and documenting the patient's therapeutic response to the medication. Which phase of the nursing process do these actions illustrate? a. Nursing diagnosis b. Planning c. Implementation d. Evaluation

ANS: D Monitoring the patient's progress, including the patient's response to the medication, is part of the evaluation phase. Planning, implementation, and nursing diagnosis are not illustrated by this example. DIF: COGNITIVE LEVEL: Understanding (Comprehension) REF: pp. 13-14 TOP: NURSING PROCESS: Evaluation

3. A patient has been taking naltrexone (ReVia) as part of the treatment for addiction to heroin. The nurse expects that the naltrexone will have which therapeutic effect for this patient? a. Naltrexone prevents the cravings for opioid drugs. b. Naltrexone works as a safer substitute for the heroin until the patient completes withdrawal. c. The patient will experience flushing, sweating, and severe nausea if he takes heroin while on naltrexone. d. If opioid drugs are used while taking naltrexone, euphoria is not produced; thus, the opioid's desired effects are lost.

ANS: D Naltrexone works to eliminate the euphoria that occurs with opioid drug use; therefore, the reinforcing effect of the drug is lost. DIF: COGNITIVE LEVEL: Understanding (Comprehension) REF: p. 279 TOP: NURSING PROCESS: Evaluation

12. The patient is complaining of a headache and asks the nurse which over-the-counter medication form would work the fastest to help reduce the pain. Which medication form will the nurse suggest? a. A capsule b. A tablet c. An enteric-coated tablet d. A powder

ANS: D Of the types of oral medications listed, the powder form would be absorbed the fastest, thus having a faster onset. The tablet, the capsule, and, finally, the enteric-coated tablet would be absorbed next, in that order. DIF: COGNITIVE LEVEL: Applying (Application) REF: p. 20 TOP: NURSING PROCESS: Implementation

13. The drug nalbuphine (Nubain) is an agonist-antagonist (partial agonist). The nurse understands that which is a characteristic of partial agonists? a. They have anti-inflammatory effects. b. They are given to reverse the effects of opiates. c. They have a higher potency than agonists. d. They have a lower dependency potential than agonists.

ANS: D Partial agonists such as nalbuphine are similar to the opioid agonists in terms of their therapeutic indications; however, they have a lower risk of misuse and addiction. They do not have anti-inflammatory effects, nor are they given to reverse the effects of opiates. They do not have a higher potency than agonists. DIF: COGNITIVE LEVEL: Understanding (Comprehension) REF: p. 156 TOP: NURSING PROCESS: Assessment

The drug nalbuphine (Nubain) is an agonist-antagonist (partial agonist). The nurse understands that which is a characteristic of partial agonists? a. They have anti-inflammatory effects. b. They are given to reverse the effects of opiates. c. They have a higher potency than agonists. d. They have a lower dependency potential than agonists.

ANS: D Partial agonists such as nalbuphine are similar to the opioid agonists in terms of their therapeutic indications; however, they have a lower risk of misuse and addiction. They do not have anti-inflammatory effects, nor are they given to reverse the effects of opiates. They do not have a higher potency than agonists. DIF: COGNITIVE LEVEL: Understanding (Comprehension) REF: p. 156 TOP: NURSING PROCESS: Assessment MSC: NCLEX: Physiological Integrity: Pharmacological and Parenteral Therapies

5. When administering a neuromuscular blocking drug, the nurse needs to remember which principle? a. It is used instead of general anesthesia during surgery. b. Only skeletal muscles are paralyzed; respiratory muscles remain functional. c. It causes sedation and pain relief while allowing for lower doses of anesthetics. d. Artificial mechanical ventilation is required because of paralyzed respiratory muscles.

ANS: D Patients receiving neuromuscular blocking drugs require artificial mechanical ventilation because of the resultant paralysis of the respiratory muscles. In addition, they do not cause sedation or pain relief. They are used along with, not instead of, general anesthesia during surgery. DIF: COGNITIVE LEVEL: Applying (Application) REF: p. 178 TOP: NURSING PROCESS: Implementation

11. A patient tells the nurse that he likes to drink kava herbal tea to help him relax. Which statement by the patient indicates that additional teaching about this herbal product is needed? a. "I will not drink wine with the kava tea." b. "If I notice my skin turning yellow, I will stop taking the tea." c. "I will not take sleeping pills if I have this tea in the evening." d. "I will be able to drive my car after drinking this tea."

ANS: D Patients should not drive after drinking this tea because it may cause sedation. Kava tea may cause skin discoloration (with long-term use). In addition, it must not be taken with alcohol, barbiturates, and psychoactive drugs. DIF: COGNITIVE LEVEL: Applying (Application) REF: p. 192 TOP: NURSING PROCESS: Evaluation

7. A patient with the diagnosis of schizophrenia is hospitalized and is taking a phenothiazine drug. Which statement by this patient indicates that he is experiencing a common adverse effect of phenothiazines? a. "I can't sleep at night." b. "I feel hungry all the time." c. "Look at how red my hands are." d. "My mouth has been so dry lately."

ANS: D Phenothiazines produce anticholinergic-like adverse effects of dry mouth, urinary hesitancy, and constipation. DIF: COGNITIVE LEVEL: Applying (Application) REF: p. 270 TOP: NURSING PROCESS: Evaluation

2. A patient who started taking orlistat (Xenical) 1 month ago calls the clinic to report some "embarrassing" adverse effects. She tells the nurse that she has had episodes of "not being able to control my bowel movements." Which statement is true about this situation? a. These are expected adverse effects that will eventually diminish. b. The patient will need to stop this drug immediately if these adverse effects are occurring. c. The patient will need to increase her fat intake to prevent these adverse effects. d. The patient will need to restrict fat intake to less than 30% to help reduce these adverse effects.

ANS: D Restricting dietary intake of fat to less than 30% of total calories can help reduce some of the GI adverse effects, which include oily spotting, flatulence, and fecal incontinence. The other options are incorrect. DIF: COGNITIVE LEVEL: Applying (Application) REF: p. 208 TOP: NURSING PROCESS: Evaluation

5. When the nurse teaches a skill such as self-injection of insulin to the patient, what is the best way to set up the teaching/learning session? a. Provide written pamphlets for instruction. b. Show a video, and allow the patient to practice as needed on his own. c. Verbally explain the procedure, and provide written handouts for reinforcement. d. After demonstrating the procedure, allow the patient to do several return demonstrations.

ANS: D Return demonstration allows the nurse to evaluate the patient's newly learned skills. The techniques in the other options are incorrect because those suggestions do not allow for evaluation of the patient's technique. DIF: COGNITIVE LEVEL: Applying (Application) REF: p. 75 TOP: NURSING PROCESS: Implementation

2. An 18-year-old basketball player fell and twisted his ankle during a game. The nurse will expect to administer which type of analgesic? a. Synthetic opioid, such as meperidine (Demerol) b. Opium alkaloid, such as morphine sulfate c. Opioid antagonist, such as naloxone HCL (Narcan) d. Nonopioid analgesic, such as indomethacin (Indocin)

ANS: D Somatic pain, which originates from skeletal muscles, ligaments, and joints, usually responds to nonopioid analgesics such as nonsteroidal anti-inflammatory drugs (NSAIDs). The other options are not the best choices for somatic pain. DIF: COGNITIVE LEVEL: Applying (Application) REF: pp. 145-146 TOP: NURSING PROCESS: Assessment

An 18-year-old basketball player fell and twisted his ankle during a game. The nurse will expect to administer which type of analgesic? a. Synthetic opioid, such as meperidine (Demerol) b. Opium alkaloid, such as morphine sulfate c. Opioid antagonist, such as naloxone HCL (Narcan) d. Nonopioid analgesic, such as indomethacin (Indocin)

ANS: D Somatic pain, which originates from skeletal muscles, ligaments, and joints, usually responds to nonopioid analgesics such as nonsteroidal anti-inflammatory drugs (NSAIDs). The other options are not the best choices for somatic pain. DIF: COGNITIVE LEVEL: Applying (Application) REF: pp. 145-146 TOP: NURSING PROCESS: Assessment MSC: NCLEX: Physiological Integrity: Basic Care and Comfort

3. The nurse is explaining the Human Genome Project to colleagues. Which of these is the main purpose of the Human Genome Project? a. The study of genetic diseases. b. The study of genetic traits in humans. c. The discovery new genetic diseases. d. To describe the entire genome of a human being.

ANS: D The Human Genome Project was undertaken to describe in detail the entire genome of a human being. The other options do not describe the Human Genome Project. DIF: COGNITIVE LEVEL: Remembering (Knowledge) REF: p. 98 TOP: NURSING PROCESS: General

3. When assessing patients in the preoperative area, the nurse knows that which patient is at a higher risk for an altered response to anesthesia? a. The 21-year-old patient who has never had surgery before b. The 35-year-old patient who stopped smoking 8 years ago c. The 40-year-old patient who is to have a kidney stone removed d. The 82-year-old patient who is to have gallbladder removal

ANS: D The elderly patient is more affected by anesthesia than the young or middle-aged adult patient because of the effects of aging on the hepatic, cardiac, respiratory, and renal systems. DIF: COGNITIVE LEVEL: Applying (Application) REF: p. 173 TOP: NURSING PROCESS: Assessment

1. A 25-year-old woman is visiting the prenatal clinic and shares with the nurse her desire to go "natural" with her pregnancy. She shows the nurse a list of herbal remedies that she wants to buy so that she can "avoid taking any drugs." Which statement by the nurse is correct? a. "Most herbal remedies are not harmful and are safe for use during pregnancy." b. "Please read each label carefully before use to check for cautionary warnings." c. "Keep in mind that products from different manufacturers are required to contain consistent amounts of the herbal products." d. "It's important to remember that herbal remedies do not have proven safety ratings for pregnant women."

ANS: D The fact that a drug is an herbal or a dietary supplement does not mean that it can be safely administered to children, infants, or pregnant or lactating women. Many herbal products have not been tested for safety during pregnancy. Simply reading the labels may not provide enough information for use during pregnancy. Last, manufacturers of herbal products are not required to guarantee the reliability of the contents. DIF: COGNITIVE LEVEL: Applying (Application) REF: p. 93 TOP: NURSING PROCESS: Implementation

8. A patient has been taking the selective serotonin reuptake inhibitor (SSRI) sertraline (Zoloft) for about 6 months. At a recent visit, she tells the nurse that she has been interested in herbal therapies and wants to start taking St. John's wort. Which response by the nurse is appropriate? a. "That should be no problem." b. "Good idea! Hopefully you'll be able to stop taking the Zoloft." c. "Be sure to stop taking the herb if you notice a change in side effects." d. "Taking St. John's wort with Zoloft may cause severe interactions and is not recommended."

ANS: D The herbal product St. John's wort must not be used with SSRIs. Potential interactions include confusion, agitation, muscle spasms, twitching, and tremors. The other responses by the nurse are inappropriate. DIF: COGNITIVE LEVEL: Analyzing (Analysis) REF: p. 260 TOP: NURSING PROCESS: Implementation

10. The nurse is administering medications to the patient who is in renal failure resulting from end-stage renal disease. The nurse is aware that patients with kidney failure would most likely have problems with which pharmacokinetic phase? a. Absorption b. Distribution c. Metabolism d. Excretion

ANS: D The kidneys are the organs that are most responsible for drug excretion. Renal function does not affect the absorption and distribution of a drug. Renal function may affect metabolism of drugs to a small extent. DIF: COGNITIVE LEVEL: Applying (Application) REF: p. 26 TOP: NURSING PROCESS: Assessment

4. A patient is undergoing abdominal surgery and has been anesthetized for 3 hours. Which nursing diagnosis would be appropriate for this patient? a. Anxiety related to the use of an anesthetic b. Risk for injury related to increased sensorium from general anesthesia c. Decreased cardiac output related to systemic effects of local anesthesia d. Impaired gas exchange related to central nervous system depression produced by general anesthesia

ANS: D The nursing diagnosis of impaired gas exchange is appropriately worded for this patient. Anxiety would not be appropriate while the patient is in surgery. Sensorium would be decreased during surgery, not increased. Cardiac output is affected by general anesthesia, not local anesthesia. DIF: COGNITIVE LEVEL: Applying (Application) REF: p. 183 TOP: NURSING PROCESS: Nursing Diagnosis

4. The patient has been taking an over-the-counter (OTC) acid-reducing drug because he has had "stomach problems" for several months. He tells the nurse that the medicine helps as long as he takes it, but once he stops it, the symptoms return. Which statement by the nurse is the best advice for this patient? a. "The over-the-counter drug has helped you, so you should continue to take it." b. "The over-the-counter dosage may not be strong enough. You should be taking prescription-strength for best effects." c. "For best results, you need to watch what you eat in addition to taking this drug." d. "Using this drug may relieve your symptoms, but it does not address the cause. You should be seen by your health care provider."

ANS: D The use of OTC drugs may postpone effective management of chronic disease states and may delay treatment of serious or life-threatening disorders because these drugs may relieve symptoms without necessarily addressing the cause of the disorder. The other options do not address the need to investigate the cause of the symptoms and are incorrect. DIF: COGNITIVE LEVEL: Applying (Application) REF: p. 87 TOP: NURSING PROCESS: Planning

11. During a routine appointment, a patient with a history of seizures is found to have a phenytoin (Dilantin) level of 23 mcg/mL. What concern will the nurse have, if any? a. The patient is at risk for seizures because the drug level is not at a therapeutic level. b. The patient's seizures should be under control because this is a therapeutic drug level. c. The patient's seizures should be under control if she is also taking a second antiepileptic drug. d. The drug level is at a toxic level, and the dosage needs to be reduced.

ANS: D Therapeutic drug levels for phenytoin are usually 10 to 20 mcg/mL (see Table 14-6). The other options are incorrect. DIF: COGNITIVE LEVEL: Analyzing (Analysis) REF: p. 224 TOP: NURSING PROCESS: Evaluation

4. A patient has been treated with antiparkinson medications for 3 months. What therapeutic responses should the nurse look for when assessing this patient? a. Decreased appetite b. Gradual development of cogwheel rigidity c. Newly developed dyskinesias d. Improved ability to perform activities of daily living

ANS: D Therapeutic responses to antiparkinson agents include an improved sense of well-being, improved mental status, increased appetite, increased ability to perform activities of daily living and to concentrate and think clearly, and less intense parkinsonian manifestations. DIF: COGNITIVE LEVEL: Applying (Application) REF: p. 247 TOP: NURSING PROCESS: Evaluation

3. When given a scheduled morning medication, the patient states, "I haven't seen that pill before. Are you sure it's correct?" The nurse checks the medication administration record and verifies that it is listed. Which is the nurse's best response? a. "It's listed here on the medication sheet, so you should take it." b. "Go ahead and take it, and then I'll check with your doctor about it." c. "It wouldn't be listed here if it were not ordered for you!" d. "Let me check on the order first before you take it."

ANS: D When giving medications, the nurse should always listen to and honor any concerns or doubts expressed by the patient. If the patient doubts an order, the nurse should check the written order and/or check with the prescriber. The other options illustrate that the nurse is not listening to the patient's concerns. DIF: COGNITIVE LEVEL: Applying (Application) REF: p. 69 TOP: NURSING PROCESS: Planning

7. When reviewing the mechanism of action of a specific drug, the nurse reads that the drug works by selective enzyme interaction. Which of these processes describes selective enzyme interaction? a. The drug alters cell membrane permeability. b. The drug's effectiveness within the cell walls of the target tissue is enhanced. c. The drug is attracted to a receptor on the cell wall, preventing an enzyme from binding to that receptor. d. The drug binds to an enzyme molecule and inhibits or enhances the enzyme's action with the normal target cell.

ANS: D With selective enzyme interaction, the drug attracts the enzymes to bind with the drug instead of allowing the enzymes to bind with their normal target cells. As a result, the target cells are protected from the action of the enzymes. This results in a drug effect. The actions described in the other options do not occur with selective enzyme interactions. DIF: COGNITIVE LEVEL: Understanding (Comprehension) REF: p. 29 TOP: NURSING PROCESS: General

The nurse is caring for a patient whose seizures are characterized by a 10- to 30-second loss of consciousness and mild, symmetric eye blinking. Which seizure type does this most closely illustrate?

Absence

The nurse is preparing to give ethosuximide [Zarontin]. The nurse understands that this drug is only indicated for which seizure type?

Absence

Which medication does the nurse anticipate for the patient admitted with an acetaminophen overdose? Atropine Acetylcysteine Glucocorticoid Antihistamine

Acetylcysteine Liver damage is associated with acetaminophen and can be minimized with use of acetylcysteine. If it is given within 8 to 10 hours of acetaminophen, serious liver damage is avoided.

While conducting a health history for an older adult patient with heart failure, the patient tells the nurse, I have chronic constipation." The nurse suspects this gastrointestinal complaint is caused by which class of drugs? A) Nonsteroidal antiinflammatory drugs B) Calcium channel blockers C) Potassium-sparing diuretics D) Anticoagulants

B) Calcium channel blockers

Knowing that the albumin in neonates and infants has a lower binding capacity for medications, the nurse anticipates the health care provider will perform which action to minimize the risk of toxicity? A) Increase the amount of drug given. B) Decrease the amount of drug given. C) Administer the medication intravenously. D) Shorten the time interval between doses.

B) Decrease the amount of drug given.

The nurse working in a prenatal clinic recognizes that the safety or potential harm of drug therapy during pregnancy relates to which factor? A) Maternal blood type B) Drug properties C) Fetal sex D) Diet of the mother

B) Drug properties

Which statement does the nurse include when teaching a patient about antipsychotic drug therapy? (Select all that apply.) A. "Restrict the use of antipsychotic drugs to 3 months to prevent the development of addiction." B. "Dilute oral preparations in fruit juice to improve their palatability." C. "Store oral preparations in a dark area." D. "Do not make skin contact with these drugs; flush the affected area with water if a spill occurs." E. "Take an over-the-counter sleep aid if you have trouble falling asleep at night."

B, C, D Patients should be informed that antipsychotic drugs do not cause addiction and that they should be taken as prescribed. Patients should be instructed to avoid all drugs with anticholinergic properties, including the antihistamines and certain over-the-counter sleep aids to prevent drug interactions. All other statements are appropriate to include in teaching the patient about the use of antipsychotic medications.

The nurse identifies which antidepressant as effective in the treatment of generalized anxiety disorder (GAD)? (Select all that apply.) A. Fluoxetine (Prozac) B. Venlafaxine (Effexor XR) C. Paroxetine (Paxil) D. Escitalopram (Lexapro) E. Duloxetine (Cymbalta)

B, C, D, E Fluoxetine (Prozac) is not approved for the treatment of generalized anxiety disorder. All the other medications listed are approved for the treatment of GAD.

A nurse is caring for several patients. In which patient is it appropriate to use the drug chlorpromazine (Thorazine)? (Select all that apply.) A. An 85-year-old man with Alzheimer's disease B. A 78-year-old man with intractable hiccups C. A 76-year-old woman with severe dementia D. A 48-year-old woman with schizoaffective disorder E. A 30-year-old man with anxiety and depression

B, D The primary indications for chlorpromazine, a first-generation antipsychotic agent, are schizophrenia and other psychotic disorders. It may also be used for schizoaffective disorder, bipolar disorder, suppression of emesis, and relief of intractable hiccups. Antipsychotics are not used for dementia because of increased mortality. Chlorpromazine is not a primary treatment for Alzheimer's disease or depression.

The nurse is working with the multidisciplinary healthcare team to optimize the care of a patient with schizophrenia. Which concepts will guide the nursing care of this patient? (Select all that apply.) A. The second-generation antipsychotics generally are more effective than the first-generation agents. B. Most antipsychotic agents increase the risk of mortality in elderly patients with dementia. C. Antipsychotic depot preparations carry a greater risk of neuroleptic malignant syndrome. D. The lipid levels of patients receiving second-generation antipsychotics should be monitored. E. Schizophrenia is characterized by disordered thinking and loss of touch with reality.

B, D, E The first- and second-generation antipsychotics are considered equally effective, even though the second-generation agents are more widely used today. Most antipsychotics should be avoided in elderly patients with dementia because of increased mortality. Antipsychotic depot preparations are effective for the long-term control of schizophrenia and do not have an increased risk of side effects. Second-generation antipsychotics may cause weight gain, diabetes, and dyslipidemia. Schizophrenia is characterized by disordered thinking and loss of touch with reality.

The healthcare provider orders amantadine 100 mg PO daily. Available is amantadine 10 mg/mL syrup. How many mL will the nurse administer? A. 1 mL B. 10 mL C. 100 mL D. 0.1 mL

B. 10 mL The available concentration of amantadine is 10 mg/mL. The ordered dose of 100 mg would equal 10 mL of syrup. 100 mg/10 = 10 mL.

The nurse is seeing several patients in the outpatient clinic today. Which patient most requires the nurse's immediate attention? A. A female patient with BPD who takes valproic acid (Depakene) and who reports nausea and vomiting. B. A male patient with BPD who takes lithium and who has a lithium level of 1.6 mEq/L. C. A male patient with depression who takes fluoxetine (Prozac) and who reports sexual dysfunction. D. A female patient with schizophrenia who takes haloperidol (Haldol) and who has a blood pressure of 102/72 mm Hg.

B. A male patient with BPD who takes lithium and who has a lithium level of 1.6 mEq/L. Lithium levels above 1.5 mEq/L should be reported, because this level may indicate impending serious toxicity. The other findings may be side effects of the drugs the patients are taking, but they are not priority problems.

Which statement should the nurse include in the teaching plan for a patient being started on levodopa/carbidopa [Sinemet] for newly diagnosed Parkinson's disease? A. Take the medication on a full stomach. B. Change positions slowly. C. The drug may cause the urine to be very dilute. D. Carbidopa has many adverse effects.

B. Change positions slowly. Postural hypotension is common early in treatment, so the patient should be instructed to change positions slowly. Administration with meals should be avoided, if possible, because food delays the absorption of the levodopa component. If the patient is experiencing side effects of nausea and vomiting, administration with food may need to be considered. The levodopa component in Sinemet may darken the color of the urine. Carbidopa has no adverse effects of its own.

A patient with Parkinson's disease is prescribed pramipexole [Mirapex] along with his levodopa/carbidopa [Sinemet]. Which symptom is most likely a manifestation of an adverse effect of these drugs when given together? A. Diarrhea B. Dyskinesia C. Wheezing D. Headache

B. Dyskinesia When pramipexole is combined with the levodopa component in Sinemet, patients are most likely to experience symptoms of dyskinesias, such as dyskinesia (head bobbing) and orthostatic hypotension. The other effects are not common responses to these drugs.

The nurse in the emergency department is caring for a patient with a suspected overdose of diazepam (Valium). Which agent is most likely to be administered to reverse the effects of diazepam? A. Naloxone (Narcan) B. Flumazenil (Romazicon) C. Acetylcysteine (Mucomyst) D. Vitamin K

B. Flumazenil (Romazicon) Flumazenil (Romazicon), a benzodiazepine receptor antagonist, is the treatment of choice for overdose of the benzodiazepine diazepam (Valium). Naloxone (Narcan) is used to reverse opioid overdose. Acetylcysteine (Mucomyst) is used to reverse acetaminophen (Tylenol) overdose. Vitamin K is used to reverse warfarin toxicity.

The nurse identifies which most common serious adverse effect of TCA therapy? A. Sedation B. Orthostatic hypotension C. Skin rash D. Sexual dysfunction

B. Orthostatic hypotension Orthostatic hypotension is the most common adverse effect of tricyclic antidepressant therapy.

The nurse is caring for a patient in the emergency department who reports the onset of agitation, confusion, muscle twitching, diaphoresis, and fever about 12 hours after beginning a new prescription for escitalopram (Lexapro). Which is the most likely explanation for these symptoms? A. Depressive psychosis B. Serotonin syndrome C. Escitalopram overdose D. Cholinergic crisis

B. Serotonin syndrome Serotonin syndrome can occur within 2 to 72 hours after initiation of treatment with an SSRI. The symptoms include altered mental status, incoordination, myoclonus, hyperreflexia, excessive sweating, tremor, and fever.

The nurse has just administered the first dose of haloperidol (Haldol) to a patient with schizophrenia. Which finding, if present, is the most important for the nurse to report to the healthcare provider before administering the next dose of medication? A. Dry mouth B. Temperature of 101 °F C. BP of 104/72 mm Hg D. Drowsiness

B. Temperature of 101 °F Sudden high fever is a symptom of neuroleptic malignant syndrome, a rare but serious complication of high-potency, first-generation antipsychotics, such as haloperidol. The other findings are potential side effects of the drug but would not necessarily need to be reported to the healthcare provider.


Kaugnay na mga set ng pag-aaral

Psychobiology and Psychopharmacology

View Set

Water, Electrolytes, and Carbohydrates

View Set

Property II -- Practice Questions

View Set